Pathophysiology Final 1

You might also like

Download as pdf or txt
Download as pdf or txt
You are on page 1of 162

1. Pathophysiology of the cell - causes and mechanisms of cell injury.

Role of
free radicals.

Cells exist in homeostasis. Where they are subject to stress they attempt to adapt. Where they
are unable to adapt they become ‘Injured cells’. Common methods of adaptation include
hypertrophy (increase in cellular size), Hyperplasia (increase in cellular number), atrophy
(shrinkage of cell by loss of substance) and metaplasia (replacing of cells by different cellular
types, think tough calluses of skin).
The most common cause of cellular injury is Hypoxia, or oxygen deficiency, as it prevents
aerobic oxidative respiration, and the conversion of carbohydrates into ATP. Hypoxia can often
be caused by Ischemia, lack of blood supply to an affected area, but also as a result of
pneumonia (lack of oxygen in blood), anemia (reduction of oxygen carrying capacity of
blood) and CO poisoning.

Other important causes include:


• Chemical agents
• Such as high glucose, salt, oxygen, CO or other toxic agents
• Temperature
• Infectious agents (Biological)

VSM
• Immunologic reactions
• Physical agents
• Nutritional imbalances
• Genetic defects
• Aging
These problems cause the cell to become nonfunctional, this dysfunction leads to cellular injury,
we’ll now consider the mechanism of cell injury. The following mechanisms of cell injury are
the most common, and usually combined:
• Depletion of ATP (failure of energy --- Necrosis)
• Causes; Influx of Sodium and Calcium ions and osmotic swelling due to failure of
ATP dependent ion pumps
• ATP depleted cells being anaerobic metabolism to derive energy from glycogen
(Glycolysis)
• Results in decrease of pH, creating acidic environment and damaging
enzymes.
• Causes clumping of nuclear chromatin, pyknosis, leading to cellular
necrosis.
• Damage to Mitochondria (leads to ATP depletion, high Ca2+ and ROS common causes)
• Influx of calcium (activates enzymes with damaging cellular effects)
• Accumulation of Oxygen-Derived free radicals (Damages membrane and DNA)
• Can lead to
• Lipid peroxidation of membranes
• DNA fragmentation
• Protein cross-linking
• Defects in membrane permeability
• Damage to DNA and proteins
• Disruption to cell membrane
Free radicals are molecules with an unpaired electron in their outer shell. Common biological
free radicals are:
• Superoxide anion (O2- .)
• Nitric oxide (NO)
• Hydroxyl radical (HO.)
• Alkoxy radical (RO.)
• Peroxyl radical (ROO.)
• and the simple oxygen molecule (O2)
Free radicals can be generated under normal physiological conditions, but these are normally
dealt with by Antioxidants. Where we have high levels of free radicals caused by pathological
processes, the body is unable to deal with them. Common causes of free radical formation
include:
• Cellular respiration (this is the normal physiological process)
• Ionizing radiation - splits H2O into an electron and a Hydroxyl radical
• Chemical cell injury - metabolism of exogenous chemicals can result in free radical
formation
• Metals - Iron can (via Fenton reaction) be converted to a Hydroxyl free radical
• This is pathologically important as it is the mechanism of tissue damage in the
condition Hemochromatosis, where the body has high iron levels.
So, how do these free radicals actually cause damage the three main mechanisms of damage are:
• Lipid peroxidation
• Hydroxyl radicals attack the lipids in the cell membrane and cause a chain
reaction which causes damage to the cell membrane
• Protein oxidation (cross links of proteins)
• DNA Oxidation/Fragmentation
• Can cause Oncogenesis
The body has a number of defences against free radicals, these include:

VSM
• Antioxidants
• E.g. Vitamin A, C and E, all donate electrons
• Metal Carrier proteins
• E.g. Transferrin, Ceruloplasmin, all hide the free radical away
• Enzymes
• Superoxide dismutase - nullifies superoxide
• Catalase - nullifies hydrogen peroxide
• Glutathione peroxidase - nullifies hydroxyl radical
A clinical example is the conversion of Carbon Tetrachloride into Trichloromethyl radical which
can go on to cause liver swelling, decreased protein synthesis, and, due to hepatocyte
dysfunction, fatty liver.

2. Pathophysiology of the cell injury - mechanisms of cell death. Necrosis, apoptosis.

Where cells are subject to stress they attempt to adapt. Where they are unable to adapt they
become ‘Injured cells’. Common methods of adaptation include hypertrophy (increase in
cellular size), Hyperplasia (increase in cellular number), atrophy (shrinkage of cell by loss of
substance) and Metaplasia (replacing of cells by different cellular types, think tough calluses of
skin).
Cells can be injured reversibly or irreversibly. The chain of cellular changes consists of the four
stages:
• Adaptation
• Reversible changes
• Irreversible changes
• Autolysis and heterolysis
Reversible cell injury consists of two primary morphological changes, cellular swelling and fatty
change.
Irreversible change, is in effect, cell death, which can either be Necrosis (unplanned cell death)
or Apoptosis, two processes which are morphologically distinct.
Where the result is Necrosis, there are different types of necrosis based on different etiologies:
• Coagulation necrosis (result of hypoxia)
• Liquefactive necrosis (enzyme digestion of necrotic cells, hypoxic damage of CNS)
• Caseous necrosis (specific bacterial diseases e.g. TB, caseous lymphadenitis)
• Gangrenous necrosis (usually ischemic necrosis of extremities)
• Fat necrosis (necrosis a product of inflammation, Vit E deficiency, trauma)
Apoptosis on the is the programmed death of cells. Importantly, inflammation does not
occur.
It is a highly regulated process, as once it has begun cell death is inevitable. There are two key
pathways:
• Intrinsic pathway
• Activated by intracellular signals generated when cells are stressed and depends
on the release of proteins from the intermembrane space of the mitochondria,
hence also known as the mitochondrial pathway.
• Extrinsic pathway
• Activated by extracellular ligands binding to cell surface ‘death receptors’
(WHAT A NAME).
The extrinsic pathway sees activation via a T-cell targeting a cell and producing a signaling
protein called Fas ligand. The Fas ligand binds to its receptor, Fas, on target cells, which in turn
binds to proteins that link the receptor to procaspase-8 molecules. These aggregate and
cleave one another, initiating a proteolytic cascade, leading to apoptosis.
The intrinsic pathway depends on mitochondrial. When cells are stressed, damaged or
abnormal, pro-apoptotic signals are released which induce mitochondria to release

VSM
cytochrome c into the cytosol, where it binds and activates protein Apaf-1. This causes Apaf-
1 to aggregate into an apoptosome, which recruits procaspase-9 molecules, which activate
and trigger a caspase cascade, leading to apoptosis.
As the caspases start to break down the cell, it shrinks and sends out distress signals which
attract macrophages to degrade the shrunken cells leaving no trace and preventing any
inflammatory response. We can view Apoptosis in the following stages:
• Loss of mitochondrial function and caspase activation
• Fragmentation of DNA by Endonucleases
• Shrinkage of nuclear and cell volumes due to destruction of cytoskeleton
• Cell membrane undergoes blebbing where proteins are degraded
• Formation and phagocyte removal
Apoptosis is an integral part of a healthy organism. In neonates, during development, tissue
present between the digits is removed by apoptosis. Furthermore, to prevent cancers and
malformations, damaged cells must be successfully removed. However, AIDS and
Alzheimer’s are examples of conditions that disrupt the Apoptosis process and cause increased
apoptosis, leading to pathology.

This initiates an inflammatory response around the cell, thereby preventing effective
phagocytosis as phagocytes are unable to reach the site of the cell death. For this reason dead
cells may have to be removed surgically (e.g. gangrene).
The key cellular changes are:
Karyolysis - where the chromatin of the nucleus fades due to loss of DNA by degradation
Pyknosis - where the nucleus shrinks and chromatin condenses
Karyorrhexis - where the shrunken nucleus fragments
Plasma membrane appears discontinuous caused by cell blebbing.
Apoptosis on the other hand is the programmed death of cells. Importantly, inflammation
does not occur.
Apoptosis can be initiated either through the intrinsic pathway, as it senses cell stress, for
instance a virus, or extrinsic pathway, as it has received a signal from other cells. Both
pathways activate caspases, which are enzymes that degrade proteins.
As the caspases start to break down the cell, it shrinks and sends out distress signals which
attract macrophages to degrade the shrunken cells leaving no trace and preventing any
inflammatory response. We can view Apoptosis in the following stages:
• Loss of mitochondrial function and caspase activation
• Fragmentation of DNA by Endonucleases
• Shrinkage of nuclear and cell volumes due to destruction of cytoskeleton
• Cell membrane undergoes blebbing where proteins are degraded
• Formation and phagocyte removal
The mechanism of external apoptosis can be be broken down as so:
• Stimulation of death receptor results in aggregation and recruitment of the Fas-
associated protein with death domain (FADD) and Caspase-8
• Caspase 8 goes on to activate other caspases within the cell
The mechanism of internal apoptosis can be viewed as the following:
• Mitochondria are engaged based on stress stimuli
• Mitochondrial membrane is made permeable and proteins are released via
transmembrane channels into the cytoplasm
• These proteins in turn go on to activate caspase-9 which trigger a caspase cascade.

VSM
3. The molecular & biochemical basis of neoplasia. Genetic changes, oncogenes &
tumor suppressor genes. Hormones, growth factors & inhibitors, and stromal,
adhesive & proteolytic proteins.
Firstly, some definitions, what is a Neoplasia. Neoplasia is quite simply ‘New Growth’,
additionally, we can divide all Neoplasia into benign and malignant (commonly known as
cancerous) tumours, while a tumour is simply the term for a swelling.
Let us now consider the first part of this question.
The molecular & biochemical basis of neoplasia
The mechanism for cancer is not overly complicated, it occurs as a result of damage to cellular
DNA, impacting one or more genes that control the cell cycle, apoptosis, response to growth
factors, or other cellular functions that control the cell cycle.
Damage to DNA can be caused by a number of factors known as carcinogens, these include:
• Environment
• Radiation, food, smoking, pollution
• Age
• Hereditary
• Including Gender and Genetics
• Acquired preneoplastic disorders (see below)
This damage results in genetic changes which impact one of two classes of normal regulatory
genes:
• Growth promoting proto-oncogenes
• These are genes that encourage normal cell division
• The term proto-oncogenes are normal genes that have the potential to become
oncogenes.
• E.g. KRAS protein
• Growth-inhibiting tumor suppressor genes
• Prevent uncontrolled cell growth either by apoptosis, DNA repair or cell cycle
regulation.
• Two classes
• Governors
• Guardians (sense genomic damage and force apoptosis)
• Important example TP53 “Guardian of the genome”
Damage to genes in these two categories lead to the formation of ‘Oncogenes’. An oncogene is
the term we give to genes that induce a transformed phenotype when expressed, which have the
potential to cause cancer.
Mutations can be in the form of point mutations, translocations, gene amplifications, deletions,
or almost any other form of genetic error.
Normal cells require growth factors, that is to say, signals from the body (be that nearby or
distant parts) to encourage them to divide. Think back to the Mitogen-activated protein kinase
(MapK) pathway. Here a ligand binds to the tyrosine kinase receptor, which goes on to stimulate
cellular division (https://tinyurl.com/yco6b2j5). Where cancer cells differ from normal cells is
they do not require growth factors to encourage them to proliferate, they can either use
autocrine signaling to produce their own growth factors, or damage to the MapK pathway
means that no growth factors are required atl all!

VSM
This means, that despite the body not giving any signals to the cell to continue dividing, it will
continue to do so.
Furthermore, to exacerbate this problem, the physiological processes of apoptosis does not
properly occur. In Neoplasms there is often an over expression of anti-apoptotic inhibitors, such
as Survivin, these anti-apoptotic inhibitors prevent the caspases from doing their job
successfully. The overexpression of such anti-apoptotic inhibitors stems from the Ubiquitin-
proteasome pathway, which in turn activates the transcription and translation of these anti-
apoptotic molecules.

4. Cellular changes in neoplasia. Differences between normal and cancer cell.


There are six key ‘Hallmarks of cancer cells’ as described by Robbins:
1. Self-sufficiency in growth signals
Normal cells require growth signals in order to divide, normal cells are not generally capable of
producing the ligand that triggers their growth, that is to say, they can stimulate the growth of
other cells, but not themselves. Cancer cells break this rule.
Many cancer cells are able to synthesize the same growth factors to which they are responsive,
for instance, glioblastomas secrete platelet-derived growth factor (PDGF) and express the PDGF
receptor!
Other cancer cells see mutations to the Tyrosine Kinase receptors, meaning they are constantly
‘on’, or so overexpressed that the slightest amount of growth factor will trigger cellular growth.
The best known example is that of mutation to the receptor for Epidermal growth factor (EGF).
The overexpression of the EGF receptor is responsible for 80% of squamous cell carcinomas of
the lung, 50% of glioblastomas and 80% to 100% of epithelial tissues of the head and neck.
2. Insensitivity to antigrowth signals
Normal cells have their cell cycle tightly controlled by tumour suppressor genes. These genes
take information from the cell and ensure that it is appropriate for the cell to divide. If it is not,
the cell will force cellular sequence (G0) or apoptosis.
In cancer cells, these tumour suppressor proteins do not function correctly, meaning that even if
the cell has serious genetic mutations, cellular reproduction will continue. The most famous
example is the Rb gene, which is described as ‘governor of the cell cycle’. Where an individual
has a homozygous mutation on the Rb gene, there is a high risk of neoplasia.

VSM
Furthermore, normal cells have what is known as ‘contact inhibition’, that is to say, upon
contact with neighbouring cells, their proliferation is stopped, this is not the case in cancer cells,
which continue aplomb!
3. Tissue invasion and metastasis
Tissue metastasis is the spread of cancer throughout the body, we can represent this as the
‘metastatic cascade;
• Clonal expansion, growth and angiogenesis

• Metastatic subclone
• Adhesion to and invasion of basement membrane
• Passage through ECM
• Intravasation (ito blood stream)
• Interaction with host lymphoid cells
• Tumor cell embolus
• Adhesion to basement membrane
• Extravasation
• Metastatic deposit
• Angiogenesis
• Tumor growth
4. Limitless replicative potential
Non-cancer cells die after a certain number of divisions due to shortened chromosomal
telomeres (Hayflick limit). However, cancer cells see the expression of the enzyme telomerase
which allows for the elongation of the telomeres and effectively allowing for indefinite cellular
growth and division (immortality).
5. Sustained angiogenesis (promoting vascular growth)
Angiogenesis is the process of forming new blood vessels. Tumors are a very dense collection of
cells, meaning a large amount of blood is required. Hypoxia within cancer cells triggers the

VSM
transcription of Vascular Endothelial growth factor (VEGF), a proangiogenic factor. The
transcription of VEGF is usually held in check by VHL, which therefore effectively acts as a
tumor suppressor, however where we see mutations in the VHL gene, often an inherited familial
mutation, we see an increased risk of neoplasia.
6. Evading apoptosis
All normal cells can undergo programed cell death, apoptosis, either via the intrinsic or extrinsic
mechanism. Both mechanisms involve caspase enzymes interacting with anti-apoptotic proteins
within the cell, once the volume of caspases has overwhelmed the volume of anti-apoptotic
proteins, apoptosis will occur.
As such, cancer cells vastly increase the number of anti-apoptotic proteins in the cell, meaning
the cell is unable to undergo apoptosis.

5. Tumor/organism relationships. Metastases. Systemic effects of neoplasia


Effects of tumor on host
The direct effects will depend on the location, in both benign and malignant tumours. The
effects can be divided into:
• Local effects
• Hormonal effects
• Systemic effects (aka Paraneoplastic syndromes) (see question 17)
The local effects of a tumour include:
• Compression
• Mechanical obstruction (e.g gut tumours producing intestinal obstruction(
• Tissue destruction (malignant tissues destroy vital structures)
• Infarction
• Ulceration
The hormonal effects are common in benign tumours on endocrine organs:
• Adenoma of islet of langerhans on pancreas can cause fatal hypoglycemia
• Thyroid adenoma-hyperthyroidism
• Parathyroid adenoma-hypercalcemia and osteoporosis
Relationship of Tumour and Immune System
The immune system is constantly trying to kill tumour cells, while the tumour cells are trying to
survive, here we will look at what each side does.
NK cells and Cytotoxic lymphocytes both recognise MHC receptors on the surface of tumour
cells and trigger an immune response, while Helper T cells also support this process by
releasing cytokines to recruit more lymphocytes and NK cells to the site of the battle! .
However, as we know tumour cells have an incredibly fast turn over, and their DNA is rapidly
altering, as such, a form of natural selection occurs, with some tumour cells evolving to avoid
the detection of these immune cells. This creates a heterogenous collection of cells within the
tumour which is harder for the immune system to deal with.
Furthermore, the tumour cells can evolve to include inhibitory molecules which suppress T-
cells, such as PDL1 which binds to a PD1 receptor on the T-cell making it inactive! In addition,
tumour cells can attract regulatory T-cells and certain myeloid cells which help to reduce the
activity of the immune system in the region.
The key systemic effects of cancer on the host are:
• Fever
• Especially in Hodgkin’s disease, renal cell carcinoma and osteogenic sarcoma
• Product of pyrogens released by tumour cell
• Anorexia and weight loss (Cachexia)
• Endocrine syndromes
• Cushing's, inappropriate antidiuresis, hypercalcemia, hypoglycemia
• Neurological syndromes
• Spinal cord motor neuropathy and sensorimotor peripheral neuropathy

VSM
• Skeletal muscle
• Dermatomyositis (inflammation of skin and muscle, associated with
inappropriate immune response)
• Hematologic syndromes
• Erythrocytosis (increased RBC mass)
• Malabsorption
• Renal syndromes - e.g. nephrotic syndrome
We describe those effects of neoplasms that are not direct as Paraneoplastic syndromes,
Cushing’s, Hypercalcemia, Hypoglycemia, Acromegaly (abnormal hands and feet) as well as
many others would be considered paraneoplastic.

6. Disturbances of peripheral circulation – arterial hyperemia. Venous hyperemia.


Stasis.
Arterial hyperemia
Definition: High blood flow in the arteries
Types (etiology): Can be physiological (e.g. exercise) or pathological (mechanical, physical such
as high temperature, chemical, biological or social).
Macroscopic effects
• Colour - redness of organ
• Volume - +
• Temperature - +
• Metabolic -
• Edema - rarely
• Size - +
Microscopic effects: increase in flow, pressure, diameter of arterial vessels
Capillary Networks: increased capillary networks
Blood volume: increased blood volume
Consequences: depends on duration, but includes - activation of local immunity, hypertrophy
and hyperplasia.
Hyperemia is in effect thought to be caused by the increased production of vasodilatory agents,
such as Carbon dioxide, Hydrogen ions, potassium, adenosine, nitric oxide and others. These act
on arterioles, causing dilation, reducing resistance, and increasing overall blood flow.
Venous hyperemia
Definition: High blood supply in the outflow of vessels
Types (etiology):Mechanical obstruction, physical change in temperature, chemical, biological
(e.g. parasites), or social (high amount of standing).
Macroscopic effects
• Colour - cyanosis
• Volume - -
• Temperature - -
• Metabolic - -
• Edema - Yes
Microscopic effects: increase in lumen diameter, tiny hemorrhages, changes in vessel structure.
Reduction in blood velocity
Capillary Networks: increased network
Blood volume: increased blood volume
Consequences: Can cause hypoxia and tissue edema
Is in effect the build up venous blood, we are talking here about increased blood in the veins.
Stasis
Stasis is a stoppage or slowdown in the flow of blood (or other fluids, but in this context we
assume blood).

VSM
Venous stasis is the condition of slow blood in the veins, often the legs. It increases the risk of
venous thrombosis (clots). I can be caused from high immobility. Symptoms are;
• Swelling
• Tight calves
• Pain during walking
• Brown-coloured skin
• Varicose veins
• Leg ulcers very resistant to treatment
In essence, the condition is similar to venous hyperemia.

7. Ischemia. Infarction.
Ischemia
Definition: Ischemia is the restriction in blood supply to tissues.
Etiology: Can be caused by:
• Embolism
• Thrombosis
• Atherosclrotic artery
• Trauma
Types:
• Chronic (O2 deficit in rest - reversible cell injury)
• Acute (Sudden O2 deficit - Angina pectoris is one such example) often not reversible
Effects: Insufficient blood supply leads to hypoxia, in highly aerobic tissues such as the heart
and the brain this can lead to irreversible damage and therefore death. The manifestations of
Ischemia can be summed up as:
• Pain
• Paleness
• Paresthesia (abnormal tingling or prickling or numbness)
• Paralysis
• Poikilothermia (change in temperature)
Important types of ischemia include:
• Cardiac ischemia - myocardium receives insufficient blood flow and can lead to a
myocardial infarction
• Brain ischemia, can lead to an ischemic stroke (cerebral infarction), where part of the
brain dies from hypoxia
Infarction
Definition: Dead tissue as a result of obstructed blood supply. Note, ischemia relates to the
reduction in blood flow, infarction relates to tissue that has deceased as a result of ischemia
(reduced blood flow).
Key types of infarction include:
• Myocardial or cerebral infarction
• Pulmonary infarction
• Bowel infarction
• Ischemic necrosis of extremities (gangrene)
Nearly 99% of all infarcts are a result of thrombotic or embolic events (see below) and result in
arterial occlusion (blocking).
Different tissues have different capacity to survive hypoxia, the brain and heart are two of the
most aerobic tissues and therefore the most susceptible to damage.
Microscopically, Infarcts can be:
• Red infarcts
• With venous occlusions, found in loose tissues (e.g. lungs).
• Also known as hemorrhagic infarcts
• White infarcts

VSM
• Affect solid organs (heart, spleen, kidneys) where the solidity of tissue limits the
nutrients
• Can occur as a result of vasoconstriction of artery.

• Less blood in infarct


Macroscopically
• Tend to be wedged shaped
• At onset, all are poorly defined and slightly hemorrhagic, become better defined with
time
• In the picture, note the red pulmonary infarct (L) and the anemic, white, infarc on the
right.
8. Thrombosis. Embolism.
A thrombosis is the formation of a blood clot inside a vessel, obstructing the flow of blood
through the vessel. A clot, or piece of clot, that travels around the body is known as an embolus
(see below).
Thrombosis may occur in the veins (venous thrombosis), arteries or heart.
• Venous thrombosis
• RBCs, platelets and fibrin make up the venous thrombosis, which attaches itself
to the endothelium
• Leads to congestion within the affected part of the body, can also cause
pulmonary embolism
• Can develop in deep vein in lower extremity, or in superficial saphenous, hepatic
and renal veins
• Arterial thrombosis
• Caused by damage to vessel wall (often rupture of atheroma, a fat-rich deposit in
the blood vessel wall), meaning thrombosis is formed.
• Can lead to hypoxia of the tissue the artery supplies, Embolization (see below),
Dissolution or organization (induce inflammation and reestablish physiological
flow).
• Heart, Aorta
An embolism is a detached intravascular solid, liquid or gaseous mass that is carried by the
blood to a site distant from its origin, 99% are displaced thrombus - thromboembolism, rarer
types include:
• Fat droplets
• Bubbles of air (the bends)
• Atheroslcerotic debris
• Tumor fragments
The consequences of an embolism depend on the location, however can include ischemic
necrosis (an infarction) of a downstream tissue due to occlusion of the vessel.
Embolisms can be classified as:
• Arterial
• Enters circulation in artery - a major cause of infarction. Strokes are caused by
an embolus lodging in the brain
• Can travel throughout the systemic circulation
• Venous

VSM
• Formed in a systemic vein which may then travel to the lungs to form a
pulmonary embolism (cannot travel into systemic circulation)
• Systemic embolism can come from Thrombi from the left side of the
heart in ay cases and travel throughout the body, causing tissue ischemia
Pulmonary thromboembolisms are relatively common, based on the normal configuration of the
venous system, all venous embolisms will end up in the lungs.
• Can cause chest pain
• Can lead to sudden death by prompting acute right heart failure if greater than 60% of
pulmonary vasculature is occluded
Other types of embolism
• Fat embolism
• Often caused by tracture to long bones
• Micro Globules of fat from marrow obstruct microvascular producing ischemia
and hemorrhage
• Symptoms 24-72 hours after trauma
• Air embolism
• Gas bubbles in circulation
• Product of obstetric procedures, chest wall injury, or deep sea diving
• Amniotic fluid embolism
• Occurs during labor or immediately postpartum
• Tears in placental membranes or uterine veins can lead to amniotic fluid
entering circulation
9. Immune disorders - essence and classification. Antibody mediated hypersensitivity
reaction type I.
A Hypersensitivity disorder is where the immune system reacts in such a way to damage the
organism rather than successfully protecting it. These disorders are classified into four different
groups, Types I-IV, which are dealt with the this and the following three questions. Here we will
consider Type I Hypersensitivity.
Type I hypersensitivity is also known as Immediate hypersensitivity, as the reaction is
instantaneous. It is the typical ‘Allergic’ response which one might think of, and is responsive to
Antihistamines, while the other three types are not! This type of sensitivity sees the generic
redness, swelling, inflammation and potentially anaphylactic shock that we associated with an
allergic reaction, but it does not involve the complement system.
As with all of the hypersensitivity reactions we will address in this section, hypersensitivity is a
two step process, with first a First Exposure (Sensitization) phase, where the body is primed to
react to the antigen, and secondly a Subsequent exposure phase, where the actual allergic
reaction occurs. We will explore this in more detail in the Pathogenesis of the disorder.
Antigens (Allergens)
• Insect venom
• Food
• Pollen
• Dust mites
Antibody
• Type I is a humoral immune response, mediated by Immunoglobulin E (IgE)
Pathogenesis
Type I hypersensitivity is a result of a genetic defect that makes T-cells over sensitive to a
certain allergen. Based on the above ‘First Exposure’ and ‘Subsequent Exposure’ model above,
here we will consider the mechanism of hypersensitivity
• First Exposure
• Antigen enters body and is picked up by Macrophages/Dendritic cells
• These carry antigen to lymph nodes which where TH-cells are overstimulated
• TH-cells become activated, which in turn activate B-cells, turning them into
plasma cells

VSM
• Active B-cells (now known as plasma cells) now produce antigen specific IgEs,
which bind to Mast Cells ‘priming’ it
• Mast cells now remain in the body, ‘geared up’ to attack the antigen
• Subsequent exposure
• Antigen enters the body once more, Mast Cells with IgE antibodies now bind to
antigen
• This binding causes the degranulation of Mast Cells, releasing lots of Histamine
and other inflammatory mediators
• Results in inflammation, bronchoconstriction, edema and urticaria

Treatment
• Adrenaline followed by corticosteroids
Diseases
• Atopic asthma
• Anaphylactic shock
• Hay fever
• Atopic dermatitis

VSM
10. Immune disorders - essence and classification. Antibody mediated
hypersensitivity reaction type II
Type II hypersensitivity reactions are also known as Cytotoxic reactions. This type of
hypersensitivity effectively sees the destruction of cells (and thereby tissues), mediated by
antibodies.
Antigen
• Often drugs, such as penicillin
• Can also be endogenous molecule (autoimmune condition)
Antibody
• IgG and IgM
Pathogenesis
• Antigen enters the body and binds to a cell
• Certain plasma cells which do not have suitable central tolerance, produce IgG and/or
IgM antibodies which then bind to the antigen on the cell
• This forms an Antigen-Antibody complex on the organism's cell
• The Antigen-Antibody complex then activates the complement system
• The complement system then triggers the hemolysis of the cell, leading to tissue damage

Diseases
• Hemolytic disease of newborn
• Hemolytic anemia
• Thrombocytopenia
• Goodpasture syndrome
• Antibodies attack certain self-antibodies, specifically found basement membrane
in lungs and kidneys

VSM
11. Immune disorders - essence and classification. Antibody mediated
hypersensitivity reaction type III.
Type III hypersensitivity is also known as Immune Complex disease as it sees the formation of
antigen-antibody immune complexes circulating around the blood stream which trigger the
complement system to kick into action, thereby damaging the blood vessels of the organism.
Antigen
• Can be drugs, endogenous or exogenous molecules or proteins
Antibody
• IgG and IgM
Pathogenesis
• As an antigen enters the body it is taken up by professional antigen presenting cells
which present them to T-Helper cells.
• These T-Helpers trigger the proliferation of B-cells and encourage the production of
antigen specific Antibodies (IgG or IgMs)
• Soluble antigens in the organism's blood stream then come into contact with these
specific antibodies
• The antibodies bind with the antigen forming an antigen-antibody complex in the
bloodstream
• These floating complexes then trigger the complement system, prompting a cascade of
inflammatory mediators to be released.
• These inflammatory mediators then trigger the degranulation of basophils and
neutrophils, which release lysosomal enzymes that damage the vascular wall
Diseases
• Vasculitis
• Arthus Phenomenon

VSM
• Serum Sickness

12. Immune disorders - cell-mediated hypersensitivity reaction type IV.


Type IV hypersensitivity, also known as delayed type, or T-cell mediated hypersensitivity is
the only one of the four types to be cell mediated. This means there are no antibodies involved,
unlike types I-IV.
Antigens
• Some metals (e.g. Nickel, Chromium)
• Cosmetics
• Detergents
• Organ Transplants
Pathogenesis
• As an antigen enters the body it is taken up by professional antigen presenting cells
which present them to T-Helper cells.
• T-Helper cells then mature into effector cells. These trigger the development of T-killer
cells and macrophage activation
• The macrophages then release inflammatory mediators, which recruit basophils and
neutrophils, and lead to the release of lysosomal enzymes which go on to damage tissues
Antigens generally enter the body through contact (e.g. jewelry, poison ivy) rather than
consumption in this form of hypersensitivity.
Diseases
• Contact dermatitis
• Tuberculin reactions
• Host vs Graft reactions
• Graft vs Host reactions

VSM
13. Mechanisms of autoimmune response. Autoimmune diseases.
Autoimmunity is the ability of the immune system to identify its own healthy cells. A disease in
this part of the immune system is known as an ‘autoimmune disease’ and leads to the cells of the
immune system destroying healthy cells.
In order to ensure the body is able to identify healthy tissue not to attack it, the body must
ensure its immune cells are ‘Tolerant’ to normal cells. Its mechanism for doing this is divided
into Central and Peripheral Tolerance.
The exact mechanism of forming central tolerance uncertain, however, our main theories are:
1. Clonal deletion
Central tolerance in clonal deletion theory suggests that as T and B cells are developed, they
come across self antigens and those that are reactive undergo apoptosis.
In T-cells the process is:
• During T-cell development in the Thymus they start without any T receptor. Then the
CD4 and CD8 receptors are added, and they are exposed to MHC I or II, undergoing
positive selection to ensure they recognise the complex, those that don’t die.
• Following this, the T-cells lose either CD4 or CD8 and then encounter self peptides
presented by MHC. Those that bind too strongly to the self peptides under clonal
deletion, a form of negative selection.
• This leaves only T-cells with a CD4 or CD8 receptor that bind correctly to MHC I
In B-cells the process of clonal deletion is:
• During development in the bone marrow, B-cells will encounter self-antigens. Those
that react will be removed
Peripheral tolerance works to destroy any T or B cells that may have escaped the central
tolerance process, identifying and destroying any T or B cells outside the thymus and bone
marrow respectively which may harm healthy cells, the mechanism is as follows:
• As T-cells are activated, not only do they do their normal job (produce cytokines etc) but
they also die through apoptosis
• All normal T-cells express the death receptor Fas. Upon activation, the ligand for Fas
(FasL) is also expressed, meaning that the T-cells death is automatically triggered.
2. Clonal Anergy
This theory states that immune cells which react to self antigens when presented on Antigen
Presenting cells but are not stimulated by the additional required receptors which one would
expect if indeed the antigen was ‘other’ are forced into apoptosis instead of triggering an
immune response.
Looking specifically at T-cells, as the TCR is stimulated, in a normal pathogenic cell a
costimulatory receptor, CD28, is also activated by the B7 receptor on the cell. Where this doesn’t
occur, and the T-cell only receives one signal (from the MHC) it will undergo apoptosis.
3. Clonal ignorance
This states that T-cells that are reactive to self not presented in the thymus, will still mature and
migrate to the periphery, but will never encounter the antigen that they are reactive to as the
body will hid them away in inaccessible tissues. Such cells will likely die out.
Auto-active B-cells that escape central tolerance may also never come into contact with their
specific antigen based on a similar model.
4. Anti-idiotype antibody
Basically states that there are antibodies produced against any antibodies that are autoreactive.
These prevent the auto-reactive antibody producing B-cells from interacting with the antigen.
5. Regulatory T-cells
It is also thought that regulatory T-cells may play a role in preventing other immune cells from
interacting with the self.

VSM
14. Immunodeficient states. Primary immunodeficiency diseases. Acquired
immunodeficiency syndrome (AIDS).
Immunodeficient states, are, simply put, circumstances when the immune system isn’t quite on
its game! We can divide immunodeficiency in a number of ways, either based on what is affected
(i.e. B-cell, T-cell, combined T & B cell, complement or phagocytic immune deficiency), or based
on if it is Primary (congenital) or Secondary (acquired). The questions are based on the latter
format, so here we’ll look at examples of immunodeficient states which are primary
(congenital, from birth).
B-cell deficiencies
B-cells are responsible for our humoral immunity, the produce immunoglobulin antibodies
which will attack antigens. Where these doesn’t occur as it should, we see a number of
conditions.
Clinical manifestations:
• Higher incidence of infections (especially respiratory infections such as pneumonia)
• Higher incidence of autoimmune diseases and malignancies
• Skin pyogenic infections (such as staphylococcus and streptococcus)
• Otitis (ear infections)
Mechanisms:
• Altered B-cell function alters synthesis of Igs
• Igs are most important for mediating extracellular antigens, not intracellular ones, this
is one method of identifying possible B-cell deficiencies
Primary B-cell deficiencies
• Transient Hypogammaglobulinemia of infancy
• IgG is the only immunoglobulin passed through the fetal placenta to the fetus.
After birth IgG levels gradually decrease, before they start to be built back up
again between the ages of 1-2
• In some children the development of B-cells is delayed and IgG levels remain low
• This is not severe, as the IgAs and IgMs will provide protection, and this is a
transient condition that will only last until the B-cells are fully matured
• X-linked hypogammaglobulinemia (Bruton's disease)
• X-linked recessive inheritance
• B-cell maturation is halted early due to mutations in a tyrosine kinase enzyme.
As a result Ig Light chains (remember Igs have Heavy and Light chains) are not
produced
• Disease becomes apparent after 6 months when IgG levels have decreased
• Leads to increase in bacterial infections
• Lab results show Immunoglobulins are absent, treatment comes in the form of
systemic application of gamma globulins
• Selective deficiency of IgA
• Affects 1:700, lack of IgA in body as a result of blocking in the terminal
differentiation of IgA-secretion B-cells.
• IgM and IgG levels are normal
• IgA is largely found in the mucous membranes
• Most sufferers are asymptomatic, however weakened mucosal defense can lead
to predisposition to recurrent sinopulmonary infections and diarrhea
• Hyper IgM Immunodeficiency
• High IgM levels, low IgG and IgA
• X-linked recessive
• Results in recurrent pyogenic infections and autoimmune phenomena
T-cell primary immunodeficiencies
• T-cell populations (mainly CD4, helper and CD8, killer) are responsible for:
• Defence against fungi, protozoa, viruses and intracellular infections
• Control of cellular proliferation

VSM
• Organisation of immune response
• Clinical manifestations of damage to these cells can result in
• Severe infections
• Children rarely survive childhood
• Defects can occur to T-cell receptor, the activation of the T-cell process or the
secretion of cytokines
Primary T-cell immunodeficiencies:
• DiGeorge Syndrome
• Defect in 22nd week of gestation (development of chest, neck and thymus)
• T-cells are absent in lymph nodes, spleen and peripheral blood
• Caused by deletion on 22nd chromosome
• Transplant of thymus tissue can be of help
Combined immunodeficiency (T and B cells)
These occur as a result of mutations to one or more of the genes that regulate lymphocyte
proliferation and, or, maturation. Including those genes that affect T-cell receptors, cytokine
production and MHCs. Whichever gene is affected the end result is altered communication
between the T and B cells.
In the two most common forms of Combined Immunodeficiency (cytokine receptor common
gamma chain mutation and Adenosine deaminase deficiency (ADA) deficiency) are a result of a
hypoplastic thymus, with lymph nodes being atrophic and a lack of germinal centers. Such
defects to important cytes of T-cell development mean there is a severe defect in T-cells
resulting in a lack of coordination of the B-cells.
Defects of the complement system
The complement system is a series of proteins that are responsible for inflammatory and
immunologic responses. The complement proteins in the blood stream undergo a cascade of
reactions which in turn trigger immune actions such as cellular attack, ROS release, increased
permeability as well as others.
The most common primary disorder is hereditary angioneurotic edema.
• Deficit of C1 esterase inhibitor, this enzyme is responsible for controlling and inhibiting
the protein cleavage cascade
• This defect leads to:
• Uncontrolled activation of complement system
• Liberation of kinis
• Vasodilation leading to edema. Can cause severe edema in the face and neck
causing asphyxia which can be deadly.
Defects of Phagocytes
Several congenital defects in phagocytes are known, these include:
• Defect in phagocyte oxidase (NADPH oxidase) enzyme
• Can cause chronic granulomatous disease
• Defects in integrins and selectin ligands
• Causes leukocyte adhesion deficiencies
Secondary B-cell immune deficiency
• This occurs as a result of chronic loss of immunoglobulins from the body
• This may be a product of a number of mechanisms
• Nephrotic syndrome - Immunoglobulins lost in urine (generally not the heavier
IgM)
• Enteropathies - Immunoglobulins lost from the intestinal tract
Secondary T-cell immune deficiencies
Secondary T-cell immune deficiencies are more common than their primary counterpart, as
they can be the product of two, not uncommon conditions:
• Viral infections (HIV, Cytomegalovirus)
• Malignancies (Hodgkin's and non-Hodgkin's lymphomas)

VSM
We shall consider the viral infections in more depth at a later point, for now, understand that
the malignancies damage the sites of T-cell maturation and therefore the T-cells are damaged,
leading to T-cell immune deficiency.
Acquired Immunodeficiency Syndrome (AIDS)
First, some definitions, a syndrome is a collection of symptoms of unknown etiology. Once the
etiology is known, it becomes a disease. Before we discovered HIV (Human immunodeficiency
virus), AIDS was a collection of symptoms of unknown etiology. Now, despite the fact we now
understand the etiology, the name stuck, and we still talk about AIDS.
So AIDS is a retroviral disease caused by HIV. It is characterised by depletion of CD4+ T-
lymphocytes (T-helpers) and profound immunosuppression leading to opportunistic infections,
malignancies, weight loss and degenerative CNS processes.
AIDS is commonly transitted via sexual intercouse, blood products, insemination from an
infected sperm donor, in utero transmission and any other method of genetic transmission
between people.
The virus works by entering a CD4 cell and replicating. It integrates the proviral genome into
the host, triggering viral gene expression, creating more copies of the virus to continue on to
infect more of the organisms T-cells.

VSM
This systematic destruction of T-cells gives us the Acquired Immunodeficiency Syndrome
(AIDS) we are familiar with. The stages of the infection can be summed up in three stages:
• Early (acute) stage - acute infection phase
• Middle (chronic) stage - largely asymptomatic, with persistent generalised
lymphadenopathy
• Final crisis stage - when T-cell levels drop too low, can cause:
• Constitutional diseases
• Neurological diseases
• Secondary infections
• And Secondary neoplasia
15. Inflammation – etiology, phases, and main clinical signs. Vascular changes in
inflammation.

Inflammation is the body's response to harmful stimuli, such stimuli include:


• Mechanical damage
• Physical damage (e.g. Radiation, UV, Temperature)
• Chemical damage
• Biological damage (e.g. Viruses, bacteria)
Inflammation can be classified acute or chronic. Acute being the initial response, achieved
through plasma and leukocyte movement, while prolonged inflammation is chronic, and tends
towards a predominance of mononuclear cells, where we observe simultaneous destruction and
healing of the tissue.

VSM
There are three main stages of inflammation which can vary in intensity and duration, these
stages are:
• Vasodilation and increased vessel permeability
• First, very brief vasoconstriction followed by prolonged vasodilation
• This is a result of an increase histamine, kinins, prostaglandins and other
cytokines
• Phagocyte migration and phagocytosis
• Increase in permeability of vessels for plasma proteins
• Margination and diapedesis of neutrophils and plasma proteins (emigration)
• Chemotaxis (due to cytokines)
• Pus formation (more on these three stages in cell response below)
• Tissue repair and regeneration
The clinical signs of inflammation are:
• Colour
• Heat
• Swelling
• Pain
• Impaired function
• Vascular changes
• And Phagocytosis
(note only the first four refer to the gross signs of inflammation, the others are physiological or
microscopic).
So, acute inflammation can be categorised into two main parts which happen in sequence:
• Vascular changes
• Cellular events
In these two questions we will deal with each one in turn.
Vascular changes can then be divided again, into two parts:
• Changes in the vascular flow and vascular diameter (aka caliber)
• Increased vascular permeability
So, firstly, how does the body increase the vascular flow and diameter. As injury occurs
pathogen associated molecular patterns or Damage associated molecular proteins
(PAMPs or DAMPs) are released. These PAMPs and DAMPs are detected by resident immune
cells (mainly macrophages, dendritic cells, histiocytes, Kupffer cells and mast cells) with
Pattern recognition receptors (PRRs). Uon detection these inflammatory mediator cells
(more on this in Question 28) release histamine, serotonin as well as eicosanoids. The release of
Histamine triggers vasodilation which increases the blood flow and therefore the redness and
heat.
To recap the process:
• Damage triggers release of PAMPs or DAMPs
• PAMPs or DAMPs are detected by immune cells with PRRs
• The immune cells then release chemicals such as Histamine and
serotonin
• Histamine binds to receptors on the smooth muscle, stimulating
vasodilation.
This increased vasodilation reduces the speed of blow flow through the arteries leading to a
state known as stasis.
So, this covers the question of increased vascular flow, now, what about the increased vascular
permeability? There are four main mechanisms of increased vascular permeability:
• Endothelial cell contraction
• This is the most common cause of increased permeability
• It is stimulated by the release of Histamine, Serotonin and other chemical
mediators released by the immune cells that have been activated by PAMPs and
DAMPs
• This is an instant reaction and it is short lived

VSM
• Direct damage to endothelium
• In cases such as burns or toxins we see damage to the endothelial layer
• This, fairly obviously, will increase the permeability of the vessels
• It is sustained for several hours until damage is repaired
• Endothelial injury due to leukocytes
• Neutrophils adhering to the endothelium (will be discussed below) may injure it
• This also is sustained for several hours until damage is repaired
• Last and Least is the process of Transcytosis
• This is the increased transport of fluids and proteins thanks to intracellular
channels
• Such movement is triggered by Vascular Endothelial Growth Factor which
promotes leakage

16. Cell response in inflammation. Mediators of inflammation.


Above, we considered the first aspect of the inflammatory response, we should now look at
what the cells are doing while all this vasodilation and increased permeability is going on, and
the answer is, they’re having a blast!
So, in effect, it’s worth asking, what is the point of increased blood flow and vasopermeability to
an area of damage, and the answer is to help us get the cells that can help defend the body to the
problem area. In this case, the cells that can help are Leukocytes (mainly neutrophils and
monocytes) and the area damaged is going to be the interstitial. These leukocytes can then
mount a valiant defence of our bodies.
The process of leukocyte movement out of the lumen and into the interstitial fluid is the one we
are really concerned about, it is a multistep process we will review here:
1. Margination
1. As the vessels dilate the blood slows down (stasis) and the leukocytes move to
the periphery of the vessel
2. Roling
1. This is the attachment and de-attachment of the leukocytes to the endothelial
wall
2. This process is mediated bo Selectin proteins, of which there are three types:
1. P-selectins (on plasma cells)
2. E-selectins (on Endothelium)
3. L-selectins (on Leukocytes)

VSM
3. In normal plasma/endothelium/leukocyte cells, these selectins are not
expressed, it is only when they are activated, by chemical mediators such as
interleukin-1 and Tumor Necrosis Factor
3. Adhesion
1. Following rolling, the Leukocytes start to express Integrins (a transmembrane
glycoprotein) which bind to the Integrin Ligand
1. There are two categories of Integrin Ligand, VCAM-1 and ICAM-1
2. Integrins are only expressed, again, when the Leukocyte has been activated by
chemical mediators
4. Diapedesis (transmigration)
1. A set of proteins known as PECAM-1 (Platelet endothelial cell adhesion molecule
1) are responsible for helping the leukocyte/platelets to migrate across the
endothelial border and into the interstitial fluid.
5. Chemotaxis
1. Following Diapedesis, the leukocyte will move through the basement membrane
2. Now it needs to migrate towards the site of injury, this is mediated by
chemoattractants
1. Chemoattractants are:
1. Exogenous - bacterial products
2. Endogenous - cytokines, C3a, C5a, Leukotriene B4

So, which leukocytes go through this process?


• Neutrophils
• Dominate in the first 24 hours, but disappear quickly after this
• They respond more rapidly to chemokines than others, and attach more firmly
to adhesion molecules
• Monocytes
• Take over from Neutrophils after about 24 hours as the the predominating
leukocyte
Mediators of inflammation are chemicals that help to control and orchestrate the process of
inflammation. We can broadly divide the mediators of inflammation into three groups, these
are:
• Cellular derived mediators
• Plasma derived mediators (produced by the liver)
• Necrosis derived mediators (sometimes not considered a true group)

VSM
Alternatively we can divide mediators based on their mechanism of action:
• Vasoconstriction
• Vasodilation
• Increased vascular permeability
• Chemotaxins and Leucocyte adhesion
It should be obvious that each one of these mechanisms of action accounts for a different
part of inflammation.
Cell derived mediators
Chemical Source Function

Histamine Mast cells and basophils Increase arteriole dilation and increased
venous permeability

Serotonin Platelets Increase arteriole dilation and increased


venous permeability

Lysosomal enzymes Neutrophils & Macrophages Enzymes that breakdown etiological


substance

Prostaglandins Leukocytes, platelets, Cause vasodilation, fever and pain


Endothelial cells

Leukotrienes Leukocytes Mediate leukocyte adhesion and activation

Platelet activating Leukocytes & Endothelial Different sources different functions:


factor (PAF) cells • Mast cells/basophils - increased
vascular permeability
• Neutrophils - leukocyte aggregation
• Monocyte - leukocyte adhesion
• Endothelium - leukocyte
priming/chemotaxis
• Platelets - platelet activation

Activated oxygen Leukocytes Attack etiological factor


species

Nitrous Oxide Macrophages Cause vasodilation

Cytokines Lymphocytes, Macrophages, Cytokine is just a general term for proteins


Endothelial cells important in signalling. Functions are vast

Tissue Necrotising Macrophages Induce fever, cytokine production,


Factor & Interleukin endothelial gene regulation, chemotaxis,
1 leukocyte adherence, activation of
fibroblasts as well as systemic effects such
as low appetite and increased heart rate.
Plasma derived mediators
Name Produced by Function

Bradykinin & Kinin system Induces vasodilation, increased vascular permeability and
Kallidin induces pain

Factor XII Liver Inactive protein, activated by collagen, platelets or


exposed basement membrane. Activates Kinin system,

VSM
fibrinolysis system and coagulation system when
activated.

Membrane Complement system Complex of C5b, C6, C7, C8 and C9 that inserts into
attack complex bacterial cell walls and causes lysis.

Plasmin Fibrinolysis system Breaks down fibrin clots, cleaves C3 and activates Factor
XII

Thrombin Coagulation system Cleaves fibrinogen to produce fibrin which can help clot.
So, lets take a quick look at some of those systems that we’ve just mentioned.
Kinin system
• Mediated by bradykinin and kallidin, two vasodilators
• Plays a role in inflammation, blood pressure, coagulation and pain control
• In effect, Neutrophils release Kallikrein, this in turn converts Kininogens into Kinins.
• Kinins (such as bradykinin) then go on to act to:
• Stimulate complement system
• Promote vasodilation and increase capillary permeability
• Activate pain receptors
• Act as chemotaxins
Complement system
This is the name we give to the biochemical cascade of the immune system that allows
the body to activate and immune response and organise its tools.
It consists of around 30 complement proteins in the blood, synthesized by the liver, they
augment the function of the immune system by opsonization, membrane attack complex
formation and enhancing inflammation.
Opsonization
• The coating of a pathogen with compliment proteins to make
phagocytosis by macrophages easier thanks to receptor binding
Membrane attack complex
• Create a membrane attack complex where a group of proteins make a holes
in the membrane of the pathogen
The proteins travel around the body in an inactive form, and only become activated when
they come into contact with a pathogen or are activated by other complement
proteins. The way in which a complement protein becomes active can be via a Classical,
Alternative or Lectin pathway.
All of these pathways help to split (cleave) the Complement 3 (C3) protein into C3a and
C3b. C3a assists within inflammation while C3b assists in Opsonization and the
formation of membrane attack complexes.
Classical pathway
• Initiated when Antibodies bind to Antigens. A C1 complex then binds to the
antibodies, forming the C4b2a complex.
• C4b2a complex then sits on surface of pathogen and splits C3 into
C3a and C3b through the alternative pathway (below)
Alternative pathway
• Picks up once C4b2a has been created by the Classical (or lectin) pathway.
• Forms C3bBb (C3 convertase) complex from C4b2a on surface of
pathogen
Fibrinolysis system
Fibrinolysis is a process that prevents blood clots from growing and becoming problematic,
i.e. it breaks down a fibrin clot. There are two types, aptly named primary and secondary
fibrinolysis. The primary type is a normal physiological process, the second is due to medical
or pathophysiological reasons.

VSM
The fibrinolytic system is mainly mediated by the enzyme plasmin which cuts the fibrin
mesh at various places, creating fragments which can be cleared by the body.
Plasminogen is first formed in the liver, and forms part of a clot when formed. Tissue
plasminogen activator (t-PA) and urokinase are slowly released by endothelium which in
turn activate the Plasminogen (which is embedded within the clot) into Plasmin, an

enzyme.
The Plasmin in turn breaks down the Fibrin mesh which held the clot together. Once it
has completed its task, the Plasmin is inhibited by alpha1-antiplasmin and alpha2-
macroglobulin, along with thrombin-activatable fibrinolysis inhibitor, which works to
make the Fibrin more resistant to the Plasmin.

17. Metabolic changes and proliferative processes during inflammation.


The metabolism of leukocytes is fundamentally different to that of lymphocytes, as they are
designed to operate at very low oxygen levels and are primarily glycolytic cells. The energy
expended by these cells is significant, not only is energy required when destroying the
etiological agent, but energy is similarly required for the chemotaxis and movement of these
leukocytes once outside of the blood supply.
As such, these leukocytes are equipped with a unique mitochondrial phenotype which gives it
the capacity to generate energy at low oxygen concentrations within deep lesions.
Proliferative process during inflammation
This is the process of cell proliferation during tissue repair. About two days after the wound
occurs, fibroblasts enter the wound and mark the onset of proliferative phase, even before
inflammation has ended. Fibroblasts along with vascular endothelial cells, stimulated by
growth factors, lay down additional tissues. In essence we can see the proliferative process in
the following stages:
• Angiogenesis
• Formation of new blood vessels in the region by endothelial cells
• Fibroplasia and granulation tissue formation
• Fibroblasts proliferate and migrate towards the wound, forming granulation
tissue (a rudimentary tissue that appears 2-5 days post wound)
• Controlled by growth factors PDGF (platelet derived growth factor) and TGF-
beta (Transforming growth factor beta)
• These fibroblasts produce and deposit collagen (Type III)
• Epithelialization
• This occurs after granular tissue has been laid down
• A product of the action of Keratinocytes
• Occurs above granular tissue, but below a scap, if one was formed

VSM
• Contraction
• Wound contracts in upon itself thanks to the actin of myofibroblasts contracting
These four steps sum up the process of tissue proliferation.

18. Types of inflammation. Development, outcome and biological significance of


inflammation. Chronic inflammation.
Inflammation can be characterised as:
• Acute
• Or Chronic
Additionally, we can classify inflammation based on the inflammatory fluid rather than the
duration of inflammation:
• Serous inflammation - clear fluid exudate, now protein mass
• Fibrinous - exudate with fibrin and fibrinogen
• Purulent - exudate has pus (contains lots of dead leukocytes - usually strep infections)
• Hemorrhagic - exudate contains blood
(note, the term exodate refers to one of two types oedema fluid, one is Exudate, inflammatory
fluid, the other is Transudate, non-inflammatory fluid, usually the product of increased oncotic
pressure.)
Development takes the form of resident immune cells (macrophages, dentritic cells, histiocytes
and mast cells) which initiate inflammation via their pattern recognition receptors (PRRs)
which recognise the pathogen associated molecular patterns (PAMPs) and Damage-
associated molecular patterns (DAMPs). These then trigger the vascular and cellular changes
that we should be familiar with from the above questions.
Inflammation is the body's defensive mechanism to help protect from infection and injury. It
localises and eliminates the injurious agent, and works to remove and heal damaged tissue.
By increasing blood flow to the region, ensuring macrophages move into the interstitial where
the injurious agent is found, and simultaneously activating the body's immune response at a
systemic level, inflammation can be seen as an efficient and effective mechanism for healing.

VSM
Inflammation can be characterised as:
• Acute
• Or Chronic
• Definition
• Inflammation with duration more than 2 weeks in which active inflammation,
tissue alteration and attempts for repair are simultaneous
• Etiology
• Persisting infection
• Prolonged exposure to toxic stimuli
• Altered immune tolerance (autoimmune disorder)
• Characterisation
• Infiltration with mononuclear cells
• Macrophages, lymphocytes, plasma cells
• Activated by epithelioid cells or other active macrophages
• As they move into the tissue, differentiate to specific macrophage
• Tissue destruction
• Caused by inflammatory cells
• Proliferative process
• Injured tissue is replaced by CT - angiogenesis and fibrosis
Acute vs Chronic inflammation comparison:
Chronic Acute

Etiological agent Non-degradable pathogen, viral infection, Bacteria, injured tissues


persistent foreign body, autoimmune
reaction

VSM
Major cells Mononuclear and fibroblasts Leukocytes (mainly neutrophils)

Primary IFN-gamma, growth factors, ROSs and Vasoactive amines and


mediators hydrolytic enzymes eicosanoids

Onset Delayed Immediate

Duration Indefinite A few days

Outcomes Tissue destruction, fibrosis, necrosis Resolution, abscess formation or


chronic inflammation

19. Fever – etiology and pathogenesis. Hyperthermia.


A Fever is defined as an elevated body temperature above the circadian variations due to a
higher set point of the thermoregulatory centre the the Hypothalamus caused by pyrogens. It is
important to note, that it is not simply an increase in body temperature (Hyperthermia) which
occurs without the resetting of the thermoregulatory centre.
Normal circadian variations occur between 35.8 and 37.4 degrees C, with the lowest point
between 4-6am and the highest between 4-6pm. When measuring temperature it is important
to note that measurements from the rectum can be up to 0.6 degrees higher than the axillary
temperature.
Additional physiological factors that affect body temperature include:
• Ovulation (2 weeks before ovulation, temperature is lower, temperature increases
during ovulation)
• Seasonal variations (associated with metabolic changes)
• Postprandial (after meal) elevation
• Pregnancy
Etiology of Fever
Fevers are caused by Pyrogens. We separate Pyrogens into endogenous and exogenous.
• Exogenous
• Microbial
• Bacteria (Gram +ve and Gram -ve)
• Viruses
• Parasites
• Non-microbial
• Vasculitis, thrombosis, infarction and trauma
• Immune reactions
• Neoplasm
• Acute metabolic disorders (e.g. Gout)
• Endogenous (all cytokines released into general circulation where they migrate to the
brain and activate arachidonic acid pathway)
• Interleukin 1 (alpha and beta)
• Interleukin 6
• Interleukin 8
• Tumor necrosis factor (TNF) alpha and beta
These are released from Leukocytes, Monocytes, Neutrophils, NK cells, Fibroblasts and others.
Pathogenesis
In essence, the exogenous factors will trigger the release of the Endogenous factors. The
Endogenous factors then travel through the bloodstream, to the Anterior hypothalamus and
activate the arachidonic acid pathway, stimulating production of Prostaglandin E2 (PgE2). PgE2
then acts to reset the thermoregulatory centre, triggering a fever.
• Infectious agent

VSM
• Monocyte/macrophage/other cell
• Pyrogenous cytokine (IL-1, IL-6, IL-8, TNF, etc..)
• Anterior Hypothalamus
• PgE2 production stimulated
• Increased set point of TRC
• Heat dissipation/heat production
• Fever

It is also worth considering at this point that Antipyretics work by preventing the formation of
Prostaglandin E2. A chemical synthesized from Arachidonic acid, which converts to
Prostaglandin H2, which is intern converted to PGE2. Mediated by PGE synthase enzyme.

20. Stages of fever. Types of temperature curves.


The stages of fever are:
• Prodrome
• 15-90 minutes.
• Release of endogenous pyrogen and PGE2 is synthesised, resetting the
thermoregulatory centre in the hypothalamus.
• Increase
• The TRC has been reset, body temperature increases but the person has the
feeling of being cold.
• Thermogenesis occurs in this part of the process, leading to increase in
cardiovascular and respiratory systems, along with increase in basal metabolism
• Stablisation
• The body temperature culminates, the body has reached the temperature of the
reset TRC

VSM
• Vasodilation of skin vessels and decrease in peripheral vascular resistance are
observed, helping to try and reduce body temperature
• Patient has warm red skin, sweats and loses heat by conduction, radiation and
evaporation.
• Decrease
• This phase sees a decrease in the body temperature
• Decrease may be critical (decreases in 1-2 hours) or lytical (1-2 days)
• Critical decrease can be very dangerous to patients

VSM
A = Fever continuous
B = Fever continues to abrupt onset + remission
C = Fever remittent

VSM
D = Intermittent fever
E = Undulating fever
F = Relapsing fever
Type Character Etiology

Continuous Does not remit, raise temp Typhoid, Typhus, drug fever

Remittent Daily fluctuation less than 2 degrees C, but never None


normal

Intermittent Temperature falls to normal every day Pyogenic infection,


lymphoma, military TB

Undulating Slow increase in body temp during some time, Brucellosis


followed by slow decrease

Replasping Temperature returns to normal for days before Malaria (3 day pattern),
rising again Hodgkin's Lymphoma

21. Metabolic and functional changes during fever. Biological significance of fever.
Metabolic and functional changes during fever
This seems an appropriate time to consider the general clinical manifestations of a fever, so
named:
• General
• Loss of appetite
• Fatigue
• Insomnia
• Leucocyte increase, along with increase in liver acute phase proteins
• Activation of latent herpes (where present)
• Systemic
• Cardiovascular - tachycardia, hypotension
• Respiratory - hyperventilation
• Gastrointestinal - nausea, vomiting, decreased intestinal motility
• CNS - Seizures, generalised aching
• Urinary system - oliguria or polyuria
• Endocrine - increased Thyroid, pituitary and adrenal activity.
When considering the metabolic changes, these can be viewed as so:
• Changes to carbohydrate metabolism
• Increased glycogenolysis
• Changes to Fat metabolism
• When no more glycogen to breakdown, we see hyperlipidemia, activation of
beta-oxidation and production of ketone bodies (see below)
• Changes in protein metabolism
• Increased protein breakdown (partly due to TNF production)
These metabolic changes lead to metabolic acidosis (ketoacidosis) and water and electrolyte
disbalances.
Biological importance of fever
• Favorable (adaptive)
• Decreased reproduction of microbes
• Increased phagocytosis
• Increased bactericidal activity of white blood cells
• Activation of specific immune response (IL-1)

VSM
• Unfavourable
• Increased oxygen consumption (13% for each degree)
• Decreases body mass (increased catabolic effects)
• General and systemic disorders, metabolism , decreased adaption
22. Red blood cell disorders. Classification of anemias.
Red blood cell disorders generally lead to anemias, or less commonly polycythemia (an
increased in RBC numbers).
Anemia is the reduction in oxygen-transporting capacity of blood. This is either due to a
reduction in RBC numbers or reduced hemoglobin.
Symptoms and signs
• Anemia leads to
• Inadequate O2 transport to tissue, which leads to
• Tissue hypoxia, which results in:
• Brain - drowsiness, fatigability
• Heart - decreased work, tachycardia, pump failure
• Lung - dyspnea, decreased respiratory reserve
• Muscle - weakness, fatigue
• Along with general paleness and clammyness
We can classify anemias in a number of different ways, either based on the morphology:
• Size (Normocytic, Microcytic or Macrocytic)
• Shape (Spherocytic, Sickle cell)
• Colour (Normochromic, Hypochromic or Hyperchromic)
Or, alternatively, and probably more usefully, by the underlying pathophysiological mechanism:
• Hemorrhagic anemia
• Acute
• Caused by sudden blood loss, such as in trauma
• Tissue fluid also starts to enter the blood vessels to try and increased
blood pressure
• Several hours after the event may see decreased hematocrit
• Reticulocytes (immature RBCs) are released in the coming days to
increase RBC numbers
• Chronic
• Caused by chronic blood loss from locations such as GI and Urogenital
tracts
• Body is unable to produce hemoglobin at the required level due to not
enough iron.
• Deficiency anemias (Impaired erythropoiesis)
• Vitamin B12 deficiency (see 36)
• Iron deficiency (see 37)
• Hemolytic anemias (excessive erythrocyte destruction)
• Can be Intravascular or extravascular (e.g. in spleen)
• Can be intrinsic to the RBC (e.g. genetic defect) or extrinsic (e.g. viral)
• Aplastic anemias (defects in bone marrow)
Hemolytic anemias are not simple so we should look at in some more detail here. They result
simply from increased destruction of the blood cells. This can be intravascularly (abnormal) or
it can be overactive extravascular destruction.
• Etiology
• Membrane defects
• Often lead to spherocytic anemia
• Hemoglobinopathies
• Enzyme defects
• Symptoms
• Often have jaundice and hepatosplenomegaly

VSM
• When intravascular hemolysis may see hemoglobinuria, as bits of destroyed RBCs are
filtered in glomeruli, this may lead to secondary renal failure.
Sickle cell anemia is one such intrinsic erythrocytic membrane deficiency. Cells are formed in
their ordinary shape, however once oxygenated and deoxygenated a few times become sickled
irreversibly. As well as increased cellular destruction, this also leads to clogging of the vessels,
this may cause:
• Retinopathy
• Acute respiratory distress
• Haematuria and polyuria
• Autosplenectomy and infections
• And possibly leg ulcers
The example for enzyme defects is that of Glucose-6-Phosphate Dehydrogenase deficiency
(G6PD). This enzymopathy results in higher levels of free radical production within
erythrocytes, leading to intravascular and extravascular hemolysis. On the slide RBCs look like
they have a ‘bite’ taken out of them.
Aplastic anemias are due to a defect in the hematopoiesis of Red Blood cells, however, as the
condition occurs at the level of the pluripotent blood cell progenitor, it causes a reduction in the
levels of all blood cells, including leukocytes.
• Etiology
• Ionising radiation
• Viral infections
• Toxic factors
• Metastasis
• Key feature is the reduction in immune capability of the patient.

23. Iron deficiency anemia.


Iron deficiency anemia is caused by insufficient iron in the body (WHAT! A! SHOCK!).
There are four main etiological factors:
• Chronic blood loss
• Due to heavy periods, GI bleeding or bleeding in the urogenital tracts
• Lack of iron in diet
• Iron comes from meat, eggs, greens
• Inability to absorb iron
• Intestinal disorders such as celiac can cause this, similarly resection of the small
bowel can affect iron absorption
• Pregnancy
• Babies use lots of iron!!
Symptoms
• Fatigue
• Weakness
• Paleness
• Tachycardia
• Headache
• Cold extremities
• Polished tongue
Slide morphology
• RBCs look lighter due to reduced hemoglobin (Hypochromic)
• RBCs are smaller (Microcytic)

VSM
24. Pernicious anemia.
Also known as Vitamin B12 deficiency anemia. (In fact, that’s not really true, but the lecture said
they were, so let's run with it!). B12 is required for RBC production, and a deficit will result in
reduced RBC numbers. Specifically, B12 is involved in DNA and Myelin synthesis.
Etiology
• Generally a result of abnormal absorption in GI tract
• Gastro Protein found in the distal ileum is used to help absorb B12, where this
intrinsic factor is missing, B12 is not absorbed.
• Similarly, in cases where the terminal ileum has been removed we can see a
deficit of B12
• More uncommonly, it can be as a result of diet (e.g. in poorly managed vegan diets)
Symptoms
• General symptoms of anemia
• Tired and weak
• Tingling and numbness in hands and feet
• Polished tongue
Pernicious anemia is a form of megaloblastic anemia, this is because lack of B12 means that
the RBCs stay in the bone marrow longer than usual as the are completed slower. As such the
bone marrow packs the RBCs with as much haemoglobin as they can fit. This means that the
RBCs are both hyperchromic and macrocytic when viewed under a slide.

25. White blood cell disorders. Leucocytosis & leucopenia.


A reminder, the leukocytes are:
• Granulocytes
• Neutrophils
• Basophils
• Eosinophils
• Lymphocytes (T and B cells)
• Monocytes (Macrophages)
White blood cell disorders are generally divided up into those that cause excessive numbers of
leukocytes (leukocytosis) and those that lead to reduced levels of leukocytes (leukopenia)
Leucocytosis is a leukocyte count above 4.5-11 x109/L. It is generally a physiological response
to infection, but can also be a result of parasitic infections or bone marrow tumours, as well as
leukemia. We look at the types of leucocytosis and what they represent below:
• Neutrophilic leukocytosis (most common form)
• Caused by acute bacterial infections, sterile inflammation and tissue necrosis
• Eosinophilic leukocytosis
• Caused by Allergic disorder, parasitic infections, lymphomas, autoimmune
diseases and vasculitis
• Basophilic leukocytosis
• Rare, can be caused by myeloproliferative diseases
• Monocytosis
• Chronic infections (e.g. TB), systemic autoimmune diseases (e.g. lupus),
inflammatory bowel diseases
• Lymphocytosis
• Chronic infections (e.g. TB), viral infections, some malignancies (e.g. lymphocytic
leukaemias)
All these increases are often seen with a ‘left upper shift’ regarding the ratio of immature to
mature neutrophils and macrophages, that is to say, we see more young leukocytes.
Leukopenia is a surfect of white blood cells, (below 4.5 x 109/L). There are two types:
• Lymphopenia (rarer)
• A decreased lymphocyte count, found in immunodeficiency states

VSM
• Neutropenia
• A decreased neutrophil count (agranulocytosis) causes can be one of two
mechanisms:
• Reduced/ineffective production due to:
• Suppression of myeloid stem cells, aplastic anemia
• Suppression of granulocyte precursors by drugs
• Ineffective granulopoiesis
• Inherited conditions
• Increased destruction of neutrophils
• Immune mediated (idiopathic
• Increased destruction in spleen
• Increased peripheral consumption via severe infection
Leukopenia results in recurrent infections in the organism.
Cyclic neutropenia is a rare blood disorder characterised by recurrent low neutrophil levels.
Symptoms include fever, mouth ulcers and recurrent infections. The disease can be inherited
(AD) or acquired. Mutations affect the ELANE gene which makes a protein called neutrophil
elastase. In effect cyclic neutropenia is an issue with neutrophil production and an example of
an inherited condition.

26. Leukemia. Lymphoma.


Both Lymphomas and Myelogenous leukemia are cancers of blood cells. Lymphomas affect the
lymph nodes, simply put, they are cancers of the lymph nodes. Meanwhile, leukemias are
different, as they affect the bone marrow directly. Firstly, we shall address Lymphomas.
There are two main classes of lymphoma:
• Hodgkin's lymphomas
• Distinctive neoplastic Reed-Sternberg giant cells
• Types , distinguished on cellular morphology and cell infiltration in lymph node
biopsy:
• Classical
• Nodular sclerosing
• Mixed-cellularity subtype
• Lymphocyte rich
• Lymphocyte depleted
• Nodular lymphocyte-predominant
• Symptoms include swelling of lymph nodes, fatigue, fever, sweats and weight
loss
• Spread in in a stepwise fashion along lymph chains
• Non-Hodgkin's lymphoma
• Can start in almost any part of the body (unlike Hodgkins which starts in a single
node)
• Generally onset is older (over 65)
• Many subtypes but generally
• Aggressive
• Indolent (slow development)
Hodgkin Lymphoma Non-Hodgkin Lymphoma

Localised to single group of nodes Frequent involvement of multiple nodes

Orderly stepwise spread Non orderly spread

Mesenteric nodes and Waldeyer's ring rarely Mesenteric nodes and waldeyer's ring
involved commonly involved

VSM
Extranodal involvement uncommon Extranodal involvement common
All types of lymphoma have four clinical stages:
• Stage I - involvement of a single lymph node region
• Stage II - involvement of two or more lymph node regions
• Stage III - involvement of lymph node regions on both sides of diaphragm
• Stage IV - disseminated involvement of one or more extralymphatic organs
There are four types of leukemia:
• Acute lymphoblastic leukemia
• Chronic lymphoblastic leukemia
• Acute myeloid leukemia
• Chronic myeloid leukemia
In effect the lymphoblastic leukemias effect the lymphoid stem cell (going on to effect
lymphocytes, that is T and B cells), while the myeloblastic leukemias effect the myeloid stem cell
(going on to effect RBCs, platelets, granulocytes and monocytes).
Acute lymphoblastic leukemia
• Overproduction of immature white blood cells (lymphoblasts). This overproduction
occurs in the bone marrow, and sees a reduction in lymphoblast quality
• Most common occurrence in childhood 2-5 years of age
• Symptoms include increased risk of bacterial infection, dyspnea, chest pain, tendency to
bleeding and general anemic symptoms
• Thought to be a cancerous gene affecting the lymphoblast
Chronic lymphoblastic leukemia
• Most common type of leukemia affecting adults
• B-cells grow in uncontrolled manner and accumulate in bone marrow where they crowd
out healthy blood cells
• Results in swollen lymph nodes, spleen and liver, eventually with anemia and infections
Acute Myeloid leukemia
• Cancer of myeloid line with rapid growth of cells that build up in bone marrow and
effect other normal cell production
• Rare, treated with bone marrow replacement therapy
• Symptoms include infections, anemia, and frequent bleeding
Chronic Myeloid leukemia
• Gradual proliferation of myeloid cells in bone marrow
• Linked with genetic abnormality known as Philadelphia chromosome.
• Has a Chronic and Accelerated phase

27. Platelet disorders. Drug-associated immune thrombocytopenia.


Platelets help to form blood clots, produces from myeloid blast cells, too few
(thrombocytopenia) and we have excessive bleeding, too many and we have excessive

VSM
clotting. So, here are the three main categories and some examples:

• Thrombocytopenia
• Altered platelet function
• Congenital
• Disorders of adhesion
• Disorders of activation
• Disorders of aggregation
• Acquired
• Disorders of adhesion
• Thrombocytosis
• Reactive
• Myeloproliferative neoplasms
• Other myeloid neoplasms
• Congenital
Thrombocytopenia
• Normal platelet count is 150k - 450k, lower is Thrombocytopenia
• Excessive bleeding, bruising, purpura and petechiae are all common symptoms of the
disease
• Etiology can be vast
• Decreased platelet production (aplastic anemia, B12 deficiency)
• Short platelet life span (immune and nonimmune mechanisms)
• Increased platelet destruction in spleen
• Dilution after massive hemotransfusion (platelets don’t transfer in blood
transfusion)
• Inherited idiopathic thormocytopenia
• Where count is less than 20 k we see spontaneous bleeding
Altered platelet function
• Often congenital, includes diseases where platelets fail to adhere, activate, or aggregate.
• Severe example would be the genetic disorder Glanzmann’s thrombasthenia where
platelets contain defective/low glycoprotein IIb/IIIa
• This is a fibrinogen receptor, meaning no fibrinogen bridging of platelets can
occur.
Thrombocytosis
• High platelet counts, often asymptomatic, however can lead to predisposition of
thrombosis.

VSM
• Above 450k = thrombocytosis
• Can be:
• Essential (primary) - a result of myeloproliferative diseases
• Reactive (secondary) - result of inflammation, splenectomy, iron deficiency
anemia, hemorrhage or others
• No real treatment other than low dose aspirin in extreme cases to avoid thrombosis
Drug-associated immune thrombocytopenia
Some non-steroidal anti-inflammatory drugs (NSAIDS) suppress normal platelet function.
• Aspirin irreversibly inhibits cyclooxygenase-1 (COX1) which helps with platelet
production
• Ibuprofen inhibits platelet function for 24 hours

28. Coagulation disorders.


Coagulopathies see patients tend towards excessive bleeding, this may be spontaneous or
following injury.
Coagulopathy may be due to reduced levels of coagulation factors (proteins) in the blood.
Common genetic disorders such as Hemophilia and Von Willebrand’s disease can cause such
diseases. Additionally, coagulation disorders can be acquired, thanks to a deficit of Vitamin K.

VSM
Von Willebrand disease

• AD, most common hereditary blood clotting disease


• Deficiency in quality or quantity of von Willebrand factor, a protein required for platelet
adhesion
Hemophilia
• Haemophilia A - lack of factor VIII
• Haemophilia B - lack of factor XI
• Both X linked
Disseminated intravascular coagulation
• In cases where you have diffuse clotting, clotting factors are used up, and this leads to
spontaneous bleeding due to a lack of clotting factors.
29. Static properties (compliance & elastic recoil) and dynamic properties (flow &
resistance) of lungs – pathological changes. Changes in ventilation/perfusion ratio.

VSM
Ventilation of lungs is a result of repeated changes in the pressure gradient between the
atmosphere and the alveolar space. These changes are due to the changes in the chest and
lung volume during inspiration and expiration.
During inspiration the external Intercostal muscles and Diaphragm muscles will all
help expand the thoracic cavity, this in turn reduces the pressure in the thoracic cavity,
which causes reduction in intrapleural pressure, which causes reduction in alveolar
pressure (as the alveolar volume increases). This brings the alveolar pressure below
atmospheric pressure so air rushes in.

During expiration the external intercostals relax while internal ones contract, along with
the relaxation of the Diaphragm. Together these decrease the size of the thoracic cavity,
reducing volume and therefore increasing pressure of the thoracic cavity, intrapleural cavity

VSM
and alveoli. The alveoli then have a pressure greater than the atmospheric pressure,

meaning air is pushed out.


When Alveolar pressure = Atmospheric pressure, there is no air movement.
The pressure gradient between the alveoli and the pleural cavity is known as the
transpulmonary pressure (PL), this is always a positive value in order to maintain the
inflation of the lungs.
Static properties
• Elastic recoil is the lung's ability to rebound following stretching of inhalation. It does so
as a result of the relaxation of the diaphragm, which increases the pressure in the
pleural cavity and forces the lung to contract. This phenomenon occurs because of the
elastin in the elastic fibers, and the surface tension of surfactant.
• Lung compliance is a measure of the lungs ability to stretch and expand. It can be
divided into static (change in volume at a specific pressure) and dynamic (change in
volume during actual movement of air) compliance.
Elastic recoil and Lung compliance are in effect opposites of one another. High elastic recoil can
be seen in cases of fibrosis, while compliance can be thought of as a very pliable lung as is often
the case in emphysema.

VSM
Dynamic properties
• Airflow will impact the overall non-elastic resistance of the lungs. Laminar flow results
in less resistance while turbulent flow results in increased resistance. Airway resistance
flows as the volume of air in the lungs increases.
• Resistance to breathing can be divided into elastic and nonelastic. Non elastic is
caused by the resistance of the airways to movements, frictional forces and the
inertia of air and tissues. The lungs and muscles of the thoracic have elastic properties,
giving them elastic resistance during inspiration. Elastic resistance accounts for about
65% of total resistance to breathing.
Restriction of the airways, or interuption of the laminar flow will increase the overall work that
the patient will have to do in order to successfully ventilate their lungs.
What should be clear here, is that changes to static properties result in restrictive
respiratory diseases, while changes to dynamic properties result in obstructive
respiratory diseases.
The ventilation/perfusion (V/Q) ratio is the air that reaches the alveoli/the blood that reaches
the alveoli via capillaries.
In a perfect scenario, this ratio would be equal (i.e. V/Q = 1).
A V/Q mismatch indicates that there is increased dead space (part of the lung not undertaking
gas exchange), or that there are more capillaries than alveoli. Let's look at some reasons for this
pathology:
• Normal V/Q = 0.8 (4L oxygen/5L of blood)
• Decreased V/Q - reduction in lung ventilation or increased perfusion (COPD)
• Chronic bronchitis - bronchospasm, mucus plugs, inflammation and airway
obstruction increased airway obstruction, worsen ventilation and decrease V/Q
• Asthma - restricted bronchioles therefore less ventilation
• Pulmonary edema - increased fluid therefore less space for gas exchange
• Airway obstruction (foreign body)
• Increased V/A - increased ventilation or decreased perfusion
• Pulmonary embolism - blood clot in the lungs, blocking off capillaries in the
lungs, reducing the blood that gets to the alveoli
• Emphysema - damaged and destroyed alveoli, therefore available surface area
goes down, ventilation therefore goes up, increasing V/Q.
30. Disturbances in the control of breathing.
In normal physiological conditions control of breathing is the product of PCO2 measurement in
peripheral chemoreceptors, and changes of pH in blood as measured by central
chemoreceptors. These feedback to the Dorsal respiratory group (responsible for inhalation)
and the ventral respiratory group (responsible for exhalation), both located in the medulla.
These neurons go on to control the respiratory muscles of the thorax (intercostals and
diaphragm).
Abnormal or uneven respiratory rhythm is known as periodic breathing. There are three main
types, Cheyne-Stokes breathing, Biot’s breathing and Kussmauls breathing.

VSM
Cheyne-Stokes Breathing
• Gradually developing rapid deep respiration (hyperpnea), depth of breathing then
decreases before complete stoppage of respiration (apnea).
• Occurs in physiological conditions (deep sleep, high altitude and newborns) as well as
during cardiac and renal diseases as well as prem babies
Biot’s breathing
• Characterised by period of apnea and hyperpnea, unlike Cheyne-Stokes there is no
gradation.
• Occurs as a results of pathology to the nervous system, such as stroke.
Kussmaul's breathing
• Deep, labored breathing pattern, although no increase in speed
• Generally a result of metabolic acidosis (particularly diabetic ketoacidosis)

VSM
31. Obstruction of airflow in respiratory passageways. Pathophysiology of asthma.
Asthma is a chronic inflammatory disorder of the airways. It is characterised by:
• Intermittent and reversible airway obstruction
• Chronic bronchial inflammation with eosinophils
• Bronchial smooth muscle cell hypertrophy and hyperreactivity
• Increased mucous secretion
There are two types of asthma:
• Atopic asthma
• Most common, example of IgE (Type I) hypersensitivity
• Family history common, asthma attacks triggered by environmental antigens,
often preceded by allergic rhinitis, urticaria or eczema
• Nona-atopic asthma
• No evidence of allergen sensitization, family history of asthma less common
• Respiratory infections due to viruses are common triggers
Pathophysiology
• On first exposure airways react to antigen (various) and dendritic cells activate T-
Helper (T2) cells.
• T-Helpers activate IgE B Cells and the IgE antibodies activate mast cells which release
granules and mediators
• On second exposure antigens instantly bind to IgE antibodies on mast cells in air ways.
These mast cells release mediators, which then help to trigger neuronal reflex and
induce bronchospasm.
• As well as bronchospasm increased vascular permeability, mucus production and
additional immunocytes recruitment is triggered.
• Among the immunocytes recruited are granulocytes, which will degranulate and cause
damage to the epithelium.

VSM
32. Obstruction of airflow in respiratory passageways. Pathophysiology of chronic
obstructive pulmonary disease (chronic bronchitis and emphysema).
Right, a reminder, Chronic bronchitis and emphysema are older terms for different types of
COPD. So they are subsections of the same thing! 9 times out of 10 Chronic Bronchitis and
Emphysema exist together, as they both share the same major risk factors.
Chronic bronchitis
• Inflammation of the bronchioles
• Diagnosed on the basis of a productive cough for 3+ months
• Risk factors
• Smoking
• Air pollutants
• Dust and Silica
• Genetics

VSM
• The closure of the airways restricts exhalation, reducing FVC (forced vital capacity),
especially FEV1 (as with all other obstructive diseases).
• The FEV1/FVC ratio for sufferers of COPD is typically much lower
• Pathophysiology
• Inhalation of irritants leads to hypertrophy and hyperplasia of bronchial mucous
glands and goblet cells (in bronchioles)
• This results in increased mucosal production and therefore a narrowing of the
airway
• Additionally, smoking damages cilia, meaning normal cilia action of mucosal
movement is impaired
• On physical examination we can hear wheezing and crackling, while blood gasses may
show hypoxemia and hypercapnia (high CO2), possibly heading to cyanosis.
• In areas with decreased gas exchange the body automatically constricts the capillaries
around that portion of the lung, as they body tries to shunt blood to areas with better
exchange. The problem is, when this is generalised throughout the lung, this increases
pulmonary vascular resistance, resulting in pulmonary hypertension, increasing work
for the right side of the heart and eventually resulting in right sided heart failure.
Emphysema
• Damaged or destroyed alveoli, resulting in permanently enlarged alveoli with limited
recoil
• Is defined by structural changes in the lungs
• The lack of recoil in the alveoli limits the amount of air exhaled, in the same way with
chronic bronchitis, we have a low FVC and very low FEV1, resulting in a much lower
FEV1/FVC ratio than normal. Meanwhile, TLC is often increased due to air trapping.
• Pathophysiology
• Irritants in the alveoli will trigger an inflammatory reaction, with cytokines
being released by immunocytes, importantly, proteases such as elastases and
collagenases are also released which cause most of the damage.
• This damage to the alveolar tubes, means they collapse in on themselves, leaving
air trapped in the alveolar sacs. This loss of elastin also increases lung
compliance, meaning more air can be trapped.
• As the alveolar are damaged, so are the septa that separate them, this means
aleoli almost merge together, decreasing the available surface area for gas
exchange
• Symptoms include dyspnea, hypoxemia, cough (with some sputum), and, over time, a
barrel chest can develop.
• Finally, in the exact same mechanism as with chronic bronchitis, pulmonary
hypertension can develop, leading to enlarged right side of the heart, and eventually
right sided heart failure.
• Types of emphysema include
• Centrilobular (where the central lobulus of the lung are most affected)
• Panlobular (where the lobules across the lungs are affected)
• Distal (paraseptal) emphysema (where the most distal lobules are affected)
33. Restrictive disturbances of breathing. Idiopathic pulmonary fibrosis.
Idiopathic = disease without a cause
Pulmonary = lung
Fibrosis = laying down of fibrous tissue
In effect, this is the ongoing repair process of the lungs which is a chronic problem leading to the
loss of lung tissue.
Pathophysiology
• Damage to type 1 alveolar cells which release Transforming Growth Factor beta 1
(TGFB1)
• Triggers type 2 alveolar cells to trigger fibroblast proliferation

VSM
• Fibroblasts convert to myofibroblasts
• These secrete reticular and elastic fibers
• While this is happening Type 2 alveolar cells, continue to
proliferate more than usual, stimulating even more reticular and
elastic fiber secretion
The etiology is unknown, but risk factors include:
• Old age
• Smoking
• And being male
This is a restrictive pulmonary disorder, this means
• Total lung capacity is decreased (unlike in obstructive forms where the TLC is
increased)
• FVC is also decreased
• FEV1 is also decreased
The disease is progressive, and can lead to dyspnea, cyanosis, coughing and digital clubbing.

34. Pulmonary edema.


Pulmonary edema is the buildup of fluid in the interstitial space of the lungs. This prevents easy
gas transfer between the alveoli and the capillaries.
Movement of fluids into the interstitial space in normal physiological conditions is governed by
three factors:
1. Hydrostatic pressure
1. Pressure of fluids in a confined space
2. Oncotic pressure
1. Higher in capillaries than in the interstitial fluid
3. Capillary permeability
In normal circumstances the sum total of these three factors is slightly in favour of movement
of fluid out of the capillaries and into the interstitial. However, where one of these factors is off,
and the sum of the factors greatly favours movement of of capillaries into the interstitial, we
see edema.
The cause of pulmonary edema can be:
• Cardiogenic
• Left sided heart failure most common form
• Blood backs up from left atrium to pulmonary veins and capillaries,
increasing hydrostatic pressure in the pulmonary vessels (pulmonary
hypertension)
• Severe systemic hypertension
• BP greater than 180/110. The high systemic pressure causes back up
into the pulmonary vessels once again
• Non-cardiogenic
• Injury to alveoli can trigger an inflammatory response
• Inflammatory response increased vessel permeability
• Sepsis
• Inflammatory process throughout the body, including lungs, therefore
once again increased vessel permeability
• Low oncotic pressure
• Due to loosing too many proteins (e.g. nephrotic syndrome) or not
making enough proteins (e.g. liver failure)
• Low oncotic pressure pushes the equation towards movement of fluid
out of the vessel
This increased fluid in the interstitial, decreases the rate of gas diffusion across the alveolar
membrane. Symptoms can include

VSM
• Dyspnea (Orthopnea in left sided heart failure)
X-ray is generally the best method of diagnosis.

35. Pulmonary embolism.


A pulmonary embolism is when a blockage lodges itself in a pulmonary artery. The most
common form of blockage is a thrombus (blood clot) which often develops in the superficial or
deep vessels of the lower limb. This process generally starts with endothelial damage, formation
of a platelet plug and coagulation cascade (for more details, see questions regarding clotting).
As the thrombus blocks the pulmonary arteries, it will prevent blood from getting to the alveoli.
This means that we’ll have a V/Q mismatch (as we’ll be getting full ventilation and reduced
blood flow so less perfusion).
The extent of the V/Q mismatch will depend on how high up the arterial tree the clot has
formed, the more of the lung deprived of blood, the greater the mismatch and the greater the
threat to the body.
Common responses to this V/Q mismatch will be hyperventilation as a result of respiratory
alkalosis.
Remember, Virchow’s triad are the three factors that contribute to thrombosis:
• Slowed blood flow (statis)
• Hypercoagulation (altered amounts of clotting factor)
• Damage to blood vessels
Small embolism may be asymptomatic. Large embolism leads to sudden and sharp chest pain,
shortness of breath and fatigue.

36. Hypoxia – definition and classification. Hypoxic hypoxia.


Hypoxia is defined as lack of oxygen in tissues (oxygen starvation) which can arise
because of various reasons. Hypoxia is usually preceded by hypoxemia – decreased
concentration of oxygen in blood. However, there can exist also hypoxia without
hypoxemia. Hypercapnia (elevated concentration of CO2 in blood) is frequently
found in hypoxic state, but not in every case. Sometimes hypoxia even can be
accompanied by hypocapnia
The consequences of hypoxia depend on
* time in which the hypoxia develops;
* duration of hypoxia;
* intensity and form of hypoxia itself;
* sensitivity of the affected tissues.
In order to understand the pathogenetic mechanism leading to hypoxia, one should
first clarify the basic external and internal factors responsible for adequate oxygen
supply of the body. These are as follows:
1/ sufficient amount of oxygen in the inhaled air;
2/ proper exchange of gases in the lungs (ventilation, diffusion and perfusion);
3/ sufficient amount of haemoglobin capable of oxygen transfer;
4/ proper function of cardiovascular system;

VSM
5/ ability of tissues to use oxygen (terminal oxidation).
The cardiovascular system integrates the various parts of the system: it assures that
sufficient amount of oxygen through basic functions of the respiratory system and the
red blood cells may reach the tissues.
The peculiar structure and function of haemoglobin enables to pick up oxygen very
quickly in the lungs, transport it at a considerable concentration to the tissues and -
last but not least - release it through a very sophisticated molecular regulatory
mechanism in the capillaries of the body.
The classification of various forms of hypoxia corresponds with the above mentioned
external and internal factors leading to hypoxia:
* Hypoxic hypoxia - The basic reason is decreased concentration or pressure of
oxygen in the inhaled air or a disturbance of respiration.
* Anaemic hypoxia - Various anaemic states, pathologic forms or blockage of
haemoglobin transport function.
* Circulatory (ischemic) hypoxia - Total or local disturbance of circulation, venous
stagnation of blood or ischemisation due to arterial closure.
* Histotoxic hypoxia (disturbed function [5]). Inability to utilize oxygen in the tissues.
Another possibility is the classification of hypoxia according to its time course:
In everyday medical practice the most frequent hypoxia is that one which is an
accompanying feature of chronic diseases of respiratory and cardiovascular system
or blood and therefore this is chronic hypoxia.
HYPOXIC HYPOXIA
The most common form of hypoxia, characterized by reduced oxygen tension in
pulmonary capillaries. Hypoxic hypoxia can be observed at low oxygen pressure in
inhaled air, at decreased ventilation of lungs or in the case of extensive pathological
processes in respiratory system leading to alveoli-capillary blockage. At reduced
oxygen tension in pulmonary capillaries haemoglobin in red cells cannot fully
saturate with oxygen and therefore hypoxemia develops.
The actual reasons of hypoxic hypoxia can be divided to two groups:
1/ "There is nothing to breathe" - in high altitude above the sea level (normal
concertation of oxygen, decreased pressure) or if the air contains decreased
proportion of oxygen despite its normal pressure (e.g. in cellars filled with CO2
instead of oxygen. Attention do not confound with CO intoxication!).
2/ "There is nothing to breathe with" - disturbed basic functions of the respiratory
system
If compensatory hyperventilation is possible and effective (e.g. in mountain climbing)
hypocapnia and respiratory alkalosis can ensue. Alkalosis further deteriorates the

VSM
release of oxygen from haemoglobin. On the contrary, if the respiration is insufficient,
hypercapnia and respiratory acidosis can develop.
37. Blood (hemic) hypoxia. Circulatory hypoxia.
From a clinical point of view, hypoxia has two main aspects:

• The state of inadequacy between delivery and necessity of O2 for the cell (resp mitochondria), tissue, organ,
organism.
• Failure of the oxygen regimen to satisfy the metabolic requirements of the organism for a normal lifestyle.

As inadequacy, hypoxia represents acute, critical condition threatening the vital functions

It can be a consequence of:

• Decreased O2 delivery in case of normal necessity (frequently)

OR

• Normal (sometimes increased) delivery, but in case of even greater O2 requirements (rarely)

As failure, hypoxia represents stable, chronically “limited O2 regimen”:

• Incapable of satisfying the vital requirements of the organism and turning normal man into “patient”.
• One or more stages in the O2 regimen are impaired with effectiveness and economy decreased to different extent.
• Engaging of body compensations ensures some O2 adequacy, but the level of daily activity is decreased and the
lifestyle of the patient is altered.

Classification of hypoxias

Anatomic Clinical
Respiratory Fulminant
Blood Acute
Cardio Circulatory Sub-acute
Tissue Chronic

Distribution Severity
Isolated (local) Light
General Medium
Occurrence Extreme
Systemic hypoxia Severe
Tissue hypoxia

Pathophysiologic classification

• Hypoxic hypoxia
• Hemic hypoxia
• Circulatory hypoxia
• Histotoxic hypoxia
• Mixed hypoxia

Hemic (blood) hypoxia

Basis of this hypoxia type is decrease of blood oxygen capacity. Two variants: anemic and toxic are possible.

• In case of anemic form, the total circulating hemoglobin level – owing to blood loss, erythrocytes hemolysis in
blood, channel or bone brain blood making function oppression decreases.

VSM
• Toxic form arises in case of hemic toxins poisoning. The general content of hemoglobin in blood remains normal,
but the contents of functionally active hemoglobin decreases. The part of hemoglobin turns into such compounds,
which are not capable to execute oxygen transport function.

More often hemoglobin gets linked with charcoal gas (carbon oxide, CO). This compound is named carboxihemoglobin. It’s
in 300 times more stable than oxyhemoglobin, therefore hypoxia arises even with insignificant CO concentration in the air.
Hemoglobin gets switched out off the transport function, oxygen capacity in blood decreases, hypoxia occurs.

Another one form toxic hemical hypoxia is known, which arises ground methhemoglobinemia origin. Methhemoglobinemia
are divided in two groups origin – hereditary (primary) and acquired (secondary).

• Hereditary methhemoglobinemia are stipulated atypic hemoglobin synthesis, derivation of endogenic products,
which turn hemoglobin to methhemoglobin; enzymes systems deficiency, which restore methemoglobin into
hemoglobin. Among exogenic methemoglobin makers the majors are following: nitrogen compounds (oxides,
nitrites), aminocompounds (hydroxylamine, aniline, phenylhydrasine, paraaminobensous acid), oxidizers
(chlorates, permanganates, hynones, pyridin, naphtaline), oxy-restoring paint (methylene blue, kresilblau),
medications (novocaine, pylocarpine, phenacetine, barbyturates, aspirin, resorcin and others).

In normally the contents of methhemoglobin in blood makes 0,3-3 g/l. Hemoglobin oxydisers can lift it’s level to extremely
high digits. If methhemoglobin concentration reaches up to 75 %, the death occurs.

Methhemoglobinemia is the purest form of hemic hypoxia. It’s mechanism is incorporated in erythrocyte itself in heme. It is
known, that hemoglobin connects oxygen in labile way not changing the valency of iron atom, which is in two-valent
peroxyde form. In action of just mentioned oxidizers there is an orbital electron offset, and the heme iron gets transformed
from two-valent into three-valent. Hemoglobin, turning into methhemoglobin (ferrohemoglobin), loses it’s active centre, and
oxygen transport function at the same time. Oxygen capacity of blood decreases.

In natural conditions in the person has got a methhemoglobin continuous derivation. This physiological
methhemoglobinemia is stipulated by effect of molecular oxygen upon hemoglobin. In a parallel to methhemoglobin
derivation there is a constant demethhemoglobinisation, that is opposite methhemoglobin transformation into hemoglobin.
This process is carried out by erythrocytes reductase system and unoxydated products – lactate, pyruvate and others. Thus
the dynamic equilibrium between the both hemoglobin and methhemoglobin contents, is supported, to hypoxia does not
arise.

In conditions of intoxication methhemoglobinmakers can oxidized hemoglobin directly or undirectly. The second mechanism
is reduced to enzymes systems damage, which catalyses restoring of methhemoglobin back into hemoglobin.

Showing of methhemoglobinemia depends on individual resistance of an organism – on general metabolism intensity on


antioxydase and reductase systems activity erythrocytes age and hemoglobin type. The oxidation speed, for example, Нb F
is higher than Нb А.

Methemoglobinemias FMetHb >1%

VSM
• Inborn (congenital) methemoglobinemia can be a consequence of:
• Atypical structure of Hb.
• Deficit of erythrocyte methemoglobin reductases.
• Acquired methemoglobinemia can be caused by the so called “methemoglobin developers” – nitrites, aniline
derivatives, same medicines – atebrine, sulphonamides, etc.
• Methemoglobin shifts the curve pO2/ctO2 to the left – impaired O2 delivery.

Sulphhemoglobinemias SHb = N/A

Sulphhemoglobin is irreversible inactive Hb (sulphonamides, H2S).

Its presence strongly shifts the curve pO2/ctO2 to the right and leads to combination of normoxia with hyposaturation.

The oxygen-transport capacity of Hb is decreased as a consequence of:

• Absolute deficit of Hb – anaemic type.


• Transport, binding and delivery of O2 by the Hb is altered – inactivity type.
• Inadequate (ineffective) oxygen transport heme to heme interaction in Hb – hypo- or hyperaffinity type.
• Combination of the above-mentioned types – combined type.

Circulatory Hypoxia

• Circulatory hypoxia is due to functionally insufficient local or general blood supply.

The main pathogenetic unit is insufficient tissue perfusion. Metabolism from “leader and conductor” of the circulation is
turned into dependent factor limited by blood flow.

• Inadequate tissue perfusion exists as two functional variants: hyperkinetic and hypokinetic.

Hyperkinetic vs. Hypokinetic

Hyperkinetic type – hypoxia at extreme exercise (limitation of physical capacity).

Hypokinetic type – usually at deranged:

• O2 transition in the lungs.


• O2 transport to the tissues.
• O2 distribution and/or
• O2 supply for the cells or intracellular structures in a tissue, organ etc.

Three types of circulatory hypoxia

• Ischemic type – spasm or occlusion of arterial blood vessel (Ischemic heart disease).
• Congestive type - impeded or completely stopped blood flow in one or more areas of the body (chronic heart
failure, venous thrombosis, etc).
• Mixed type – with initial predominantly ischemic and subsequent predominantly congestive stage (different kinds
of shock).

Ischemic type is characterized with:

• Combination of O2 and substrate deficit, together with impaired end metabolites wash out – quick
decompensation of cell metabolism.
• Predominantly hypoextraction tissue O2 deficit due to arterial-venous shunts and impaired tissue O2 diffusion.

Congestive type is characterized with:

VSM
• Prolonged contact time blood/tissue with increased O2 extraction. Cells get more O2 than usual so venous
hypoxia and hypoxemia ensue. The latter are the causes for increased arterial-venous O2 changes and cyanosis.
• Substrate delivery and end metabolites wash out are less affected

Impairment in O2 utilization (dysoxia, O2 utilization hypoxia, normoxic hypoxidosis)

Exists in two main pathogenetic variants:

• Insufficient or impossible reduction and extraction of O2 from the environment – histotoxic type, histohypoxia,
histohypoxic hypoxia.
• Insufficient or lost utilization of the O2 already entered the cells, resp. mitochondria – ineffective O2 utilization.

Ineffective utilization of O2

Can be seen in:

• Substrate deficit in mitochondria:


• blocked shuttle mechanisms
• suppressed Krebs cycle incapable flavoprotein pathway for electron delivery for the transelectronases
• selective block in substrate phosphorylation
• Uncoupling of oxidative phosphorilation pathogenic agents (decouplers) – 2,4 dinitrophenol, oligomycin, thyroid
hormones, bacterial toxins, low molecular peptides, free fatty acids, etc
• pathologic conditions – homeostatic disturbances, cell edema, excess of free radicals, jaundice, cholestasis, etc.
• Genetic or acquired (intoxications and infections) damage to the mitochondrial membranes or the so called
“special mitochondrial states”
• Ca2+ “leakage” from the mitochondria suppressed anionic and nucleotide transport by alkalosis
• blocked reverse electron transport by hyperoxia

38. Tissue hypoxia. Metabolic changes during hypoxia. Adaptation to hypoxia.


Tissue hypoxia is the end stage of different forms and pathogenetic variants of system hypoxia. It is a condition
characterized with low tissue O2 and passes through four stages (phases):

• Saturation - Compensated O2 deficit (without O2 debt)


• Adaptation
• Hypoaerobiosis - Uncompensated O2 deficit (with O2 debt)
• Destruction

Saturation phase

• Most frequently it is transitional phase, merging with the next one – metabolic modulation of the aerobic
metabolism. Saturation phase is best revealed in striated muscles and the heart and is absent in the neurons of
CNS.
• The cells function in low pO2 environment. Their work possibility and reserve functional capacity are decreased,
resp. the product activity x time is also lowered

Adaptation (metabolic) phase

Adaptive modulation of aerobic metabolism includes:

• Increased flow of reduction equivalents from the cytosol to the mitochondria.


• Mobilization of electron supply for cytochrome C (activate electron sideways).
• Complete utilization of O2 by oxysomes, resp. respiratory chains as a result of increased electron flow.
• Keeping O2 flow unchanged at low pO2.
• Initial preservation and later increase of the coupling of oxidative, substrate and total phosphorilation in the
mitochondria.
• Stimulating the synthesis of stress proteins by hypoxic signals.

Gains for the cells during this phase:

VSM
• Increased cell resp. mitochondrial resistance in hypoxic environment.
• Chance to preserve function in progressively hypoxic environment.

Disadvantages for the cells as a result of adaptation:

• Slow and imprecise response of the hypoxic cells to tele- and paracrinic signals.
• Faster “wear and tear” of cell structures.
• Higher risk of activating cell’s apoptosis program

Hypoaerobiosis (dysoxic) phase

• Deficit of oxidative ATP production.


• Activating the production of non-oxidative ATP production– active glycolysis.
• O2 delivery dependent O2 consumption.
• Low respiratory mitochondrial control, resp. initial decoupling of the oxidative phosphorilation.
• Restructuring of the energy expenditures with restriction of facultative and supporting of the essential (vital) cell
metabolism.

Destructive (necrotic) phase

• It is consequence to extreme or medium but persistent and/or progressive tissue hypoxia.


• Cell’s hypo- or anoxic alteration can be reversible or irreversible.
• Cell’s essential metabolism is not working properly or is entirely suppressed.

Changes in cells

• Mitochondrial changes - transition to the so called “highly permeable state”


• Cytosolic changes - intracellular acidosis with secondary glycolytic block
• Nuclear changes - Suppressed polymerases
• Lysosomal changes - destroyed lysosomes and their enzymes spill in the cytosol

Adaptation to hypoxia

General principles:

• Adaptations are effective in light, relatively effective in medium and ineffective in severe hypoxic states.
• Adaptations are most stable in hypoxic hypoxia, less expressed in hemic and circulatory hypoxia and least stable
in histotoxic hypoxia.
• The following factors influence the expression and effectiveness of compensation:
- The degree and duration of hypoxia.
- Pathogenetic factors forming different types of hypoxia. Ontogenetic stage of the organism.
- The hypoxic pattern – continuous, periodic, impulse, etc
• Genetic integrity of O2 dependent cell genes and mechanisms for their activation.

Lung adaptations

• Increased lung and alveolar ventilation:


- hemoreflex mechanism (hours, days)
- compensatory change in the relation ventilation/perfusion - VA/Q (months, years)
• Increased pulmonary blood volume with higher O2 extraction in the alveoli. Increased diffusion capacity of the
alveolarcapillary membrane.
• More O2 brought to the organism

Cardio – vascular adaptations

• Increased cardiac output increased heart rate (tachycardia) increased stroke volume
• Rise in the arterial blood pressure
- better tissue perfusion
• Increased vascularization and capillarization
-recruiting of reserve capillaries

VSM
-formation of new capillaries
• Augmented transition of interstitial fluid to the vessels
-better rheology of the blood
-easy O2 diffusion

Larger amount of O2 transported and delivered to the cells.

Blood adaptations

They are among the most effective adaptations!

• Compensatory erythrocytosis: -Rapid – mobilization of erythrocytes fro the depots.


-Slow – intense erythropoesis (as a result of erythropoetin production).
• Adaptive change of O2 binding function of hemoglobin
-High concentration of 2,3 DPG in erythrocytes, shift of Hb dissociation curve to the right with easier O2 release.

More O2 bound and released per unit breathing area.

Cell adaptations

• Increased O2 reserve in the cells with O2 depots like myoglobin in muscles.


• More effective delivery of electrons to transelectronase enzymes.
• Increased O2 affinity of the terminal cytochromoxydases.
• Growing number of respiratory chains in a mitochondrion and mitochondria in a cell

39. Cardiac arrhythmias. Bradycardia. Atrioventricular block.


An abnormality of the cardiac rhythm is called a cardiac arrhythmia. Arrhythmias may cause sudden death, syncope, heart
failure, dizziness, palpitations or no symptoms at all. There are two main types of arrhythmia:

1. bradycardia: the heart rate is slow (< 60 b.p.m.)

2. tachycardia: the heart rate is fast (> 100 b.p.m.).

Tachycardias are more symptomatic when the arrhythmia is fast and sustained. Tachycardias are subdivided into
supraventricular tachycardias, which arise from the atrium or the atrioventricular junction, and ventricular tachycardias,
which arise from the ventricles.

Bradycardia:

Atrial bradycardias are divided into three types.

Respiratory sinus arrhythmia:

• Usually found in young and healthy adults.

• Heart rate increases during inhalation and decreases during exhalation.

• If the decrease during exhalation drops the heart rate below 60 bpm on each breath, this type of bradycardia is
usually deemed benign and a sign of good autonomic tone.

Sinus bradycardia:

• A sinus rhythm of less than 60 BPM.

• It is a common condition found in both healthy individuals and those considered well-conditioned athletes. The
heart muscle of athletes has become conditioned to have a higher stroke volume, so requires fewer contractions
to circulate the same volume of blood.

Sick sinus syndrome:

VSM
• Covers conditions that include severe sinus bradycardia, sinoatrial block, sinus arrest, and bradycardia-
tachycardia syndrome (atrial fibrillation, flutter, and paroxysmal supraventricular tachycardia).

Atrioventricular bock:

It is a partial or complete interruption of impulses transmission from Atrium to Ventricle.

According to relation between Atrium and Ventricle, we can detect three degrees of AV heart block

• First Degree Heart Block: slowing of Conduction.


• Second Degree Heart Block: intermittent interruption of conduction subtype into ▪ Mobitz Type I. ▪ Mobitz Type II.
• Third Degree (Complete) Heart Block: Complete interruption of conduction

First Degree Heart Block:

It is not considered complete block, it is just slowdown of impulses that come from SA node more than the normal

Second Degree Heart Block:

Mobitz I

• This problem occurs at the level of AV node itself.


• It also is not considered a complete block

ECG Manifestations:

• It is characterized by progressive prolongation of PR interval until dropped QRS, then the cycle start again .
• Constant PP interval
• Irregular Rhythm
• Normal or slightly slow Rate

Mobitz II

• This type of block occurs below AV node at the level of Hiss Bundle.
• Also, is considered incomplete but high risk to be complete.
• Some of electrical impulses are unable to reach ventricles

ECG Manifestation:

• Recurrent appearance of non-conducted P waves which is blocked and not followed by QRS complex (indicate to
block of impulses to reach ventricle)
• PR interval and PP interval are constant
• QRS usually normal but sometimes become Wide

Third Degree (Complete) Heart Block

• Characterized by Atrio-ventricular dissociation.


• This blockage level is infra-nodal (Bilateral Bundle Branches).
• Atrial and ventricular activities are unrelated due to complete block of electrical impulses to reach the ventricle.
• Another pacemaker distal to the block takes over in order to activate the ventricles or ventricular standstill will
occur.

ECG manifestation:

• Dissociation between P wave and QRS


• P wave may overlap on T wave or QRS complex
• PR interval is not constant Rate usually less than 40.
• QRS complex usually wide and sometimes normal

Clinical Manifestation:

VSM
• Usually first degree and sometimes second degree are asymptomatic.
• The most common signs and symptoms:
• Sever Bradycardia
• Hypotension
• Syncope (fainting)
• Chest pain
• Dyspnea
• Dizziness

General Management:

• Cardiac monitoring: for close observation.


• Oxygen supply: to Manage de-saturated patients.
• IV Line: To support blood pressure with fluids.
• Atropine standby: to treat bradycardia especially incomplete degrees

Management of heart block depend on symptoms

First degree heart block: this type usually is asymptomatic and not indicated for treatment

• Close observation of Hemodynamic status.


• Discontinue of some medication that cause bradycardia such as:
• Beta-blockers: Concor
• Digoxins : Lanoxine
• calcium channel blockers: Diltiazem

Second Degree and Complete heart block:

• Usually these degrees are associated with sever bradycardia which can be treated by atropine.
• Associated conditions should be treated correctly such as: ▪ Myocardial infarction. ▪ Electrolyte disturbance
(hyperkalemia). ▪ Digitals intoxication.
• Transvenous temporary pacemakeris indicated for pt with sever bradycardia who has no effect of Atropine
administration (For 24 hours : 48 hours)
• Transcutanous permanent pace-maker is indicated for chronic AV block

40. Tachyarrhythmias.
Tachycardia, also called tachyarrhythmia, is a heart rate that exceeds the normal
resting rate. In general, a resting heart rate over 100 beats per minute is accepted as
tachycardia in adults. Heart rates above the resting rate may be normal (such as with
exercise) or abnormal (such as with electrical problems within the heart).
Tachycardia of ectopic origin. Even when the stimulus formation in the sinus node is
normal, abnormal ectopic excitations can start from a focus in an atrium(atrial), the
AV node (nodal), or a ventricle (ventricular). High-frequency ectopic atrial
depolarizations (saw-toothed base line instead of regular P waves in the ECG) cause
atrial tachycardia, to which the human ventricles can respond to up to a rate of ca.
200 per minute. At higher rates, only every second or third excitation may be
transmitted, as the intervening impulses fall into the refractory period of the more
distal conduction system, the conduction component with the longest AP being the
determining factor. This is usually the Purkinje fibres which act as frequency filters,
because their long action potential stays refractory the longest, so that at a certain
rate further transmission of the stimulus is blocked. At higher rates of discharge of
the atrial focus (up to 350 per minute=atrial flutter; up to 500 per minute=atrial
fibrillation), the action potential is transmitted only intermittently. Ventricular

VSM
excitation is therefore completely irregular (absolutely arrhythmic). Ventricular
tachycardia is characterized by a rapid succession of ventricular depolarizations. It
usually has its onset with an extra systole. Ventricular filling and ejection are reduced
and ventricular fibrillation occur (high-frequency and uncoordinated twitching’s of the
myocardium). If no countermeasures are taken, this condition is just as fatal as
cardiac arrest, because of the lack of blood flow.
41. Congestive heart failure. Left ventricular failure.
Heart failure is a physiological state in which cardiac output is insufficient to meet the
needs of the body and lungs. The term "congestive heart failure" is often used, as
one of the common symptoms is congestion, or build-up of fluid in a person's tissues
and veins in the lungs or other parts of the body. Specifically, congestion takes the
form of water retention and swelling (edema), both as peripheral edema (causing
swollen limbs and feet) and as pulmonary edema (causing breathing difficulty), as
well as ascites (swollen abdomen).
Heart failure symptoms are traditionally and somewhat arbitrarily divided into "left"
and "right" sided, recognizing that the left and right ventricles of the heart supply
different portions of the circulation.

The left side of the heart is responsible for receiving oxygen-rich blood from the
lungs and pumping it forward to the systemic circulation (the rest of the body except
for the pulmonary circulation). Failure of the left side of the heart causes blood to
back up (be congested) into the lungs, causing respiratory symptoms as well as
fatigue due to insufficient supply of oxygenated blood. Common respiratory signs are
increased rate of breathing and increased work of breathing (non-specific signs of
respiratory distress). Rales or crackles, heard initially in the lung bases, and when
severe, throughout the lung fields suggest the development of pulmonary edema
(fluid in the alveoli). Cyanosis which suggests severe low blood oxygen, is a late sign
of extremely severe pulmonary edema.
Additional signs indicating left ventricular failure include a laterally displaced apex
beat (which occurs if the heart is enlarged) and a gallop rhythm (additional heart
sounds) may be heard as a marker of increased blood flow or increased intra-cardiac
pressure. Heart murmurs may indicate the presence of valvular heart disease, either
as a cause (e.g. aortic stenosis) or as a result (e.g. mitral regurgitation) of the heart
failure.
Backward failure of the left ventricle causes congestion of the lungs' blood vessels,
and so the symptoms are predominantly respiratory in nature. Backward failure can
be subdivided into the failure of the left atrium, the left ventricle or both within the left
circuit. The patient will have dyspnea (shortness of breath) on exertion and in severe
cases, dyspnea at rest. Increasing breathlessness on lying flat, called orthopnea,
occurs. It is often measured in the number of pillows required to lie comfortably, and
in orthopnoea, the patient may resort to sleeping while sitting up. Another symptom
of heart failure is paroxysmal nocturnal dyspnea: a sudden night time attack of

VSM
severe breathlessness, usually several hours after going to sleep. Easy fatigability
and exercise intolerance are also common complaints related to respiratory
compromise.
"Cardiac asthma" or wheezing may occur.
Compromise of left ventricular forward function may result in symptoms of poor
systemic circulation such as dizziness, confusion and cool extremities at rest.
Heart failure is caused by any condition which reduces the efficiency of the heart
muscle, through damage or overloading. As such, it can be caused by a wide
number of conditions, including myocardial infarction (in which the heart muscle is
starved of oxygen and dies), hypertension (which increases the force of contraction
needed to pump blood) and amyloidosis (in which misfolded proteins are deposited
in the heart muscle, causing it to stiffen). Over time these increases in workload will
produce changes to the heart itself:
The heart of a person with heart failure may have a reduced force of contraction due
to overloading of the ventricle. In a healthy heart, increased filling of the ventricle
results in increased contraction force (by the Frank–Starling law of the heart) and
thus a rise in cardiac output. In heart failure, this mechanism fails, as the ventricle is
loaded with blood to the point where heart muscle contraction becomes less efficient.
This is due to reduced ability to cross-link actin and myosin filaments in over-
stretched heart muscle
42. Congestive heart failure. Right ventricular failure.

Right-sided heart failure is often caused by pulmonary heart disease (cor


pulmonale), which is usually caused by difficulties of the pulmonary circulation, such
as pulmonary hypertension or pulmonic stenosis.
Physical examination may reveal pitting peripheral edema, ascites, and liver
enlargement. Jugular venous pressure is frequently assessed as a marker of fluid
status, which can be accentuated by eliciting hepatojugular reflux. If the right
ventricular pressure is increased, a parasternal heave may be present, signifying the
compensatory increase in contraction strength.
Backward failure of the right ventricle leads to congestion of systemic capillaries.
This generates excess fluid accumulation in the body. This causes swelling under
the skin (termed peripheral edema or anasarca) and usually affects the dependent
parts of the body first (causing foot and ankle swelling in people who are standing
up, and sacral edema in people who are predominantly lying down). Nocturia
(frequent night time urination) may occur when fluid from the legs is returned to the
bloodstream while lying down at night. In progressively severe cases, ascites (fluid
accumulation in the abdominal cavity causing swelling) and liver enlargement may
develop. Significant liver congestion may result in impaired liver function (congestive

VSM
hepatopathy), and jaundice and even coagulopathy (problems of decreased or
increased blood clotting) may occur.

43. Endocarditis and myocarditis.


Endocarditis is an inflammation of the inner layer of the heart, the endocardium. It
usually involves the heart valves (native or prosthetic valves). Other structures that
may be involved include the interventricular septum, the chordae tendineae, the
mural endocardium, or even on intracardiac devices. Endocarditis is characterized
by a prototypic lesion, the vegetation, which is a mass of platelets, fibrin,
microcolonies of microorganisms, and scant inflammatory cells. In the subacute form
of infective endocarditis, the vegetation may also include a center of granulomatous
tissue, which may fibrose or calcify.
There are multiple ways to classify endocarditis. The simplest classification is based
on etiology: either infective or non-infective, depending on whether a microorganism
is the source of the inflammation or not. Regardless, diagnosis of endocarditis is
based on the clinical features, investigations such as echocardiogram, as well as any
blood cultures demonstrating the presence of endocarditis-causing microorganisms.
Infective endocarditis Since the valves of the heart do not receive any dedicated
blood supply, defensive immune mechanisms (such as white blood cells) cannot
directly reach the valves via the bloodstream. If an organism (such as bacteria)
attaches to a valve surface and forms a vegetation, the host immune response is
blunted. The lack of blood supply to the valves also has implications on treatment,
since drugs also have difficulty reaching the infected valve.
Non-infective endocarditis Nonbacterial thrombic endocarditis (NBTE) or marantic
endocarditis is most commonly found on previously undamaged valves.[2] As
opposed to infective endocarditis, the vegetations in NBTE are small, sterile, and
tend to aggregate along the edges of the valve or the cusps.[2] Also unlike infective
endocarditis, NBTE does not cause an inflammation response from the body.[2]
NBTE usually occurs during a hypercoagulable state such as system wide bacterial
infection, or pregnancy, though it is also sometimes seen in patients with venous
catheters.[2] NBTE may also occur in patients with cancers, particularly mucinous
adenocarcinoma[2] where Trousseau syndrome can be encountered. Typically
NBTE does not cause many problems on its own, but parts of the vegetations may
break off and embolize to the heart or brain, or they may serve as a focus where
bacteria can lodge, thus causing infective endocarditis.[2]

Myocarditis or inflammatory cardiomyopathy is inflammation of heart muscle


(myocardium).The pathogenesis of myocarditis is not entirely clear. However, in
viral-mediated myocarditis, animal models have implicated 3 significant mechanisms
An infectious organism directly invades the myocardium
Local and systemic immunological activation quickly ensues

VSM
Cellular (CD4+) and humoral (B-cell clonal multiplication) activation occurs,
causing worsening local inflammation, anti-heart antibody production and further
myonecrosis.
All 3 may occur in the same host. However, the predominant pathway may differ
depending on the individual characteristics of the infectious species and the innate
defenses of the host organism.
The first phase is characterised by viraemia in the host organism. During this time,
the cardiotropic RNA virus enters the host myocyte via receptor-mediated
endocytosis. Here, the viral RNA is translated into viral protein, and the viral
genome is incorporated into the host-cell DNA as doublestranded RNA. This latter
mechanism has been shown to cleave dystrophin, which is thought to directly cause
myocyte dysfunction. During the second and third phases, macrophages, natural
killer cells, and other inflammatory cells infiltrate the myocardium. Once in the
myocardium, these cells express inflammatory cytokines including interleukin-1,
interleukin-2, interferon-gamma, and TNF. This results in increased cytokine
production and causes endothelial cell activation resulting in further infiltration of
inflammatory cells. TNF alone also acts as a negative inotrope. In addition, auto-
antibodies directed against myocardial contractile and structural proteins are
produced. This is thought to have cytopathic effects on energy metabolism, calcium
homeostasis and signal transduction, and also to cause complement activation
resulting in the lysis of myocytes.
44. Valvular heart disease. Aortic stenosis. Aortic regurgitation.
-Valvular heart disease is any disease process involving one or more of the valves of
the heart. Valve problems may be congenital or acquired. Treatment may be with
medication but often involves valve repair or replacement.
-Aortic valve stenosis is a disease of the heart valves in which the opening of the
aortic valve is narrowed. The aortic valve is the valve between the left ventricle of the
heart and the aorta, which is the largest artery in the body and carries the entire
output of blood.
symptoms:
-Congestive heart failure carries a grave prognosis in patients with AS.
CHF in the setting of AS is due to a combination of systolic dysfunction and diastolic
dysfunction.
-Syncope (fainting spells) from aortic valve stenosis is usually exertional.In the
setting of heart failure it increases the risk of death.When the patient exercises, their
peripheral vascular resistance will decrease as the blood vessels of the skeletal
muscles dilate to allow the muscles to receive more blood to allow them to do more
work. This decrease in peripheral vascular resistance is normally compensated for
by an increase in the cardiac output. Since patients with severe AS cannot increase
their cardiac output, the blood pressure falls and the patient will syncopize due to
decreased blood perfusion to the brain.

VSM
-Angina in the setting of heart failure also increases the risk of death.
Angina in the setting of AS is secondary to the left ventricular hypertrophy that is
caused by the constant production of increased pressure required to overcome the
pressure gradient caused by the AS. The ischemia may first be evident during
exercise, when the heart muscle requires increased blood supply to compensate for
the increased workload.
Cause:
-Aortic stenosis is most commonly caused by age-related progressive calcification of
a normal (threeleafed) aortic valve mean age 65 to 70 years old - most common
cause. Other causes include calcification of a congenital bicuspid aortic valve most
common cause and acute rheumatic fever post-inflammatory.
pathophysiology:
The aortic valve normally consists of three leaflets. When the left ventricle contracts,
it forces blood through the valve to the aorta and then to the rest of the body. When
the LV expands again, the aortic valve prevents the blood from returning to the
ventricle. When the opening of the aortic valve becomes narrowed or constricted, the
blood can't be pumped adequately and the pressure in the left ventricle increases.
Initially, the LV compensates by thickening its walls (myocardial hypertrophy) in
order to maintain adequate pumping pressure. The type of hypertrophy most
commonly seen in AS is concentric hypertrophy, in which the walls of the LV are
equally thickened. In the later stages, the left ventricle dilates, the wall thins, and the
systolic function deteriorates. Morris and Inasmuch et al showed that different
coronary anatomy is associated with different valve diseases. There is more
research going on to see if different coronary anatomy might lead to turbulent flow at
the level of valves leading to inflammation and degeneration.

-Valvular heart disease is any disease process involving one or more of the valves of
the heart. Valve problems may be congenital or acquired. Treatment may be with
medication but often involves valve repair or replacement.
-Aortic insufficiency, also known as aortic regurgitation, is the leaking of the aortic
valve of the heart that causes blood to flow in the reverse direction during ventricular
diastole, from the aorta into the left ventricle.
Aortic insufficiency can be due to abnormalities of either the aortic valve or the aortic
root.
-Pathophysiology:
In aortic insufficiency, when the pressure in the left ventricle falls below the pressure
in the aorta, the aortic valve is not able to completely close. This causes a leaking of
blood from the aorta into the left ventricle. This means that some of the blood that
was already ejected from the heart is regurgitating back into the heart. The
percentage of blood that regurgitates back through the aortic valve due to AI is
known as the regurgitant fraction. For instance, if an individual with AI has a stroke

VSM
volume of 100 ml and during ventricular diastole 25 ml regurgitates back through the
aortic valve, the regurgitant fraction is 25%. This regurgitant flow causes a decrease
in the diastolic blood pressure in the aorta, and therefore an increase in the pulse
pressure. Thus, physical examination will reveal a bounding pulse, especially in the
radial artery.
Since some of the blood that is ejected during systole regurgitates back into the left
ventricle during diastole, there is decreased effective forward flow in AI.
-while diastolic blood pressure is diminished and the pulse pressure widens, systolic
blood pressure generally remains normal or can even be slightly elevated. This is
because sympathetic nervous system and the renin-angiotensin-aldosterone axis of
the kidneys compensate for the decreased cardiac output. Catecholamines will
increase the heart rate and increase the strength of ventricular contraction, directly
increasing cardiac output. Catecholamines will also cause peripheral
vasoconstriction, which causes increased systemic vascular resistance and ensures
that core organs are adequately perfused. Renin, a proteolytic enzyme, cleaves
angiotensinogen to angiotensin I, which is converted to angiotensin II, which is also a
potent vasoconstrictor. In the case of chronic aortic insufficiency with resultant
cardiac remodeling, heart failure will develop, and it is possible to see systolic
pressures diminish.
Aortic insufficiency causes both volume overload and pressure overload of the heart.
The pressure overload (due to elevated pulse pressure and the systemic effects of
neuroendocrine hormones) causes left ventricular hypertrophy. There is both
concentric hypertrophy and eccentric hypertrophy in AI. The concentric hypertrophy
is due to the increased left ventricular systolic pressures associated with AI, while
the eccentric hypertrophy is due to volume overload caused by the regurgitant
fraction.
45. Valvular heart disease. Mitral stenosis. Mitral regurgitation.
-Valvular heart disease is any disease process involving one or more of the valves of
the heart. Valve problems may be congenital or acquired. Treatment may be with
medication but often involves valve repair or replacement.
-Mitral stenosis is a valvular heart disease characterized by the narrowing of the
orifice of the mitral valve of the heart.
-Symptoms include:
dyspnea, orthopnea
Palpitations
Chest pain
Thromboembolism in later stages when the left ventricular volume is increased
Ascites and edema and hepatomegaly
-Cause:

VSM
Almost all cases of mitral stenosis are due to disease in the heart secondary to
rheumatic fever and the consequent rheumatic heart disease. Uncommon causes of
mitral stenosis are calcification of the mitral valve leaflets, and as a form of
congenital heart disease. However, there are primary causes of mitral stenosis that
emanate from a cleft mitral valve.Other causes include infective endocarditis where
the vegetations may favor increase risk of stenosis. It is the most common valvular
heart disease in pregnancy.
-Pathophysiology:
The normal area of the mitral valve orifice is about 4 to 6 cm2. In normal cardiac
physiology, the mitral valve opens during left ventricular diastole, to allow blood to
flow from the left atrium to the left ventricle. A normal mitral valve will not impede the
flow of blood from the left atrium to the left ventricle during diastole, and the
pressures in the left atrium and the left ventricle during ventricular diastole will be
equal. The result is that the left ventricle gets filled with blood during early ventricular
diastole, with only a small portion of extra blood contributed by contraction of the left
atrium during late ventricular diastole.
When the mitral valve area goes below 2 cm2, the valve causes an impediment to
the flow of blood into the left ventricle, creating a pressure gradient across the mitral
valve. This gradient may be increased by increases in the heart rate or cardiac
output. As the gradient across the mitral valve increases, the amount of time
necessary to fill the left ventricle with blood increases. Eventually, the left ventricle
requires the atrial kick to fill with blood. As the heart rate increases, the amount of
time that the ventricle is in diastole and can fill up with blood decreases. When the
heart rate goes above a certain point, the diastolic filling period is insufficient to fill
the ventricle with blood and pressure builds up in the left atrium, leading to
pulmonary congestion.
When the mitral valve area goes less than 1 cm2, there will be an increase in the left
atrial pressures. Since the normal left ventricular diastolic pressures is about 5
mmHg, a pressure gradient across the mitral valve of 20 mmHg due to severe mitral
stenosis will cause a left atrial pressure of about 25 mmHg. This left atrial pressure is
transmitted to the pulmonary vasculature and causes pulmonary hypertension.
Pulmonary capillary pressures in this level cause an imbalance between the
hydrostatic pressure and the oncotic pressure, leading to extravasation of fluid from
the vascular tree and pooling of fluid in the lungs.
The constant pressure overload of the left atrium will cause the left atrium to
increase in size. As the left atrium increases in size, it becomes more prone to
develop atrial fibrillation. When atrial fibrillation develops, the atrial kick is lost.
In individuals with severe mitral stenosis, the left ventricular filling is dependent on
the atrial kick. The loss of the atrial kick due to atrial fibrillation can cause a
precipitous decrease in cardiac output and sudden congestive heart failure.[citation
needed]
Patients with mitral stenosis prompts a series of hemodynamic changes that
frequently cause deterioration of the patient's clinical status. A reduction in cardiac

VSM
output, associated with acceleration of heart rate and shortening of the diastolic time,
frequently leads to congestive heart failure. In addition, when AF sets in, systemic
embolization becomes a real danger.
-Valvular heart disease is any disease process involving one or more of the valves of
the heart. Valve problems may be congenital or acquired. Treatment may be with
medication but often involves valve repair or replacement.
-Mitral regurgitation, mitral insufficiency or mitral incompetence is a disorder of the
heart in which the mitral valve does not close properly when the heart pumps out
blood. It is the abnormal leaking of blood from the left ventricle, through the mitral
valve, and into the left atrium, when the left ventricle contracts. MR is the most
common form of valvular heart disease.
-symptoms:
shortness of breath, pulmonary edema, orthopnea, and paroxysmal nocturnal
dyspnea, palpitations, cardiogenic shock.
-causes:
The mitral valve is composed of two valve leaflets, the mitral valve annulus, the
papillary muscles and the chordae tendineae. A dysfunction of any of these portions
of the mitral valve apparatus can cause mitral regurgitation.
The most common cause of mitral regurgitation is mitral valve prolapse, which in turn
is caused by myxomatous degeneration.The most common cause of primary mitral
regurgitation is myxomatous degeneration of the valve. Myxomatous degeneration of
the mitral valve is more common in females, and is more common in advancing age.
This causes a stretching out of the leaflets of the valve and the chordae tendineae.
The elongation of the valve leaflets and the chordae tendineae prevent the valve
leaflets from fully coapting when the valve is closed, causing the valve leaflets to
prolapse into the left atrium, thereby causing mitral regurgitation.
Ischemic heart disease causes mitral regurgitation by the combination of ischemic
dysfunction of the papillary muscles, and the dilatation of the left ventricle that is
present in ischemic heart disease, with the subsequent displacement of the papillary
muscles and the dilatation of the mitral valve annulus.
Rheumatic fever and Marfan's syndrome are other typical causes of mitral
regurgitation.
Secondary mitral regurgitation is due to the dilatation of the left ventricle, causing
stretching of the mitral valve annulus and displacement of the papillary muscles. This
dilatation of the left ventricle can be due to any cause of dilated cardiomyopathy,
including aortic insufficiency, nonischemic dilated cardiomyopathy and
Noncompaction Cardiomyopathy. It is also called functional mitral regurgitation,
because the papillary muscles, chordae, and valve leaflets are usually normal.
-Pathophysiology:
1-Acute phase

VSM
Acute mitral regurgitation causes a sudden volume overload of both the left atrium
and the left ventricle. The left ventricle develops volume overload because with every
contraction it now has to pump out not only the volume of blood that goes into the
aorta, but also the blood that regurgitates into the left atrium.
In the acute setting, the stroke volume of the left ventricle is increased, this happens
because of more complete emptying of heart. However, as it progresses the LV
volume increases and the contractile function deteriorates and thus leading to
dysfunctional LV and a decrease in ejection fraction.
The regurgitant volume causes a volume overload and a pressure overload of the
left atrium. The increased pressures in the left atrium inhibit drainage of blood from
the lungs via the pulmonary veins. This causes pulmonary congestion.
2-Chronic phase:
a) Compensated:
If the mitral regurgitation develops slowly over months to years or if the acute phase
cannot be managed with medical therapy, the individual will enter the chronic
compensated phase of the disease. In this phase, the left ventricle develops
eccentric hypertrophy in order to better manage the larger than normal stroke
volume. The eccentric hypertrophy and the increased diastolic volume combine to
increase the stroke volume so that the forward stroke volume approaches the normal
levels.
In the left atrium, the volume overload causes enlargement of the chamber of the left
atrium, allowing the filling pressure in the left atrium to decrease. This improves the
drainage from the pulmonary veins, and signs and symptoms of pulmonary
congestion will decrease.
b) Decompensated
An individual may be in the compensated phase of mitral regurgitation for years, but
will eventually develop left ventricular dysfunction, the hallmark for the chronic
decompensated phase of mitral regurgitation. It is currently unclear what causes an
individual to enter the decompensated phase of this disease. However, the
decompensated phase is characterized by calcium overload within the cardiac
myocytes.
In this phase, the ventricular myocardium is no longer able to contract adequately to
compensate for the volume overload of mitral regurgitation, and the stroke volume of
the left ventricle will decrease.The left ventricle begins to dilate during this phase.
This causes a dilatation of the mitral valve annulus, which may worsen the degree of
mitral regurgitation. The dilated left ventricle causes an increase in the wall stress of
the cardiac chamber as well.
While the ejection fraction is less in the chronic decompensated phase than in the
acute phase or the chronic compensated phase of mitral regurgitation, it may still be
in the normal range and may not decrease until late in the disease course. A
decreased ejection fraction in an individual with mitral regurgitation and no other

VSM
cardiac abnormality should alert the physician that the disease may be in its
decompensated phase.
46. Coronary artery disease. Angina pectoris.
-Coronary artery disease is the result of the accumulation of atheromatous plaques
within the walls of the coronary arteries that supply the myocardium with oxygen and
nutrients,the filling up of the plaque in the lumen of artery causes narrowing of lumen
of the artery by decreasing its diameter. It is sometimes also called coronary heart
disease.
-Pathophysiology:
Limitation of blood flow to the heart causes ischemia of the myocardial cells.
Myocardial cells may die from lack of oxygen and this is called a myocardial
infarction. It leads to heart muscle damage, heart muscle death and later myocardial
scarring without heart muscle regrowth. Chronic high-grade stenosis of the coronary
arteries can induce transient ischemia which leads to the induction of a ventricular
arrhythmia, which may terminate into ventricular fibrillation leading to death. -Angina
pectoris is chest pain due to ischemia of the heart muscle, generally due to
obstruction or spasm of the coronary arteries. Coronary artery disease, the main
cause of angina, is due to atherosclerosis of the coronary arteries.
-Classification:
Stable angina
Typical presentations of stable angina is that of chest discomfort and associated
symptoms precipitated by some activity with minimal or non-existent symptoms at
rest.
Unstable angina
this is defined as angina pectoris that changes or worsens.
It has at least one of these three features:
it occurs at rest, usually lasting >10 min;
it is severe and of new onset;
it occurs with a crescendo pattern.
UA may occur unpredictably at rest which may be a serious indicator of an
impending heart attack. What differentiates stable angina from unstable angina is the
pathophysiology of the atherosclerosis. The pathophysiology of unstable angina is
the reduction of coronary flow due to transient platelet aggregation on apparently
normal endothelium, coronary artery spasms or coronary thrombosis.The process
starts with atherosclerosis, and when inflamed leads to an active plaque, which
undergoes thrombosis and results in acute ischemia, which finally results in cell
necrosis after calcium entry.
Microvascular angina

VSM
Microvascular Angina or Angina Syndrome X is characterized by angina-like chest
pain.The cause of is unknown, but it appears to be the result of poor function in the
tiny blood vessels of the heart, arms and legs.
-symptoms:
the discomfort is usually described as a pressure, heaviness, tightness, squeezing,
burning, or choking sensation. Typical locations for referred pain are arms,
shoulders, and neck into the jaw. Angina is typically precipitated by exertion or
emotional stress.
-Major risk factors:
Age (= 55 years for men, = 65 for women)
Cigarette smoking
Diabetes mellitus
Hypertension
Kidney disease
Obesity
Medications
vasodilators
excessive thyroid replacement
vasoconstrictors
-Pathophysiology:
Angina results when there is an imbalance between the heart's oxygen demand and
supply. This imbalance can result from an increase in demand without a proportional
increase in supply.
47. Coronary artery disease. Myocardial infarction.
-Coronary artery disease is the result of the accumulation of atheromatous plaques
within the walls of the coronary arteries that supply the myocardium with oxygen and
nutrients,the filling up of the plaque in the lumen of artery causes narrowing of lumen
of the artery by decreasing its diameter. It is sometimes also called coronary heart
disease.
-Myocardial infarction results from the interruption of blood supply to a part of the
heart, causing heart cells to die. This is most commonly due to blockage of a
coronary artery following the rupture of a vulnerable atherosclerotic plaque, which is
an unstable collection of lipids and white blood cells in the wall of an artery.
Classical symptoms of acute myocardial infarction include sudden chest pain,
shortness of breath, nausea, vomiting, palpitations, sweating, and anxiety. Women

VSM
may experience fewer typical symptoms than men, most commonly shortness of
breath, weakness, a feeling of indigestion, and fatigue.
-Classification:
There are two basic types based on pathology:
Transmural: associated with atherosclerosis involving a major coronary artery.
Transmural infarcts extend through the whole thickness of the heart muscle and are
usually a result of complete occlusion of the area's blood supply.
Subendocardial: involving a small area in the subendocardial wall of the left ventricle,
ventricular septum, or papillary muscles.
The other classification:
Type 1 – Spontaneous myocardial infarction related to ischaemia due to a primary
coronary event such as plaque erosion or rupture, fissuring. Type 2 – Myocardial
infarction secondary to ischaemia due to either increased oxygen demand or
decreased supply, e.g. coronary artery spasm, coronary embolism, anaemia,
arrhythmias, hypertension, or hypotension
Type 3 – Sudden unexpected cardiac death, including cardiac arrest, often with
symptoms suggestive of myocardial ischaemia, accompanied by new ST elevation or
evidence of fresh thrombus in a coronary artery by angiography or at autopsy, but
death occurring before blood samples could be obtained, or at a time before the
appearance of cardiac biomarkers in the blood
Type 4 – Associated with coronary angioplasty or stents
Type 5 – Myocardial infarction associated with CABG
-Pathophysiology:
The most common triggering event is the disruption of an atherosclerotic plaque in
an epicardial coronary artery, which leads to a clotting cascade, sometimes resulting
in total occlusion of the artery. Atherosclerosis is the gradual build-up of cholesterol
and fibrous tissue in plaques in the wall of arteries, typically over decades. Blood
stream column irregularities visible on angiography reflect artery lumen narrowing
because of decades of advancing atherosclerosis. Plaques can become unstable,
rupture, and additionally promote a thrombus (blood clot) that occludes the artery;
this can occur in minutes. When a severe enough plaque rupture occurs in the
coronary vasculature, it leads to myocardial infarction.
If impaired blood flow to the heart lasts long enough, it triggers a process called the
ischemic cascade; the heart cells in the territory of the occluded coronary artery die
and do not grow back. A collagen scar forms in its place. Recent studies indicate that
another form of cell death called apoptosis also plays a role in the process of tissue
damage after myocardial infarction. As a result, the patient's heart will be
permanently damaged. This myocardial scarring also puts the patient at risk for
potentially life-threatening arrhythmias, and may result in the formation of a
ventricular aneurysm that can rupture with catastrophic consequences.

VSM
48. Pericardial disease. Pericarditis, pericardial effusion & tamponade.
Pericarditis is a swelling and irritation of the pericardium, the thin sac-like membrane
that surrounds your heart. Pericarditis often causes chest pain and sometimes other
symptoms. Pericarditis is usually sudden and short-lived (acute). When symptoms
develop more gradually or persist, the pericarditis is considered chronic. The sharp
chest pain associated with pericarditis occurs when the inflamed or irritated two
layers of the pericardium rub against each other. Symptoms:
Sharp, piercing chest pain over the center or left side of chest
Shortness of breath when reclining
Low-grade fever
An overall sense of weakness, fatigue or feeling sick
Dry cough
Abdominal or leg swelling
Causes:
Pericarditis can develop shortly after a major heart attack, due to the irritation of the
underlying damaged heart muscle. In addition, a delayed form of pericarditis may
occur weeks after a heart attack or heart surgery because of antibody formation.
This delayed pericarditis is known as Dressler's syndrome.
Other causes of pericarditis include:
Systemic inflammatory disorders. These include lupus and rheumatoid arthritis.
Trauma. Injury to your heart or chest may occur as a result of a motor vehicle or
other accident.
Other health disorders. These may include kidney failure, AIDS, tuberculosis and
cancer.
Certain medications. Some medications can cause pericarditis, although this is
unusual.
Pathophysiology: The pericardium has two layers: visceral and parietal. The visceral
layer is closely apposed to the heart, whilst the fibrous parietal layer provides a more
rigid outer shell to the pericardial cavity. The normal volume of pericardial fluid is in
the region of 50 mL. Viral pericarditis is the most common type, but in many cases
either the diagnosis is never made or the viral infection is never identified. Another
common presentation is characteristic pain 3–14 days postmyocardial infarction or
postcardiac surgery, which tends to be self-limiting. Relapsing episodes are referred
to as Dressler's syndrome or postcardiotomy syndrome, and are thought to represent
an autoimmune process.

VSM
Pericardial effusion is an abnormal accumulation of fluid in the pericardial cavity. A
pericardial effusion with enough pressure to adversely affect heart function is called
cardiac tamponade. Pericardial effusion usually results from a disturbed equilibrium
between the production and reabsorption of pericardial fluid.
Normal levels of pericardial fluid are from 15 to 50 mL.
Types
It may be:
transudative (congestive heart failure, myxoedema, nephrotic syndrome),
exudative (tuberculosis, spread from empyema) haemorrhagic (trauma, rupture of
aneurysms, malignant effusion). malignant (due to fluid accumulation caused by
metastasis)
symptoms:
Chest pain or pressure are common symptoms. Larger effusions may cause cardiac
tamponade, a lifethreatening complication; signs of impending tamponade include
dyspnea, low blood pressure, and distant heart sounds.
Causes:
Pericarditis Viral infection Infection Inflammatory disorders, such as lupus
and post myocardial infarction pericarditis (Dressler's syndrome) Cancer that has
spread to the pericardium Trichinosis Kidney failure with excessive blood levels
of urea nitrogen Hypothyroidism The pericardium plays a pivotal role in cardiac
changes during inspiration. As the right atrium and ventricle fill during normal
inspiration, the pericardium, by limiting the ability of these chambers to dilate,
contributes to the bowing of the atrial and ventricular septums to the left. This
reduces left ventricular (LV) filling volumes, which lead to the drop in cardiac output.
As intrapericardial pressures rise, this effect becomes pronounced, eventually
leading to the finding of pulsus paradoxus, which heralds the development of
pericardial tamponade.
Cardiac tamponade is an acute type of pericardial effusion in which fluid
accumulates in the pericardium. Cardiac tamponade is pressure on the heart muscle
which occurs when the pericardial space fills up with fluid faster than the pericardial
sac can stretch. If the amount of fluid increases slowly the pericardial sac can
expand to contain a liter or more of fluid prior to tamponade occurring. If the fluid
occurs rapidly as little as 100 ml can cause tamponade.
Causes of increased pericardial effusion include hypothyroidism, physical trauma,
pericarditis, iatrogenic trauma and myocardial rupture.
One of the most common settings for cardiac tamponade is in the first 24 to 48 hours
after heart surgery. After heart surgery, chest tubes are placed to drain blood. These
chest tubes, however, are prone to clot formation. When a chest tube becomes
occluded or clogged, the blood that should be drained can accumulate around the
heart, leading to tamponade.

VSM
Pathophysiology
The outer layer of the heart is made of fibrous tissue which does not easily stretch,
and so once fluid begins to enter the pericardial space, pressure starts to increase.
If fluid continues to accumulate, then with each successive diastolic period, less and
less blood enters the ventricles, as the increasing pressure presses on the heart and
forces the septum to bend into the left ventricle, leading to decreased stroke volume.
This causes obstructive shock to develop, and if left untreated then cardiac arrest
may occur.
49. Primary hypertension.
Essential hypertension remains a major modifiable risk factor for cardiovascular
disease (CVD) despite important advances in our understanding of its
pathophysiology and the availability of effective treatment strategies. High blood
pressure (BP) increases the risk of CVD for millions of people worldwide, and there
is evidence that the problem is only getting worse. In the past decade, age-adjusted
rates of stroke incidence have risen, and the slope of the age-adjusted rate of
decline in coronary disease has leveled off. The incidence of end-stage renal
disease and the prevalence of heart failure have also increased. A major contributor
to these trends is inadequate control of BP in the hypertensive population. This
review of current concepts regarding the definition, etiology, and treatment of
essential hypertension is intended to aid the clinician in identifying those individuals
at high risk who need to undergo evaluation and treatment, as well as in selecting
optimal treatment strategies for hypertensive patients with comorbid conditions
and/or target organ damage. The part of the review that deals with the genetic basis
of hypertension and the gene/environment interaction that may lead to elevated BP
is still a work in progress. Information gained from the Human Genome Project and
from ongoing studies of the genetic basis of hypertension both in animal models and
human populations may revolutionize the treatment of hypertension by replacing
current empirical therapy with more effective, targeted treatments based on the
genotype of the patient. Concepts introduced in this review form the basis for such
"pharmacogenomic" approaches to antihypertensive therapy.

Definition of Essential or Primary Hypertension


BP is a quantitative trait that is highly variable1 ; in population studies, BP has a
normal distribution that is slightly skewed to the right. There is a strong positive and
continuous correlation between BP and the risk of CVD (stroke, myocardial
infarction, heart failure), renal disease, and mortality, even in the normotensive
range. This correlation is more robust with systolic than with diastolic BP.2 There is
no specific level of BP where cardiovascular and renal complications start to occur;
thus the definition of hypertension is arbitrary but needed for practical reasons in
patient assessment and treatment. The Sixth Report of the Joint National Committee
on Prevention, Detection, Evaluation, and Treatment of High Blood Pressure (JNC
VI) defined and classified hypertension in adults, The diagnosis of hypertension is
made when the average of 2 or more diastolic BP measurements on at least 2

VSM
subsequent visits is ≥90 mm Hg or when the average of multiple systolic BP
readings on 2 or more subsequent visits is consistently ≥140 mm Hg. Isolated
systolic hypertension is defined as systolic BP ≥140 mm Hg and diastolic BP <90
mm Hg. Individuals with high normal BP tend to maintain pressures that are above
average for the general population and are at greater risk for development of definite
hypertension and cardiovascular events than the general population.
Known Etiological Factors in Essential Hypertension
Although it has frequently been indicated that the causes of essential hypertension
are not known, this is only partially true because we have little information on genetic
variations or genes that are overexpressed or under expressed as well as the
intermediary phenotypes that they regulate to cause high BP.4 A number of factors
increase BP, including (1) obesity, (2) insulin resistance, (3) high alcohol intake, (4)
high salt intake (in salt-sensitive patients), (5) aging and perhaps (6) sedentary
lifestyle, (7) stress, (8) low potassium intake, and (9) low calcium intake.5 6
Furthermore, many of these factors are additive, such as obesity and alcohol intake.
Inherited BP
The identification of variant (allelic) genes that contribute to the development of
hypertension is complicated by the fact that the 2 phenotypes that determine BP,
cardiac output and total peripheral resistance, are controlled by intermediary
phenotypes, including the autonomic nervous system, vasopressor/vasodepressor
hormones, the structure of the cardiovascular system, body fluid volume and renal
function, and many others. Furthermore, these intermediary phenotypes are also
controlled by complex mechanisms including BP itself.12 Thus there are many
genes that could participate in the development of hypertension. Glucocorticoid-
Remediable Aldosteronism This is an autosomal dominant form of monogenic
hypertension in which aldosterone secretion is regulated by adrenocorticotropic
hormone. Glucocorticoid treatment causes BP to decrease and gives the syndrome
its name. The genetic mutation that causes GRA has been identified by Lifton14 as a
chimeric gene fusing nucleotide sequences of the promoter-regulatory region of 11ß-
hydroxylase (controlled by adrenocorticotropic hormone) and the structural portion of
the aldosterone synthase gene. The chimeric gene results from a meiotic mismatch
and unequal crossing over. The patients are usually thought to have primary
aldosteronism because they exhibit volume expansion, metabolic alkalosis with
hypokalemia, low plasma renin, and high aldosterone
Liddle’s Syndrome This is an autosomal dominant form of monogenic hypertension
that results from mutations in the amiloride-sensitive epithelial sodium channel,
leading to increased channel activity.17 The mutations reported to date result in the
elimination of 45 to 75 amino acids from the cytoplasmic carboxyl terminus of ß- or γ-
subunits of the channel; thus Liddle’s syndrome is genetically heterogeneous. It is
characterized by the early onset of hypertension with hypokalemia and suppression
of both plasma renin activity and aldosterone, the latter differentiating this syndrome
from primary aldosteronism. Both the hypertension and the hypokalemia vary in
severity, raising the possibility that some patients classified as having salt-sensitive
essential hypertension actually have Liddle’s syndrome.

VSM
Autosomal Dominant Hypertension With Brachydactyly In this monogenic syndrome,
hypertension and brachydactyly are always inherited together (100%
cosegregation).4 Affected persons are shorter than nonaffected relatives. The gene
for hypertension has been mapped to the short arm of chromosome 12 (12p) in a
large Turkish kindred. Two other families with this syndrome have been reported, 1
in Canada and 1 in the United States. In addition, the study of a Japanese child with
hypertension and type E brachydactyly has allowed the area on 12p containing the
gene mutation to be pinpointed further, although the gene responsible for this
syndrome has not yet been cloned.
Essential or Primary Hypertension The genetic alterations responsible for inherited
"essential" hypertension remain largely unknown.4 Results from family studies
suggest several possible intermediary phenotypes (genetic traits) that may be
related to inherited high BP, such as high sodium-lithium countertransport, low
urinary kallikrein excretion, high fasting plasma insulin concentrations, high-density
LDL subfractions, fat pattern index, and body mass index (BMI).12 Jeunemaitre et
al20 21 first reported a polymorphism in the angiotensinogen gene linked with
essential hypertension in hypertensive siblings from Utah and France. This
polymorphism consists of a substitution of thymidine for cytosine in nucleotide 704,
which causes substitution of methionine for threonine at position 235 (M235T) and is
associated with increased concentrations of plasma angiotensinogen. This variant
appears to be in tight linkage disequilibrium with a promoter mutation in which
adenine replaces guanine (-6A) upstream from the initiation side of transcription.22
Tests of promoter activity suggest that this nucleotide substitution increases the rate
of gene transcription, which may explain the higher angiotensinogen concentration
found in subjects with the variant M235T.22 Many studies have been published on
various racial groups with regard to the association between allelic variations in
angiotensinogen and hypertension.23 However, these variations explain only a small
part of the BP variation (≈6%). Furthermore, plasma angiotensinogen
concentrations, though higher in patients with the polymorphism, clearly overlap with
normotensive patients.
Obesity and Insulin Resistance Obesity, and especially abdominal obesity, is the
main hyporeninaemic factor. It was estimated in the Framingham study that each
10% weight gain is associated with a 6.5 mm Hg increase in systolic BP.24 Obesity
is also the cause of insulin resistance, adult-onset diabetes mellitus, left ventricular
hypertrophy, hyperlipidemia, and atherosclerotic disease. Thus, obesity is an
important cardiovascular health problem; its incidence and prevalence are rising in
most industrialized societies and in the United States has reached epidemic
proportions. The relationship between BP and body fat is not restricted to the
morbidly obese but is continuous throughout the entire range of body weight. A direct
association between hypertension and BMI (weight in kilograms divided by the
square height in meters) has been observed in cross-sectional and longitudinal
population studies from early childhood to old age.25 A BMI of <25 is considered
normal or healthy, whereas a BMI of 26 to 28 (as compared with BMI <23) increases
the risk of high BP by 180% and the risk of insulin resistance by >1000%. Thus,
insulin resistance is present in many patients with obesity and hypertension.

VSM
50. Secondary hypertension.
Secondary hypertension (secondary high blood pressure) is high blood pressure
that's caused by another medical condition. Secondary hypertension differs from the
usual type of high blood pressure (essential hypertension), which is often referred to
simply as high blood pressure. Essential hypertension, also known as primary
hypertension, has no clear cause and is thought to be linked to genetics, poor diet,
lack of exercise and obesity.
Diabetes complications (diabetic nephropathy). Diabetes can damage your kidneys'
filtering system, which can lead to high blood pressure. Polycystic kidney disease. In
this inherited condition, cysts in your kidneys prevent the kidneys from working
normally, and can raise blood pressure.
Glomerular disease. Your kidneys filter waste and sodium using microscopic-sized
filters called glomeruli that can sometimes become swollen. If the swollen glomeruli
can't work normally, you may develop high blood pressure.
Renovascular hypertension. This is a type of secondary hypertension caused by
narrowing (stenosis) of one or both arteries leading to your kidneys. Renovascular
hypertension can cause severe hypertension and irreversible kidney damage. It's
often caused by the same type of fatty plaques that can damage your coronary
arteries (atherosclerosis) or a condition in which the muscle and fibrous tissues of
the renal artery wall thicken and harden into rings (fibromuscular dysplasia).
Cushing's syndrome. In this condition, corticosteroid medications, a pituitary tumor or
other factors cause the adrenal glands to produce too much of the hormone cortisol.
This raises blood pressure.
Aldosteronism. In this condition, a tumor in the adrenal gland, increased growth of
normal cells in the adrenal gland or other factors cause the adrenal glands to release
an excessive amount of the hormone aldosterone. This makes your kidneys retain
salt and water and lose too much potassium, which raises blood pressure.
Pheochromocytoma. This rare tumor, usually found in an adrenal gland, increases
production of the hormones adrenaline and noradrenaline, which can lead to long-
term high blood pressure or shortterm spikes in blood pressure.
Thyroid problems. When the thyroid gland doesn't produce enough thyroid hormone
(hypothyroidism) or produces too much thyroid hormone (hyperthyroidism), high
blood pressure can result.
Hyperparathyroidism. The parathyroid glands regulate levels of calcium and
phosphorus in your body. If the glands secrete too much parathyroid hormone, the
amount of calcium in your blood rises — which triggers a rise in blood pressure.
Coarctation of the aorta. With this defect you're born with, the body's main artery
(aorta) is narrowed (coarctation). This forces the heart to pump harder to get blood
through the aorta and to the rest of your body. This, in turn, raises blood pressure —
particularly in your arms.

VSM
Sleep apnea. In this condition, often marked by severe snoring, breathing repeatedly
stops and starts during sleep, meaning you don't get enough oxygen. Not getting
enough oxygen may damage the lining of the blood vessel walls, which may make
your blood vessels less effective in regulating your blood pressure. In addition, sleep
apnea causes part of the nervous system to be overactive and release certain
chemicals that increase blood pressure.
Obesity. As you gain weight, the amount of blood circulating through your body
increases. This puts added pressure on your artery walls, increasing your blood
pressure. In addition, excess weight often is associated with an increase in heart rate
and a reduction in the capacity of your blood vessels to transport blood. All of these
factors can increase blood pressure.
Pregnancy. Pregnancy can make existing high blood pressure worse, or may cause
high blood pressure to develop (pregnancy-induced hypertension or preeclampsia).
Medications and supplements. Various prescription medications — from pain
relievers to antidepressants and drugs used after organ transplants — can cause or
aggravate high blood pressure in some people. Birth control pills, decongestants and
certain herbal supplements, including ginseng.
51. Circulatory failure. Shock.
Circulatory failure may result from decreased myocardial contractility, arrhythmias
that allow insufficient time for diastolic filling or impair atrioventricular synchrony,
circulatory stresses such as increased afterload or hypovolaemia, valvular
dysfunction, tamponade, or a variety of less common insults. Regardless of the
aetiology, circulatory fail-ure elicits characteristic compensatory haemodynamic
adjustments, mediated in large part by activation of the sympathetic nervous system.
The cause of shock is usually reduced cardiac output, with the following possible
causes:
In hypovolemia (hypovolemic shock) the central venous pressure is reduced, and
thus the venous return decreased. As a result stroke volume falls. The cause of the
hypovolemia can be bleeding (hemorrhagic shock) or some other loss of fluid to the
outside.
Cardiogenic shock. Primary or secondary heart failure can be caused by acute
myocardial infarction, acute decompensating heart failure, malignant arrhythmias,
cardiomyopathy, acute valvar regurgitation, obstruction of the large vessels (e.g.,
pulmonary embolism) or by impairment of cardiac filling (mitral stenosis, pericardial
tamponade, constrictive pericarditis). In these conditions, in contrast to hypovolemic
shock, the central venous pressure is raised.
Hormonal causes of shock include adrenal hypofunction (Addisonian crisis),
diabetic coma (.), hypoglycaemic shock (insulin over dosage, insulinoma;),
hypothyroid or hyperthyroid coma, and coma in hypoparathyroidism or
hyperparathyroidism. Reduced cardiac output may also be caused by peripheral
vascular distension (no pallor) with venous pooling of blood (decreased venous

VSM
return), as may happen in anaphylactic shock (food or drug allergy), in which
vasoactive substances are released (histamine.
In septic–toxic shock the cardiac output is at first raised by the action of toxins of,
usually Gram-negative bacteria (tachycardia and reduced total peripheral
resistance). The initially normal blood pressure then falls, respiratory failure occurs,
and finally a late stage develops with reduced cardiac output and high total
peripheral resistance.
Neurogenic shock is rare, but it may occur when, for example, brain stem or spinal
cord trauma or intoxication (barbiturates, narcotics) disturb autonomic nervous
system regulation of the heart and circulation and the venous return is markedly
reduced.
Symptoms (left). Hypovolemic and haemorrhagic shock is often associated with a
reduced blood pressure (narrow pulse amplitude), increased heart rate, pallor with
cold sweat (not in shock that is due to vascular distension), diminished urine output.
The resulting (blood) volume deficit can be estimated by means of the shock index
(heart rate per minute/systolic blood pressure in mmHg):
• 0.5 = normal or blood loss < 10%;
• 1.0 = blood loss < 20–30% (incipient shock);
• 1.5 = blood loss > 30–50% (manifest shock).
52. Hypotension.
Hypotension is abnormally low blood pressure, especially in the arteries of the
systemic circulation. It is best understood as a physiological state, rather than a
disease. It is often associated with shock, though not necessarily indicative of it.
Hypotension is the opposite of hypertension, which is high blood pressure. Blood
pressure is the force of blood pushing against the walls of the arteries as the heart
pumps out blood. If it is lower than normal, then it is called low blood pressure or
hypotension. Hypotension is generally considered as systolic blood pressure less
than 90 millimeters of mercury (mm Hg) or diastolic less than 60 mm Hg.
Symptoms Chest pain
Shortness of breath
Irregular heartbeat
Fever higher than 38.3°C (101°F)
Headache
Stiff neck
Severe upper back pain
Cough with phlegm
Prolonged diarrhea or vomiting

VSM
Dyspepsia
Dysuria
Blood pressure is continuously regulated by the autonomic nervous system, using an
elaborate network of receptors, nerves, and hormones to balance the effects of the
sympathetic nervous system, which tends to raise blood pressure, and the
parasympathetic nervous system, which lowers it. The vast and rapid compensation
abilities of the autonomic nervous system allow normal individuals to maintain an
acceptable blood pressure over a wide range of activities and in many disease
states.
Low blood pressure causes can be due to low blood volume, hormonal changes,
widening of blood vessels, medicine side effects, anemia, and heart and endocrine
problems.
Reduced blood volume, hypovolemia, is the most common mechanism producing
hypotension. This can result from hemorrhage, or blood loss; insufficient fluid intake,
as in starvation; or excessive fluid losses from diarrhea or vomiting. Hypovolemia is
often induced by excessive use of diuretics. Other medications can produce
hypotension by different mechanisms.
53. Overview of gastrointestinal disorders.
Constipation is the difficult passage of stools (bowel movements) or the infrequent
(less than three times a week) or incomplete passage of stools. Constipation is
usually caused by inadequate "roughage" or fibre in the diet, or a disruption of the
regular routine or diet. Constipation causes a person to strain during a bowel
movement. It might include small, hard stools, and sometimes causes anal problems
such as fissures and haemorrhoids. Constipation is rarely the sign of a more serious
medical condition. Treatment of constipation includes increasing the amount of fiber
you eat, exercising regularly, and moving your bowels when you have the urge.
Irritable bowel syndrome (IBS)
Irritable bowel syndrome (also called spastic colon, irritable colon, or nervous
stomach) is a condition in which the colon muscle contracts more readily than in
people without IBS. Many factors can trigger IBS including certain foods, medicines,
and emotional stress. Symptoms of IBS include abdominal pain and cramps, excess
gas, bloating, and a change in bowel habits such as harder, looser, or more urgent
stools than normal. Often people with IBS have alternating constipation and
diarrhoea. Treatment includes avoiding caffeine, increasing fiber in the diet,
monitoring which foods trigger IBS.
Structural disorders
Structural disorders are those in which the bowel looks abnormal and doesn’t work
properly. Sometimes, the structural abnormality needs to be removed surgically.
Anal disorders Haemorrhoids Haemorrhoids are swollen blood vessels that line the
anal opening caused by chronic excess pressure from straining during a bowel

VSM
movement, persistent diarrhoea, or pregnancy. There are two types of
haemorrhoids: internal and external.
Internal haemorrhoids
Internal haemorrhoids are normal structures cushioning the lower rectum and
protecting it from damage by stool. When they fall into the anus as a result of
straining, they become irritated and start to bleed. Ultimately, internal haemorrhoids
can fall down enough to prolapse (sink or protrude) out of the anus.
External haemorrhoids
External haemorrhoids are veins that lie just under the skin on the outside of the
anus. Sometimes, after straining, the external hemorrhoidal veins burst and a blood
clot forms under the skin. This very painful condition is called a pile. Treatment
includes removing the clot and vein under local anaesthesia in the doctor’s office.
Anal fissures Anal fissures are splits or cracks in the lining of the anal opening. The
most common cause of an anal fissure is the passage of very hard or watery stools.
The crack in the anal lining exposes the underlying muscles that control the passage
of stool through the anus and out of the body. An anal fissure is one of the most
painful problems because the exposed muscles become irritated from exposure to
stool or air, and leads to intense burning pain, bleeding, or spasm after bowel
movements.
Diverticular disease
Diverticulosis is the presence of small outpouchings (diverticula) in the muscular wall
of the large intestine that form in weakened areas of the bowel. They usually occur in
the sigmoid colon, the high-pressure area of the lower large intestine. Diverticular
disease is very common and occurs in 10 percent of people over age 40 and in 50
percent of people over age 60 in Western cultures. It is often caused by too little
roughage (fiber) in the diet.
Colon polyps and cancer
Each year 130,000 Americans are diagnosed with colorectal cancer, the second
most common form of cancer in the United States. Fortunately, with advances in
early detection and treatment, colorectal cancer is one of the most curable forms of
the disease. By using a variety of screening tests, it is possible to prevent, detect,
and treat the disease long before symptoms appear. There are several types of
colitis, conditions that cause an inflammation of the bowel. These include:

Infectious colitis Ulcerative colitis (cause not known) Crohn's disease (cause not
known) Ischemic colitis (caused by not enough blood going to the colon) Radiation
colitis (after radiotherapy).

VSM
54. Acid-peptic disease. Acute erosive gastritis & chronic atrophic gastritis.
Excessive secretion of this acid and pepsin or a weakened stomach mucosal
defense is responsible for damage to the delicate mucosa and the lining of the
stomach, esophagus and duodenum resulting in ulceration which is known as “Acid
Peptic Disease”.Physiologically, a certain amount of acid is secreted by the gastric
cells lining the stomach as a natural mechanism which serves to activate the
digestive enzymes and help in the digestion and assimilation of important proteins so
that they can be easily absorbed by the body. Acid peptic disease” is a collective
term used to include many conditions such as gastro-oesophageal reflux disease
(GERD), gastritis, gastric ulcer, duodenal ulcer, oesophageal ulcer, Zollinger Ellison
Syndrome (ZES) and Meckel’s diverticular ulcer. Symptoms of peptic ulcers include
abdominal pain, nausea, water brash, vomiting, loss of appetite and weight loss.
Complications include bleeding, perforation, obstruction in the digestive tract and
sometimes cancer. Peptic ulcer is diagnosed using blood and stool tests, breath
tests, and endoscopy and barium radiography. The patient is treated with drugs that
reduce acidity and sometimes in addition with certain antibiotics. Diagnosis; An
esophagogastroduodenoscopy (EGD), a form of endoscopy, also known as a
gastroscopy, is carried out on patients in whom a peptic ulcer is suspected. By direct
visual identification, the location and severity of an ulcer can be described.
Moreover, if no ulcer is present, EGD can often provide an alternative diagnosis.
Urea breath test (non-invasive and does not require EGD); Direct culture from an
EGD biopsy specimen; this is difficult to do, and can be expensive. Most labs are not
set up to perform H. pylori cultures; Direct detection of urease activity in a biopsy
specimen by rapid urease test; Measurement of antibody levels in blood. An
esophagogastroduodenoscopy (EGD), a form of endoscopy, also known as a
gastroscopy, is carried out on patients in whom a peptic ulcer is suspected. By direct
visual identification, the location and severity of an ulcer can be described.
Moreover, if no ulcer is present, EGD can often provide an alternative diagnosis.
Gastritis; Gastritis is an inflammation of the lining of the stomach, and has many
possible causes. The main acute causes are excessive alcohol consumption or
prolonged use of nonsteroidal anti-inflammatory drugs (also known as NSAIDs) such
as aspirin or ibuprofen. Sometimes gastritis develops after major surgery, traumatic
injury, burns, or severe infections.
Many people with gastritis experience no symptoms at all. However, upper central
abdominal pain is the most common symptom; the pain may be dull, vague, burning,
aching, gnawing, sore, or sharp. Nausea, Vomiting (if present, may be clear, green
or yellow, blood-streaked, or completely bloody, depending on the severity of the
stomach inflammation), Belching (if present, usually does not relieve the pain much)
,Bloating.
Erosive and haemorrhagic gastritis can have many causes, for example: – intake of
nonsteroidal anti-inflammatory drugs (NSAIDs), whose local and systemic mucosa-
damaging effect is described in greater detail on p.146;– ischemia (e.g., vasculitis or
while running a marathon)– stress (multi-organ failure, burns, surgery, central

VSM
nervous system trauma), in which the gastritis is probably in part caused by
ischemia;
– alcohol abuse, corrosive chemicals;
– trauma (gastroscope, swallowed foreign body, retching, vomiting, etc.);
– radiation trauma.
This type of gastritis can quickly produce an acute ulcer (e.g., through stress or
NSAIDs), with the risk of massive gastric bleeding or perforation of the stomach wall
Nonerosive, chronic active gastritis is usually restricted to the antrum.
It has become increasingly clear in the last decade that its determining cause is a
bacterial colonization of the antrum with Helicobacter pylori, which can be effectively
treated with antibiotics.
Helicobacter colonization not only diminishes mucosal protection, but can also
stimulate antral gastrin liberation and thus gastric juice secretion in the fundus, a
constellation that favours the development of chronic ulcer. A fourth type, reactive
gastritis, occurs in the surroundings of erosive gastritis (see above), of ulcers or of
operative wounds. The latter may partly be caused after operations on the antrum or
pylorus by enter gastric reflux (reflux gastritis), resulting in pancreatic and intestinal
enzymes and bile salts attacking the gastric mucosa. On the other hand, the alkaline
milieu of the intestinal juice counteracts gastrin release and is also a hostile medium
for Helicobacter pylori.
55. Acid-peptic disease. Gastric ulcer.
Excessive secretion of this acid and pepsin or a weakened stomach mucosal
defence is responsible for damage to the delicate mucosa and the lining of the
stomach, oesophagus and duodenum resulting in ulceration which is known as “Acid
Peptic Disease”. Physiologically, a certain amount of acid is secreted by the gastric
cells lining the stomach as a natural mechanism which serves to activate the
digestive enzymes and help in the digestion and assimilation of important proteins so
that they can be easily absorbed by the body. Acid peptic disease” is a collective
term used to include many conditions such as gastro-oesophageal reflux disease
(GERD), gastritis, gastric ulcer, duodenal ulcer, oesophageal ulcer, Zollinger Ellison
Syndrome (ZES) and Meckel’s diverticular ulcer.
Symptoms of peptic ulcers include abdominal pain, nausea, water brash, vomiting,
loss of appetite and weight loss. Complications include bleeding, perforation,
obstruction in the digestive tract and sometimes cancer. Peptic ulcer is diagnosed
using blood and stool tests, breath tests, and endoscopy and barium radiography.
The patient is treated with drugs that reduce acidity and sometimes in addition with
certain antibiotics. Diagnosis; An esophagogastroduodenoscopy (EGD), a form of
endoscopy, also known as a gastroscopy, is carried out on patients in whom a peptic
ulcer is suspected. By direct visual identification, the location and severity of an ulcer
can be described. Moreover, if no ulcer is present, EGD can often provide an
alternative diagnosis. Urea breath test (non-invasive and does not require EGD);
Direct culture from an EGD biopsy specimen; this is difficult to do, and can be

VSM
expensive. Most labs are not set up to perform H. pylori cultures;Direct detection of
urease activity in a biopsy specimen by rapid urease test; Measurement of antibody
levels in blood
An esophagogastroduodenoscopy (EGD), a form of endoscopy, also known as a
gastroscopy, is carried out on patients in whom a peptic ulcer is suspected. By direct
visual identification, the location and severity of an ulcer can be described.
Moreover, if no ulcer is present, EGD can often provide an alternative diagnosis.
Gastric Ulcer; Gastric ulcers are most often localized on the lesser curvature of the
stomach. The ulcer is a round to oval parietal defect ("hole"), 2 to 4 cm diameter,
with a smooth base and perpendicular borders. These borders are not elevated or
irregular in the acute form of peptic ulcer, regular but with elevated borders and
inflammatory surrounding in the chronic form. In the ulcerative form of gastric cancer,
the borders are irregular. Surrounding mucosa may present radial folds, because of
the parietal scarring. A gastric peptic ulcer is a mucosal defect which penetrates the
muscular is mucosae and muscular is propria, produced by acid-pepsin aggression.
Ulcer margins are perpendicular and present chronic gastritis. During the active
phase, the base of the ulcer shows 4 zones: inflammatory exudate, fibrinoid
necrosis, granulation tissue and fibrous tissue. The fibrous base of the ulcer may
contain vessels with thickened wall or with thrombosis.
56. Acid-peptic disease. Duodenal ulcer.
Excessive secretion of this acid and pepsin or a weakened stomach mucosal
defence is responsible for damage to the delicate mucosa and the lining of the
stomach, oesophagus and duodenum resulting in ulceration which is known as “Acid
Peptic Disease”. Physiologically, a certain amount of acid is secreted by the gastric
cells lining the stomach as a natural mechanism which serves to activate the
digestive enzymes and help in the digestion and assimilation of important proteins so
that they can be easily absorbed by the body. Acid peptic disease” is a collective
term used to include many conditions such as gastro-oesophageal reflux disease
(GERD), gastritis, gastric ulcer, duodenal ulcer, oesophageal ulcer, Zollinger Ellison
Syndrome (ZES) and Meckel’s diverticular ulcer. Symptoms of peptic ulcers include
abdominal pain, nausea, water brash, vomiting, loss of appetite and weight loss.
Complications include bleeding, perforation, obstruction in the digestive tract and
sometimes cancer. Peptic ulcer is diagnosed using blood and stool tests, breath
tests, and endoscopy and barium radiography. The patient is treated with drugs that
reduce acidity and sometimes in addition with certain antibiotics.
Diagnosis; An esophagogastroduodenoscopy (EGD), a form of endoscopy, also
known as a gastroscopy, is carried out on patients in whom a peptic ulcer is
suspected. By direct visual identification, the location and severity of an ulcer can be
described. Moreover, if no ulcer is present, EGD can often provide an alternative
diagnosis. Urea breath test (non-invasive and does not require EGD); Direct culture
from an EGD biopsy specimen; this is difficult to do, and can be expensive. Most
labs are not set up to perform H. pylori cultures;
Direct detection of urease activity in a biopsy specimen by rapid urease test;

VSM
Measurement of antibody levels in blood An esophagogastroduodenoscopy (EGD), a
form of endoscopy, also known as a gastroscopy, is carried out on patients in whom
a peptic ulcer is suspected. By direct visual identification, the location and severity of
an ulcer can be described. Moreover, if no ulcer is present, EGD can often provide
an alternative diagnosis. Duodenum Ulcers; A duodenal ulcer is a type of peptic
ulcer that occurs in the duodenum, the beginning of the small intestine. Peptic ulcers
are eroded areas in the lining of stomach and duodenum, which result in abdominal
pain, possible bleeding, and other gastrointestinal symptoms. The most common
cause of duodenal ulcer is a stomach infection associated with the Helicobacter
pylori (H pylori) bacteria. Symptoms; abdominal pain, classically epigastric with
severity relating to mealtimes, after around three hours of taking a meal (duodenal
ulcers are classically relieved by food, while gastric ulcers are exacerbated by it);
bloating and abdominal fullness; water brash (rush of saliva after an episode of
regurgitation to dilute the acid in oesophagus - although this is more associated with
gastroesophageal reflux disease); nausea, and copious vomiting; loss of appetite
and weight loss.
Cause; Another major cause is the use of NSAIDs. The gastric mucosa protects
itself from gastric acid with a layer of mucus, the secretion of which is stimulated by
certain prostaglandins. NSAIDs block the function of cyclooxygenase 1 (cox-1),
which is essential to produce these prostaglandins. COX-2 selective anti-
inflammatories (such as celecoxib or the since withdrawn forecoxa) preferentially
inhibit cox-2, which is less essential in the gastric mucosa, and roughly halve the risk
of NSAID-related gastric ulceration. As the prevalence of H. pylori-caused ulceration
declines in the Western world due to increased medical treatment, a greater
proportion of ulcers will be due to increasing NSAID use among individuals with pain
syndromes as well as the growth of aging populations that develop arthritis.
57. Disorders of the gallbladder. Cholelithiasis.
Cholelithiasis - Gallstones Solid crystalline precipitates in the BILIARY TRACT,
usually formed in the GALLBLADDER. Gallstones, derived from the BILE, consist
mainly of calcium, cholesterol, or bilirubin.
Biliary Dyskinesia;Acalculous cholecystopathy which means disease or condition of
the gallbladder without the presence of gallstones. You might also call it functional
gallbladder disorder or impaired gallbladder emptying. Some causes may be chronic
inflammation, stress, a problem with the smooth muscles of the gallbladder or the
muscle of the Sphincter of Oddi being too tight.
Inflammation of the gallbladder. Acute cholecystitis is nearly always due to
gallstones but may be due to infection (bacterial). It can also be due to chemical
irritation. Chronic cholecystitis occurs with or without stones (acalculous cholecystitis
is without). If there are no stones present the medical treatment used is often
antispasmodics and/or laxatives.
Cholangitis; Inflammation of the bile duct itself. Chole = bile and angi = duct. Acute
cholangitis is usually caused by a bacterial infection resulting from stagnation of the

VSM
bile in the duct. Choledocholithiasis, a gallstone that gets stuck or lodged in the bile
duct can create an obstruction that results in an infection.
Primary Sclerosing Cholangitis; PSC is a disease where the bile ducts of the liver
harden, obstructing the flow of bile. It is characterized by inflammation, breaking
down of and eventual hardening or fibrosis of the bile ducts within the liver and
outside the liver both (intrahepatic and extrahepatic bile ducts.) It is most likely an
autoimmune liver disease.
Gallbladder Polyps; Gallbladder polyps are growths that protrude from the lining of
the gallbladder. They're usually innocuous and rarely cancerous (malignant). 95%
are non-cancerous. 10% are the result of inflammation.2. Most polyps are the result
of cholesterol deposits. Cholesterol gallstones develop when bile contains too much
cholesterol and not enough bile salts. Besides a high concentration of cholesterol,
two other factors are important in causing gallstones.
The first is how often and how well the gallbladder contracts; incomplete and
infrequent emptying of the gallbladder may cause the bile to become
overconcentrated and contribute to gallstone formation. The second factor is the
presence of proteins in the liver and bile that either promote or inhibit cholesterol
crystallization into gallstones. In addition, increased levels of the hormone estrogen
as a result of pregnancy, hormone therapy, or the use of combined (estrogen-
containing) forms of hormonal contraception, may increase cholesterol levels in bile
and also decrease gallbladder movement, resulting in gallstone formation. A
gallstone is a crystalline concretion formed within the gallbladder by accretion of bile
components. These calculi are formed in the gallbladder, but may pass distally into
other parts of the biliary tract such as the cystic duct, common bile duct, pancreatic
duct, or theampulla of Vater.Reduced bile salt secretion. This isdue to either a
decrease in the bile salt pool, as in Crohn’s disease or after gut resection, or a
prolonged sequestration of bile salts in the gallbladder, as in fasting (possibly even if
only overnight) or parenteral nutrition. The latter decreases the enterohepatic
circulation of bile salts so that their secretion into the bile is reduced. The causes of
an increased concentration of unconjugated bilirubin are :Increased liberation of
hemoglobin, for example, in hemolytic anaemia, in which there is so much bilirubin
that the glucuronidase- mediated process of conjugation in the liver does not meet
demand.
58. Disorders of the small intestine. Diarrhea.
Problems with the small intestine can include:Bleeding,Celiac disease,Crohn's
disease,Infections,Intestinal cancer,Intestinal obstruction,Irritable bowel
syndrome,Ulcers, such as peptic ulcer. Coeliac disease ( /ˈsiːli.æk/; spelled celiac
disease in North America[1] and often celiac sprue) is an autoimmune disorder of the
small intestine that occurs in genetically predisposed people of all ages from middle
infancy onward. Symptoms include chronic diarrhoea, failure to thrive (in children),
and fatigue, but these may be absent, and symptoms in other organ systems have
been described. The diarrhoea that is characteristic of coeliac disease is (chronic)
pale, voluminous and malodorous. Abdominal pain and cramping, bloatedness with
abdominal distension (thought to be due to fermentative production of bowel gas)

VSM
and mouth ulcers may be present. As the bowel becomes more damaged, a degree
of lactose intolerance may develop. The changes in the bowel make it less able to
absorb nutrients, minerals and the fat-soluble vitamins A, D, E, and K. The inability to
absorb carbohydrates and fats may cause weight loss (or failure to thrive/stunted
growth in children) and fatigue or lack of energy. Anaemia may develop in several
ways: iron malabsorption may cause iron deficiency anaemia, and folic acid and
vitamin B12 malabsorption may give rise to megaloblastic anaemia. Calcium and
vitamin D malabsorption. Crohn's disease, also known as regional enteritis, is a type
of inflammatory bowel disease that may affect any part of the gastrointestinal tract
from mouth to anus, causing a wide variety of symptoms. It primarily causes
abdominal pain, diarrhoea.
During a colonoscopy, biopsies of the colon are often taken to confirm the diagnosis.
Certain characteristic features of the pathology seen point toward Crohn's disease; it
shows a transmural pattern of inflammation, meaning the inflammation may span the
entire depth of the intestinal wall. Ulceration is an outcome seen in highly active
disease. There is usually an abrupt transition between unaffected tissue and the
ulcer - a characteristic sign known as skip lesions.
A peptic ulcer, also known as PUD or peptic ulcer disease, is the most common ulcer
of an area of the gastrointestinal tract that is usually acidic and thus extremely
painful. It is defined as mucosal erosions equal to or greater than 0.5 cm. As many
as 70–90% of such ulcers are associated with Helicobacter pylori.
Four times as many peptic ulcers arise in the duodenum—the first part of the small
intestine, just after the stomach—as in the stomach itself. About 4% of gastric ulcers
are caused by a malignant tumour, so multiple biopsies are needed to exclude
cancer.
Osmotic diarrhoea results from the intake of a large number of substances that are
poorly absorbable even normally, or in malabsorption. Among the first group are
sorbitol (in “sugar-free” medications and sweets or certain fruits), fructose (in
lemonades, diverse fruits, honey), magnesium salts (antacids, laxatives) as well as
poorly absorbed anions such as sulphate, phosphate, or citrate.
Secretory diarrhoea (in the narrow sense) occurs when Cl– secretion of the small
intestinal mucosa is activated. Within the mucosal cells Cl– is secondarily actively
enriched by a basolateral Na+-K+-2 Cl– symport carrier and is secreted via luminal
Cl– channels. These open more frequently when the intracellular concentration of
cAMP rises. cAMP is formed in greater amounts in the presence of, for example,
certain laxatives and bacterial toxins.
59. Diverticular disease. Constipation.
Diverticulosis, also known as "diverticular disease", is the condition of having
diverticula in the colon, which are outpocketings of the colonic mucosa and
submucosa through weaknesses of muscle layers in the colon wall. These are more
common in the sigmoid colon, which is a common place for increased pressure. This
is uncommon before the age of 40, and increases in incidence after that age.
Cramps and tenderness in the affected areas. Uncomplicated diverticulosis

VSM
Some patients with diverticulosis complain of symptoms such as cramping, bloating,
flatulence, and irregular defecation. However, it is unclear if these symptoms are
attributable to the underlying diverticulosis or to coexistent irritable bowel syndrome.
Complicated colonic diverticulosis This is very uncommon but highly dangerous. The
diverticula may bleed, either rapidly (causing bleeding through the rectum) or slowly
(causing anaemia). Constipation: The symptom of constipation may signify different
things in individual cases, depending on what is considered to be “normal”: too little,
too hard or rare a stool, difficult defecation, or the sensation of incomplete emptying.
Constipation is often harmless, but it can be a sign of numerous diseases. The
causes of constipation are: Low-fibre diet, as intestinal motility depends on the
volume of intestinal contents. The larger the volume the greater the motility.! Reflex
and/or psychogenic disorders. These include: 1) anal fissure that is painful and reflex
raises the tone of the anal sphincter, thus increasing the pain, and so on; 2) so-
called animus (outlet obstruction), i.e., contraction (rather than the normal relaxation)
of the pelvic floor when the rectum is stretched. Such a “false” reflex is commonly
found in women who were abused as children, but also in patients with Parkinson’s
disease; 3) paralytic ileus (acute pseudo-obstruction) that may be caused reflex by
operations (particularly in the abdomen), trauma, or peritonitis, and may persist in
the colon for several days. Functional disorders of transport, whether of neurogenic,
myogenic, reflex (see above), medicinal (e.g., opiates), or ischemic cause (e.g.,
trauma or arteriosclerosis of the mesenteric arteries). Functional intestinal
obstruction is called pseudo-obstruction.! Neurogenic causes. Congenital absence of
ganglion cells near the anus (aganglionosis in Hirschsprung’s disease), resulting in
persisting spasm of the affected segment due to failure of receptive relaxation (! A,
bottom right) and absence of anorectal inhibitory reflexes. Myogenic causes.
Muscular dystrophies, scleroderma, dermatomyositis, and systemic lupus
erythematosus.
60. Disorders of the exocrine pancreas. Pancreatitis. Pancreatic insufficiency.
Acute Pancreatitis Most pancreatic enzymes are activated by enteropeptidase only
when they reach the intestinal lumen. The activation of trypsinogen to trypsin is a key
feature in this, because trypsin activates other enzymes. If it is activated in the acinar
cells, the pancreatic trypsin inhibitor–protein is responsible for trypsin not being
effective there. However, if this protective mechanism does not keep up with the
trypsin activation, or trypsin becomes active in the lumen of the pancreatic duct, self-
digestion of the pancreas occurs, i.e., acute pancreatitis. Even though there is a
history of high alcohol consumption and gallstones in 80% of cases, the
pathogenetic mechanism is not quite clear. The following possibilities are being
discussed as playing a part,either in combination or separately depending on the
case: Increased pressure in the pancreatic duct(flow resistance and/or flow too high)
can play a part in the development of acute pancreatitis.
Alcohol, acetylsalicylic acid, histamine, etc. increase the permeability of the
pancreatic duct epithelium, so that larger molecules can pass through it. Enzymes
secreted by the acinar cells thus diffuse into periductal interstitial tissue and damage
it (!A4). In addition,alcohol in the duct system seems to precipitate proteins, causing
a rise in upstream pressure.

VSM
Trypsin activates other enzymes (phospholipase A2, elastase, etc.), clotting factors
(prothrombin to thrombin), tissue hormones (bradykinin and kallidin are activated via
kallikrein), and cytotoxic proteins (complement system). In the pancreas (!A6; P in
the computed tomogram) there is at first generalized cell swelling (pancreatic
edema; Activated elastase, in particular,causes vessel arrosion with bleeding
(hemorrhagic pancreatitis)
As the activated enzymes appear in plasma,where their presence is of diagnostic
significance,hypoalbuminemia develops with resulting hypocalcemia, as well as
systemic vasodilation and plasma exudation (triggered by bradykinin and kallidin),
ultimately ending in circulatory shock. Phospholipase A2 and free fatty acids (due to
increased lipolysis) in plasma destroy the surfactant on the alveolar epithelium,
causing arterial hypoxia. Finally, the kidneys will also be damaged(danger of anuria.)
Chronic Pancreatitis Chronic pancreatitis is an inflammatory process that destroys
the exocrine and endocrine tissues and leads to fibrosis of the organ.Chronic
calcifying pancreatitis (!A, left) is the most common form (70– 80% of cases),caused
by chronic alcohol abuse (> 80 g/d over many years) and is characterized by
irregularly distributed tissue lesions with intraductal protein plugs and stones as well
as atrophy and stenosis of the ductal system. Three mechanisms play a role in its
pathogenesis:
1-While normally, in parallel with the stimulation of the acini (enzyme-rich secretion),
there is greater secretion in the ducts (HCO3–, water), it is reduced in chronic
pancreatitis.As a result, protein concentration in the pancreatic juice is increased.
2. Calcium salts are deposited on the precipitated protein, resulting in the formation
of stones in the lumen of small ducts, and concentric calcium deposits on the walls of
the larger ducts. The cause of all this may be that two components of pancreatic
juice are diminished in chronic pancreatitis, namely those that normally prevent the
precipitation of calcium salts from pancreatic juice. One of these components is
citrate, which binds calcium complexly, the other is the14 kDa protein, lithostatin (=
pancreatic stone protein [PSP]), which holds calcium salts in solution during.
3-Similar to acute pancreatitis (!p.158), intraductal activation of trypsin occurs. This
not only contributes to the autodigestion of pancreatic tissue, but also activates other
aggressive enzymes, such as elastase and phospholipase.
The rarer chronic–obstructive pancreatitis (!A, right) is caused by occlusion of the
main excretory duct(s) by tumors, scar stricture, or stenosis of the papilla, among
others. There is no calcification, but the ductal system is markedly dilated upstream
of the stenosis. The results of chronic pancreatitis are tissue atrophy, ductal
stenosis, and periductal fibrosis with scarring.
Exocrine pancreatic insufficiency (EPI) is the inability to properly digest food due to a
lack of digestive enzymes made by the pancreas. This disease is found frequently in
dogs. EPI is also found in humans afflicted with cystic fibrosis and Shwachman-
Diamond Syndrome. EPI is caused by a progressive loss of the pancreatic cells that
make digestive enzymes. Most commonly in dogs, this is caused by pancreatic
acinar atrophy. The atrophy in turn can be caused by previous infections, a blocked

VSM
pancreatic duct, or genetics. Chronic pancreatitis is the most common cause of EPI
in humans and cats, but it is an uncommon cause in dogs. Loss of digestive
enzymes leads to maldigestion and malabsorption of nutrients. In humans, the
common causes of EPI are Cystic Fibrosis, which is a hereditary recessive disease
of Europeans and Ashkenazi Jews involving a chloride channel called CFTR, and
chronic pancreatitis. Shwachman-Diamond Syndrome is the second most common
cause of pancreatic insufficiency in humans. EPI is also common in diabetes - both
type 1 and type 2. Studies have shown that about 35% of type 2 diabetics and 50%
of type 1 diabetics exhibit symptoms and characteristics of EPI. However, treatment
is normally only initiated once the patient complains of problems with steatorrhea,
bulky stools, abdominal pain, and/or flatulence. Some clinicians refer to this
phenomenon as "diabetic diarrhoea", however this term is rarely explained as a
symptom of EPI possibly because it could also be linked to GI motility problems.
Limited and preliminary studies have indicated that treatment of EPI with products
such as Pancrelipase have an effect on glucose control. Also, there have been only
small differences in the fat-soluble vitamin status of diabetics treated with products
such as Pancrelipase, as steatorrhea can lead to a decrease in the absorption of fat
soluble vitamins.
In animals, signs of EPI are not present until 85 to 90 percent of the pancreas is
unable to secrete its enzymes. Treatment
Often this is treated with Pancreatic Enzyme Products (PEPs), such as pancrelipase,
that are used to break down fats (lipases), proteins (proteases) and carbohydrates
(amylases) into units that can be digested by those with EPI.
61. Etiology and pathogenesis of liver diseases. Types of liver dysfunction.
Manifestations of liver dysfunction.
Liver dysfunction and subsequent damage can be caused by:Metabolic disorders
such as obesity, diabetes, and high triglycerides – When the body’s metabolism is
not running properly, fat can accumulate around the liver, causing what is known as
fatty liver.Alcoholism – Excessive and chronic alcohol consumption is the leading
cause of cirrhosis. Alcohol converts to acetaldehyde, a carcinogenic toxin. Illness –
Certain illnesses, such as tuberculosis and malabsorption syndrome, can cause liver
damage. Pharmaceuticals – Certain prescription medications and over-the-counter
drugs list liver damage as a side effect and a risk. Some of these include
psychotropic medications (such as antidepressants, antipsychotics, and mood
stabilizers), corticosteroids, non-steroidal anti-inflammatory medications (NSAIDs),
and others. It’s always important to review side effects and risks of pharmaceuticals
with your healthcare professional and pharmacist. Chemotherapy – Chemotherapy
drugs can damage the liver as a side effect. Pregnancy – In rare cases, fatty liver
can be a complication of pregnancy.
Poor diet – A diet high in trans fats and heavily refined simple carbohydrates can
cause metabolic disturbances that lead to fatty liver.
Pesticides and heavy metals – These can be stored in body fat over a lifetime and
have the potential to produce liver damage. Viral infections – Hepatitis A – E can

VSM
lead to liver damage. Excess vitamin A – Megadosing of vitamin A can damage the
liver. Candida – Candida yeast ferments sugars into acetaldehyde, which is the
same carcinogen that causes alcohol hangovers. Candida also appears to increase
gut and urinary levels of ammonia, which is another liver toxin. The basic
pathophysiology of all forms of liver disease represents failures of the numerous and
complex hepatic metabolic functions. Although there is some variability in basic
pathophysiology from one type of liver disease to another, all forms of liver disease
can reasonably be called hepatic failure.It should be made clear at the outset that
jaundice, an obvious physical sign, sometimes represents problems other than liver
disease and is by no means synonymous with it. There are other, far more important
and more complex metabolic problems associated with liver disease so that the
problem of jaundice will be dealt with only briefly in this minicourse. The liver plays
several complex roles in amino acid metabolism, protein synthesis, carbohydrate
metabolism and lipid metabolism. It is also the site of manufacture of a number of
blood coagulation proteins.
The basic derangements of hepatic failure will be discussed first. These will then be
applied to the most common forms of liver disease, acute hepatitis and cirrhosis.
In the diseased liver, there are two prime manifestations of liver failure with regard to
lipid metabolism. The first of these is the deposition of triglycerides within the organ
itself. This is the basic mechanism of socalled "fatty liver," which develops most often
as a result of chronic alcoholism. The second prime feature of disordered hepatic
lipid metabolism is a diminution in the rate of synthesis of cholesterol
The liver is responsible for detoxifying, by chemical modification, many substances
that enter it. This includes substances drained from the gut, many kinds of drugs,
and circulating hormones. Some substances are converted to water soluble salts or
esters and are excreted in the bile. Others are chemically modified and released into
the general circulation for renal excretion.
Portal Hypertension; Since the liver receives two thirds of its blood supply from the
hepatic portal vein and onethird from the hepatic artery, circulatory impairment
through the liver results primarily in an increase in venous pressure in the portal
drainage system. This direct increase in hydrostatic pressure in the portal system,
which drains the entire G.I. tract, results in an increase in hydrostatic pressure in all
of the G.I. tract's venous drainage system. This increase in pressure is called portal
hypertension. Acute Liver Failure:Causes of acute liver failure are poisoning and
inflammation, for example, fulminant cholangitis or viral hepatitis (especially in
hepatitis B and E). The causes of chronic liver failure that is accompanied by fibrosis
(cirrhosis) of the liver are: – inflammation, for example, chronic persistent viral
hepatitis; – alcohol abuse, the most common cause;– in susceptible patients, side
effects of drugs, for example, folic acid antagonists, phenylbutazone; –
Cardiovascular causes of impairment of venous return, for example, in right heart
failure – a number of inherited diseases for example, glycogen storage
diseases,Wilson’s disease, galactosemia, hemochromatosis,"1-antitrypsin
deficiency;– intrahepatic or posthepatic cholestasisProtein synthesis in the liver is
reduced.This can lead to hypoalbuminemia that may result in ascites.

VSM
Cholestasis occurs, producing not only liver damage but also aggravating any
bleeding tendency, because the lack of bile salts decreases micellar formation and
with it the absorption of vitamin K from the intestine. Portal hypertension develops
and may make the ascites worse because oflymphatic flow impairment. It may
causethrombocytopenia resulting from splenomegaly. Chronic liver disease: in the
clinical context is a disease process of the liver that involves a process of
progressive destruction and regeneration of the liver parenchyma leading to fibrosis
and cirrhosis. Causes; Hepatitis B,Hepatitis C,Cytomegalovirus (CMV),Epstein Barr
Virus (EBV),Toxic and drugs,Alcoholic liver
disease,Amiodarone,Methotrexate,Nitrofurantoin,Non-alcoholic fatty liver
disease,Haemochromatosis,Wilson's Disease, Autoimmune Chronic
Hepatitis,Primary Biliary Cirrhosis,Primary Sclerosing Cholangitis. Sign of Chronic
Liver Dysfunction: Clubbing,Palmar erythema,Spider nevi (angiomata),Scratch
marks,Gynaecomastia,Feminising hair distribution,Testicular atrophy. The treatment
of chronic liver disease depends on the cause. While some conditions may be
treated with medications, others may require surgery or a transplant. Transplant is
required when the liver fails and there is no other alternative.
The symptoms related to liver dysfunction include both physical signs and a variety
of symptoms related to digestive problems, blood sugar problems, immune
disorders, abnormal absorption of fats, and metabolism problems. The
malabsorption of fats may lead to symptoms that include indigestion, reflux, deficit of
fatsoluble vitamins, hemorrhoids, gall stones, intolerance to fatty foods, intolerance
to alcohol, nausea and vomiting attacks, abdominal bloating, and
constipation.Nervous system disorders include depression, mood changes,
especially anger and irritability, poor concentration and "foggy brain", overheating of
the body, especially the face and torso, and recurrent headaches (including
migraine) associated with nausea. The blood sugar problems include
hypoglycaemia.Abnormalities in the level of fats in the blood stream, whether too
high or too low levels of lipids in the organism. Hypercholesterolemia: elevated LDL
cholesterol, reduced HDL cholesterol, elevated triglycerides, clogged arteries leading
to high blood pressure, heart attacks and strokes, build up of fat in other body organs
(fatty degeneration of organs), lumps of fat in the skin (lipomas and other fatty
tumors), excessive weight gain (which may lead to obesity), inability to lose weight
even while dieting, sluggish metabolism, protuberant abdomen (pot belly), cellulite,
fatty liver, and a roll of fat around the upper abdomen (liver roll) etc.[citation needed]
Or too low levels of lipids: hypocholesterolemia: low total cholesterol, low LDL and
VLDL cholesterol, low triglyderides Hepatitis, inflammation of the liver, is caused
mainly by various viruses (viral hepatitis) but also by some liver toxins (e.g. alcoholic
hepatitis), autoimmunity (autoimmune hepatitis) or hereditary conditions.
Alcoholic liver disease is a term used to describe any hepatic manifestation of
alcohol overconsumption, including fatty liver disease, alcoholic hepatitis, and
cirrhosis.
Fatty liver disease (hepatic steatosis) is a reversible condition where large vacuoles
of triglyceride fat accumulate in liver cells. Cirrhosis is the formation of fibrous tissue
(fibrosis) in the place of liver cells that have died due to a variety of causes, including

VSM
viral hepatitis, alcohol overconsumption, and other forms of liver toxicity. Cirrhosis
causes chronic liver failure. Primary biliary cirrhosis is a serious autoimmune disease
of the bile capillaries. Primary sclerosing cholangitis is a serious chronic
inflammatory disease of the bile duct, which is believed to be autoimmune in origin.
Gilbert's syndrome, a genetic disorder of bilirubin metabolism found in about 5% of
the population, can cause mild jaundice. A number of liver function tests are
available to test the proper function of the liver. These test for the presence of
enzymes in blood that are normally most abundant in liver tissue, metabolites or
products. LFT's, serum proteins, serum albumin, serum globulin, A/G Ratio, alanine
transaminase, aspartate transaminase.
62. Jaundices.
Bilirubin, largely originating from hemoglobin breakdown (ca. 230 mg/d), is taken up
by the liver cells and coupled by glucuronyl transferase to form bilirubin-
monoglucuronide and bilirubindiglucuronide. This water- soluble conjugated (direct
reacting) bilirubin is secreted into the bile canaliculi and 85% is excreted in the stool.
The remaining 15% is deglucuronated and absorbed in the intestine for
enterohepatic recirculation. The normal plasma concentration of bilirubin is
maximally 17 μmol/L (1 mg/dL). If it rises to more than 30 μmol/L, the sclera become
yellow; if the concentration rises further,the skin turns yellow as well (jaundice
[icterus]). Several forms can be distinguished: ! Prehepatic jaundice is the result of
increased bilirubin production, for example, in hemolysis (hemolytic anemia, toxins),
inadequate erythropoiesis (e.g., megaloblastic anemia), massive transfusion
(transfused erythrocytes are short-lived), or absorption of large hematomas. In all
these conditions unconjugated (indirect reacting) bilirubin in plasma is increased.
Intrahepatic jaundice is caused by a specific defect of bilirubin uptake in the liver
cells (Gilbert syndrome Meulengracht), conjugation (neonatal jaundice, Crigler–
Najjar syndrome), or secretion of bilirubin in the bile canaliculi (Dubin–Johnson
syndrome,Rotor syndrome)
In the first two defects it is mainly the unconjugated plasma bilirubin that is
increased; in the secretion type it is the conjugated bilirubin that is increased. All
three steps may be affected in liver diseases and disorders, for example, in viral
hepatitis, alcohol abuse, drug side effects (e.g., isoniazid, phenytoin, halothane),
liver congestion (e.g., right heart failure), sepsis (endotoxins), or poisoning.
When red blood cells have completed their life span of approximately 120 days, or
when they are damaged, their membranes become fragile and prone to rupture. As
each red blood cell traverses through the reticuloendothelial system, its cell
membrane ruptures when its membrane is fragile enough to allow this. Cellular
contents, including hemoglobin, are subsequently released into the blood. The
hemoglobin is phagocytosed by macrophages, and split into its heme and globin
portions. The globin portion, a protein, is degraded into amino acids and plays no
role in jaundice. Two reactions then take place with the heme molecule. The first
oxidation reaction is catalyzed by the microsomal enzyme heme oxygenase and
results in biliverdin (green color pigment), iron and carbon monoxide. The next step
is the reduction of biliverdin to a yellow color tetrapyrol pigment called bilirubin by

VSM
cytosolic enzyme biliverdin reductase. This bilirubin is "unconjugated," "free" or
"indirect" bilirubin. Approximately 4 mg of bilirubin per kg of blood is produced each
day.The majority of this bilirubin comes from the breakdown of heme from expired
red blood cells in the process just described. However approximately 20 percent
comes from other heme sources, including ineffective erythropoiesis, and the
breakdown of other heme-containing proteins, such as muscle myoglobin and
cytochromes.
63. Acute hepatitis. Chronic hepatitis. Etiology & pathogenesis.
Initial features are of nonspecific flu-like symptoms, common to almost all acute viral
infections and may include malaise, muscle and joint aches, fever, nausea or
vomiting,diarrhea, and headache. More specific symptoms, which can be present in
acute hepatitis from any cause, are: profound loss of appetite, aversion to smoking
among smokers, dark urine, yellowing of the eyes and skin (i.e., jaundice) and
abdominal discomfort. Physical findings are usually minimal, apart from jaundice in a
third and tender hepatomegaly(swelling of the liver) in about 10%. Some exhibit
lymphadenopathy (enlarged lymph nodes, in 5%) or splenomegaly (enlargement of
the spleen, in 5%).
Acute viral hepatitis is more likely to be asymptomatic in younger people.In acute
hepatitis the lesions (areas of abnormal tissue) predominantly contain diffuse
sinusoidal and portal mononuclear infiltrates (lymphocytes, plasma cells, Kupffer
cells) and swollen hepatocytes. Acidophilic cells (Councilman bodies) are common.
Hepatocyte regeneration and cholestasis (canalicular bile plugs) typically are
present. Bridging hepaticnecrosis (areas of necrosis connecting two or more portal
tracts) may also occur. There may be some lobular disarray. Although aggregates of
lymphocytes in portal zones may occur these are usually neither common nor
prominent. The normal architecture is preserved. There is no evidence of fibrosis or
cirrhosis (fibrosis plus regenerative nodules). In severe cases prominent
hepatocellular necrosis around the central vein (zone 3) may be seen. In submassive
necrosis – a rare presentation of acute hepatitis – there is widespread hepatocellular
necrosis beginning in the centrizonal distribution and progressing towards portal
tracts. The degree of parenchymal inflammation is variable and is proportional to
duration of disease. Two distinct patterns of necrosis have been recognised: (1)
zonal coagulative necrosis or (2) panlobular (nonzonal) necrosis. Numerous
macrophages and lymphocytes are present. Necrosis and inflammation of the biliary
tree occurs.Hyperplasia of the surviving biliary tract cells may be present. Stromal
haemorrhage is common.
Causes of Acute hepatitis Viral hepatitis :Adenoviridae: Adenoviruses Arenaviruses:
Guanarito virus, Bacterial:Anaplasma,Babesia,Bartonella,Chlamydia
trachomatis,Coxiella Parasitic;Ascaris
lumbricoides,Ancylostoma,Baylisascaris,Capillaria hepatica,Clonorchis sinensis
Fungal;Aspergillus, Candida, Cryptococcus
Hepatitis ; is a medical condition defined by the inflammation of the liver and
characterized by the presence ofinflammatory cells in the tissue of the organ.
Chronic hepatitis often leads to nonspecific symptoms such as malaise, tiredness

VSM
and weakness, and often leads to no symptoms at all. It is commonly identified on
blood testsperformed either for screening or to evaluate nonspecific symptoms. The
occurrence of jaundice indicates advanced liver damage. On physical examination
there may be enlargement of the liver. Extensive damage and scarring of liver (i.e.
cirrhosis) leads to weight loss, easy bruising and bleeding tendencies, peripheral
edema (swelling of the legs) and accumulation ofascites (fluid in the abdominal
cavity). Eventually, cirrhosis may lead to various complications: esophageal varices
(enlarged veins in the wall of the esophagus that can cause life-threatening bleeding)
hepatic encephalopathy (confusion and coma) and hepatorenal syndrome (kidney
dysfunction). Chronic lobular hepatitis was the term used to describe chronic
hepatitis with persistent parenchymal focal hepatocyte necrosis (apoptosis) with
mononuclear sinusoidal infiltrates. This is now referred to as 'chronic hepatitis
without piecemeal necrosis (or interface hepatitis).' Cirrhosis is a diffuse process
characterized by regenerative nodules that are separated from one another by bands
of fibrosis. It is the end stage for many chronic liver diseases. The pathophysiological
process that results in cirrhosis is as follows: hepatocytes are lost through a gradual
process of hepatocellular injury and inflammation. This injury stimulates a
regenerative response in the remaining hepatocytes. The fibrotic scars limit the
extent to which the normal architecture can be reestablished as the scars isolate
groups of hepatocytes. This results in nodules formation. Angiogenisis (new vessel
formation) accompanies scar production which results in the formation of abnormal
channels between the central hepatic veins and the portal vessels. This in turn
causes shunting of blood around the regenerating parenchyma. Normal vascular
structures including the sinusoidal channels may be obliterated by fibrotic tissue
leading to portal hypertension. The overall reduction in hepatocyte mass, in
conjunction with the portal blood shunting, prevents the liver from accomplishing its
usual functions – the filtering of blood from the gastrointestinal tract and serum
protein production. These changes give rise to the clinical manifestations of cirrhosis.
Acne, abnormal menstruation, lung scarring, inflammation of the thyroid gland and
kidneys may be present in women with autoimmune hepatitis Chronic active hepatitis
was the term used to described cases of hepatitis for more than 6 months with portal
based inflammation, fibrosis, disruption of the terminal plate and piecemeal necrosis.
This term has now been replaced by the diagnosis of 'chronic hepatitis with
piecemeal (periportal) necrosis (or interface hepatitis) with or without fibrosis.
Causes of chronic hepatitis:Alcohol,Autoimmune,Autoimmune
hepatitis,Drugs;Isoniazid,Ketoconazole,Methyldopa,Nitrofurantoin .Wilson's disease
64. Cirrhosis. Etiology & pathogenesis.
Cirrhosis is a consequence of chronic liver disease characterized by replacement of
liver tissue by fibrosis, scar tissue and regenerative nodules (lumps that occur as a
result of a process in which damaged tissue is regenerated),[1][2][3] leading to loss
of liver function. Cirrhosis is most commonly caused by alcoholism, hepatitis B and
C, and fatty liver disease, but has many other possible causes. Some cases are
idiopathic, i.e., of unknown cause.Ascites (fluid retention in the abdominal cavity) is
the most common complication of cirrhosis, and is associated with a poor quality of
life, increased risk of infection, and a poor long-term outcome. Other potentially life-

VSM
threatening complications are hepatic encephalopathy (confusion and coma) and
bleeding from esophageal varices. Cirrhosis is generally irreversible, and treatment
usually focuses on preventing progression and complications. In advanced stages of
cirrhosis the only option is a liver transplant.
Pathogenesis; The liver plays a vital role in synthesis of proteins e.g., albumin,
clotting factors and complement), detoxification and storage (e.g., vitamin A). In
addition, it participates in the metabolism of lipids and carbohydrates.Cirrhosis is
often preceded by hepatitis and fatty liver (steatosis), independent of the cause. If
the cause is removed at this stage, the changes are still fully reversible. The
pathological hallmark of cirrhosis is the development of scar tissue that replaces
normal parenchyma, blocking the portal flow of blood through the organ and
disturbing normal function. Recent research shows the pivotal role of the stellate cell,
a cell type that normally stores vitamin A, in the development of cirrhosis. Damage to
the hepatic parenchyma leads to activation of the stellate cell, which becomes
contractile (called myofibroblast) and obstructs blood flow in the circulation. In
addition, it secretes TGF-β1, which leads to a fibrotic response and proliferation of
connective tissue. Etiology of cirrhosis Alcoholic liver disease (ALD). Alcoholic
cirrhosis develops for between 10% and 20% of individuals who drink heavily for a
decade or more.[8] Alcohol seems to injure the liver by blocking the normal
metabolism of protein, fats, and carbohydrates. Chronic hepatitis B. The hepatitis B
virus causes liver inflammation and injury that over several decades can lead to
cirrhosis. Hepatitis D is dependent on the presence of hepatitis B and accelerates
cirrhosis in co-infection. Primary biliary cirrhosis. May be asymptomatic or complain
of fatigue, pruritus, and non-jaundice skin hyperpigmentation with hepatomegaly.
Autoimmune hepatitis. This disease is caused by the immunologic damage to the
liver causing inflammation and eventually scarring and cirrhosis. Findings include
elevations in serum globulins, especially gamma globulins. Wilson's disease.
Autosomal recessive disorder characterized by low serum ceruloplasmin and
increased hepatic copper content on liver biopsy. Cardiac cirrhosis. Due to chronic
right sided heart failure which leads to liver congestion.
Galactosemia.
65. Cirrhosis. Clinical manifestations - portal hypertension, ascites and hepatic
encephalopathy.
Liver cirrhosis is a disease in which necrosis, inflammation, fibrosis, nodular
regeneration,and formation of vascular anastomoses develop more or less
simultaneously. It is usually caused by the long-term action of noxious factors,
especially alcohol abuse,which is the cause in 50% of cases worldwide.While the
probability of cirrhosis developing after a cumulative uptake of 13 kg
ethanol/kg body weight is only about 20%, it rises to over 90% after 40 kg. The
substance that is most responsible for the development of fibrosis, and thus
cirrhosis, is the ethanol metabolite acetaldehyde. Cirrhosis can also be the final
stage of viral hepatitis (20–40% of cirrhosis cases in Europe). In acute fulminant
disease it may develop in a matter of weeks; in chronic recurrent disease after
months or years. It can also occur after an obstruction to blood outflow (congestive

VSM
liver; !p.170) or after other liver damage, for example, as final stage of a storage
disease (hemochromatosis, Wilson’s disease) or genetically determined enzyme
deficiency. Factors involved in liver-cell damage are:
– ATP deficiency due to abnormal cellular
energy metabolism;
– increased formation of highly reactive
oxygen metabolites (•O2
–, •HO2, H2O2) with
– concomitant deficiency of antioxidants
(e.g., glutathione) and/or damage of protective
enzymes (glutathione peroxidase,
superoxide dismutase).
The O2 metabolites react with, for example,unsaturated fatty acids in phospholipids
(lipidperoxidation). This contributes to damage of plasma membranes and cell
organelles (lysosomes, endoplasmic reticulum). As a result, cytosolic Ca2+
concentration rises, activating proteases and other enzymes so that the cells are
ultimately irreversibly damaged.
Portal Hypertension: Venous blood from stomach, intestines, spleen, pancreas, and
gallbladder passes via the portal vein to the liver where, in the sinusoids after
mixture with oxygen-rich blood of the hepatic artery, it comes into close contact with
the hepatocytes.
Consequences of portal hypertension. Wherever the site of obstruction, an increased
portal vein pressure will lead to disorders in the preceding organs (malabsorption,
splenomegaly with anemia and thrombocytopenia)as well as to blood flowing from
abdominal organs via vascular channels that bypass the liver. These portal bypass
circuits use collateral vessels that are normally thin-walled but are now greatly
dilated (formation of varices; “haemorrhoids” of the rectal venous plexus; caput
medusae at the paraumbilical veins). The enlarged esophageal veins are particularly
in danger of rupturing. Ascites ; is a gastroenterological term for an accumulation of
fluid in the peritoneal cavity. The medical condition is also known as peritoneal cavity
fluid, peritoneal fluid excess, hydroperitoneum or more archaically as abdominal
dropsy. Although most commonly due to cirrhosis and severe liver disease, its
presence can portend other significant medical problems. Diagnosis of the cause is
usually with blood tests, an ultrasound scan of the abdomen, and direct removal of
the fluid by needle or paracentesis (which may also be therapeutic). Treatment may
be with medication (diuretics), paracentesis, or other treatments directed at the
cause. Hepatic encephalopathy (also known as portosystemic encephalopathy) is
the occurrence of confusion, altered level of consciousness, and coma as a result of
liver failure. In the advanced stages it is called hepatic coma or coma hepaticum. It
may ultimately lead to death. It is caused by accumulation in the bloodstream of toxic

VSM
substances that are normally removed by the liver. The diagnosis of hepatic
encephalopathy requires the presence of impaired liver function and the exclusion of
an alternative explanation for the symptoms. Blood tests (ammonia levels) may
assist in the diagnosis. Attacks are often precipitated by an intercurrent problem,
such as infection or constipation.
66. Disturbances in lipids digestion and absorption in the gastro-intestinal tract.
The mechanisms for lipid digestion and absorption are more complex and involve
more steps than those for carbohydrate and protein. Thus, there are also more steps
at which an abnormality of lipid digestion or absorption can occur. Each step in the
normal process is essential: pancreatic enzyme secretion and function, bile acid
secretion, emulsification, micelle formation, diffusion of lipids into intestinal epithelial
cells, chylomicron formation, and transfer of chylomicrons into lymph. An abnormality
at any one of the steps will interfere with lipid absorption and result in steatorrhea (fat
excreted in feces).
Pancreatic insufficiency. Diseases of the exocrine pancreas (e.g., chronic
pancreatitis and cystic fibrosis) result in failure to secrete adequate amounts of
pancreatic enzymes, including those involved in lipid digestion, pancreatic lipase and
colipase, cholesterol ester hydrolase, and phospholipase A2. For example, in the
absence of pancreatic lipase, triglycerides cannot be digested to monoglycerides
and free fatty acids. Undigested triglycerides are not absorbable and are excreted in
feces. Acidity of duodenal contents. If the acidic chyme delivered to the
duodenum is not adequately neutralized by the HCO3--containing pancreatic
secretions, then pancreatic enzymes are inactivated (i.e., the pH optimum for
pancreatic lipase is 6). The gastric chyme, which is delivered to the duodenum, has
a pH ranging from 2 at the pylorus to 4 at the duodenal bulb. Sufficient HCO3- must
be secreted in pancreatic juice to neutralize the H+ and increase the pH to the range
where pancreatic enzymes function optimally. There are two reasons that all of the
H+ delivered from the stomach might not be neutralized: (1) Gastric parietal cells
may be secreting excessive quantities of H+, causing an overload to the duodenum;
or (2) the pancreas may fail to secrete sufficient quantities of HCO3- in pancreatic
juice. The first reason is illustrated by Zollinger-Ellison syndrome, in which a tumor
secretes large quantities of gastrin. The elevated levels of gastrin stimulate
excessive secretion of H+ by the gastric parietal cells, and this H+ is delivered to the
duodenum, overwhelming the ability of pancreatic juices to neutralize it. The second
reason is illustrated by disorders of the exocrine pancreas (e.g., pancreatitis) in
which there is impaired HCO3- secretion (in addition to impaired enzyme secretion).
Deficiency of bile salts. Deficiency of bile salts interferes with the ability to form
micelles, which are necessary for solubilization of the products of lipid digestion. Ileal
resection (removal of the ileum) interrupts the enterohepatic circulation of bile salts,
which then are excreted in feces rather than being returned to the liver. Because the
synthesis of new bile salts cannot keep pace with the fecal loss, the total bile salt
pool is reduced. Bacterial overgrowth. Bacterial overgrowth reduces the
effectiveness of bile salts by deconjugating them. In other words, bacterial actions
remove glycine and taurine from bile salts, converting them to bile acids. Recall that
at intestinal pH, bile acids are primarily in the nonionized form (since their pKs are

VSM
higher than intestinal pH); the nonionized form is lipid soluble and readily absorbed
by diffusion across the intestinal epithelial cells. For this reason, the bile acids are
absorbed "too early" (before reaching the ileum), before micelle formation and lipid
absorption is completed. Similarly, decreased pH in the intestinal lumen promotes
"early" absorption of bile acids by converting them to their nonionized form.
Decreased intestinal cells for absorption. In conditions such as tropical sprue, the
number of intestinal epithelial cells is reduced, which reduces the microvillar surface
area. Since lipid absorption across the apical membrane occurs by diffusion, which
depends on surface area, lipid absorption is impaired because the surface area for
absorption is decreased. Failure to synthesize apoproteins. Failure to synthesize
Apo B (β-lipoprotein) causes abetalipoproteinemia. In this disease, chylomicrons
either do not form or are unable to be transported out of intestinal cells into lymph. In
either case, there is decreased absorption of lipids into blood and a buildup of lipid
within the intestinal cells.
67. Disturbances in lipids transport. Hyperlipidemia. Fatty liver.
Lipoproteins are large, mostly spherical complexes that transport lipids (triglycerides,
cholesteryl esters, and fat-soluble vitamins) through body fluids (plasma, interstitial
fluid, and lymph) to and from tissues.Lipids such as free fatty acids (FFAs),
cholesterol, and triglycerides are hydrophobic molecules that bind protein for
transport. Nonesterified FFAs travel as anions complexed to albumin. Esterified
complex lipids are transported in lipoprotein particles. Lipoproteins have a
hydrophobic core (cholesteryl esters and triglycerides) and an amphiphilic surface
monolayer (phospholipids, unesterified cholesterol, and apolipoproteins). Proteins on
the surface of lipoproteins, apolipoproteins (apo), activate enzymes and receptors
that guide lipid metabolism. Ultracentrifugation separates lipoproteins into five
classes based on their density: chylomicrons, very low density lipoproteins (VLDL),
intermediate-density lipoproteins (IDL), low-density lipoproteins (LDL), and high-
density lipoproteins (HDL).
The density of a lipoprotein is determined by the amount of lipid and protein per
particle. HDL is the smallest and most dense lipoprotein, whereas chylomicrons and
VLDL are the largest and least dense lipoprotein particles.Most triglyceride is
transported in chylomicrons or VLDL, and most cholesterol is carried as cholesteryl
esters in LDL and HDL. ApoA-I, which is synthesized in the liver and intestine, is
found on virtually all HDL particles. ApoA-II is the second most abundant HDL
apolipoprotein. ApoB is the major structural protein of chylomicrons, VLDL, IDL, and
LDL; one molecule of apoB, either apoB-48 (chylomicrons) or apoB-100 (VLDL, IDL,
or LDL), is present on each lipoprotein particle. The human liver makes only apoB-
100, and the intestine makes apoB-48. ApoE is present in multiple copies on
chylomicrons, VLDL, and IDL and plays a critical role in the metabolism and
clearance of triglyceride-rich particles. Three apolipoproteins of the C-series (apoC-
I, -II, and -III) also participate in the metabolism of triglyceride-rich lipoproteins.
Transport of Dietary Lipids (Exogenous Pathway): The exogenous pathway of
lipoprotein metabolism permits efficient transport of dietary lipids. Dietary
triglycerides are hydrolyzed by pancreatic lipases within the intestinal lumen and are

VSM
emulsified with bile acids to form micelles. Dietary cholesterol and retinol are
esterified (by the addition of a fatty acid) in the enterocyte to form cholesteryl esters
and retinyl esters, respectively. Longer-chain fatty acids (12 carbons) are
incorporated into triglycerides and packaged with apoB- 48, cholesteryl esters, retinyl
esters, phospholipids, and cholesterol to form chylomicrons.
Nascent chylomicrons are secreted into the intestinal lymph and delivered directly to
the systemic circulation, where they are extensively processed by peripheral tissues
before reaching the liver.
The particles encounter lipoprotein lipase (LPL), which is anchored to proteoglycans
that decorate the capillary endothelial surfaces of adipose tissue, heart, and skeletal
muscle. The triglycerides of chylomicrons are hydrolyzed by LPL, and free fatty acids
are released; apoC-II, which is transferred to circulating chylomicrons, acts as a
cofactor for LPL in this reaction. The released free fatty acids are taken up by
adjacent myocytes or adipocytes and either oxidized or reesterified and stored as
triglyceride. Some free fatty acids bind albumin and are transported to other tissues,
especially the liver. The chylomicron particle progressively shrinks in size as the
hydrophobic core is hydrolyzed and the hydrophilic lipids (cholesterol and
phospholipids) on the particle surface are transferred to HDL. The resultant smaller,
more cholesterol ester–rich particles are referred to as chylomicron remnants.
The remnant particles are rapidly removed from the circulation by the liver in a
process that requires apoE. Transport of Hepatic Lipids (Endogenous Pathway):
The endogenous pathway of lipoprotein metabolism refers to the hepatic secretion
and metabolism of VLDL to IDL and LDL. VLDL particles resemble chylomicrons in
protein composition but contain apoB-100. The triglycerides of VLDL are derived
predominantly from the esterification of long-chain fatty acids. The packaging of
hepatic triglycerides with the other major components of the nascent VLDL particle
(apoB-100, cholesteryl esters, phospholipids, and vitamin E) requires the action of
the enzyme microsomal transfer protein (MTP). After secretion into the plasma,
VLDL acquires multiple copies of apoE and apolipoproteins of the C series. The
triglycerides of VLDL are hydrolyzed by LPL, especially in muscle and adipose
tissue. As VLDL remnants undergo further hydrolysis, they continue to shrink in size
and become IDL, which contain similar amounts of cholesterol and triglyceride. The
liver removes approximately 40 to 60% of VLDL remnants and IDL by LDL receptor–
mediated endocytosis via binding to apoE. The remainder of IDL is remodeled by
hepatic lipase (HL) to form LDL; during this process, most of the triglyceride in the
particle is hydrolyzed and all apolipoproteins except apoB-100 are transferred to
other lipoproteins. The cholesterol in LDL accounts for70% of the plasma cholesterol
in most individuals. Approximately 70% of circulating LDLs are cleared by LDL
receptor–mediated endocytosis in the liver. ¤ Familial Hypercholesterolemia:
Mutations in the gene that encodes the LDL (apo B/E) receptor results in familial
hypercholesterolemia (FH). Impairment in LDL receptor synthesis or function
decreases the clearance of LDL and increases circulating LDL levels. Excess LDL
then enters macrophages through scavenger receptors resulting in foam cell and
cholesterol plaque formation. These plaques deposit in arteries (atheroma), skin or
tendons (xanthoma), eyelids (xanthelasma), and iris (corneal arcus). Homozygous

VSM
form (rare): Affected individuals present early in life with elevated total cholesterol
(600 to 1000 mg/dL) and LDL (550 to 950 mg/dL). Triglyceride and HDL levels are
normal. They develop CHD, aortic stenosis due to atherosclerosis of the aortic root,
and tendon xanthomas. These xanthomas commonly involve the Achilles tendon and
present as tendonitis. If untreated, patients with HM FH typically die from myocardial
infarction before age 20 years.
The heterozygous (HZ) form: Partial receptor defect resulting in cells displaying half
the normal number of fully functional LDL receptors. These individuals have lower
elevated total cholesterol (>300 to 600 mg/dL) and LDL (250 to 500 mg/dL) levels.
Premature CHD and tendon xanthomas are characteristic clinical findings.
FH can be established by identifying one of the many known gene mutations in the
LDL receptor or by demonstrating diminished LDL receptor function. The diagnosis
of FH usually is made on the basis of clinical features. Elevated total cholesterol
(>300 mg/dL) and LDL (>250 mg/dL) in an individual with personal or family history
of premature CHD and tendon xanthomas identifies individuals at risk for FH.
Treatment requires a low-fat (<20% of total calories), low cholesterol (<100 mg per
day) diet in combination with drug therapy.
¤ Familial Defective Apolipoprotein B-100: In this autosomal dominant disorder, a
defect in the apo B-100 protein results in impaired binding of LDL particles to the
LDL receptor. Most cases are linked to a single mutation (glutamine for arginine
substitution) in the apo B-100 gene.
The clinical presentation is similar to FH, with elevated total cholesterol and LDL
levels associated with premature CHD and tendon xanthomas but are milder than in
FH because apo E–mediated clearance of remnant particles is still functional. Total
cholesterol ranges from 350 to 550 mg/dL in HM and 200 to 350 mg/dL in HZ.
¤ Polygenic Hypercholesterolemia: Apo E play a role in the pathogenesis. Apo E4 on
chylomicron and VLDL remnants has a high affinity for the LDL receptor. Elevated
binding of apo E4–containing lipoproteins to LDL receptors downregulate LDL
receptor synthesis and
increase circulating LDL levels. Environmental factors such as diet can influence
chylomicron and VLDL production, resulting in downregulation of the LDL receptor in
high apo E4 conditions. This leads to an increased propensity for CHD.
¤ Familial Hypertriglyceridemia: Familial hypertriglyceridemia is an autosomal
dominant disorder characterized by overproduction of hepatic VLDL. Secondary
factors that increase VLDL, such as diabetes, alcohol ingestion, and estrogen
therapy, appear to exacerbate this condition.
Low HDL associated with familial hypertriglyceridemia is related to increased
catabolism. Individuals with this condition present with hypertriglyceridemia (200 to
500 mg/dL) and low HDL (<35 mg/dL). The decrease in HDL increases the risk for
CHD. Secondary factors can exacerbate the problem by raising triglyceride levels
above 1000 mg/dL and increasing the risk for pancreatitis and xanthomas.

VSM
This diagnosis is considered in individuals with a family and personal history of
hypertriglyceridemia, CHD, and normal LDL levels. A cloudy infranant noted after
overnight refrigeration of plasma identifies a disorder of VLDL metabolism.
Treatment starts with management of secondary factors that may exacerbate the
condition. Dietary fat restriction (<10% of calories) and drug therapy with niacin and
fibric acid derivates should be initiated.
¤ Lipoprotein Lipase Deficiency: Mutations in the LPL gene resulting in deficiency of
LPL synthesis or function lead to increased circulating chylomicron and VLDL
particles and severe hypertriglyceridemia.
Homozygous (rare): It presents in childhood with triglycerides higher than 1000
mg/dL.
Heterozygous: Usually requires a precipitating factor such as uncontrolled diabetes
to manifest the phenotype. These individuals have moderate hypertriglyceridemia
(250 to 750 mg/dL) that can increase to levels above 1000 mg/dL with secondary
factors, such as insulin resistance or estrogen therapy. This can result in the
chylomicronemia syndrome, characterized by marked hypertriglyceridemia (>1000 to
2000 mg/dL), pancreatitis, eruptive xanthomas, lipemia retinalis, and
hepatosplenomegaly.
Visual inspection demonstrates lipemic plasma. After refrigeration for 12 hours, a
creamy top layer (increased chylomicrons) or turbid plasma infranatant (increased
VLDL), or both, can be demonstrated.
A diet low in fat (<10% of total calories or <20 to 25 g per day) is the primary
treatment. Secondary factors such as uncontrolled diabetes and alcohol use should
be addressed.
¤ Apolipoprotein C-II Deficiency: Apo C-II is an activating cofactor for lipoprotein
lipase. Deficiency of apo C-II is a rare autosomal recessive disorder that leads to
increased chylomicrons and VLDL particles in the circulation, resulting in severe
hypertriglyceridemia.Clinical manifestations are similar to LPL deficiency, including
hypertriglyceridemia (>1000 mg/dL) and symptoms of pancreatitis, eruptive
xanthomas, lipemia retinalis, and hepatosplenomegaly. ¤ Familial Combined
Hyperlipoproteinemia: is an autosomal dominant polygenic disorder. In FCHL,
excess VLDL is synthesized in the liver. VLDL is hydrolyzed by LPL to produce LDL.
Mutations in the LPL gene affecting its expression or function can decrease the
efficiency of VLDL catabolism. Diminished LPL activity increases circulating VLDL
triglyceride. Furthermore, fewer VLDL remnant particles are available for HDL
synthesis. Therefore, FCHL needs to be considered in all patients with total
cholesterol level higher than 250 mg/dL, triglycerides higher than 175 mg/dL, or HDL
lower than 35 mg/dL. Patients typically have high apo B (>120 mg/dL) and a low LDL
cholesterol–to–apo B-100 ratio (<1.2). They accumulate small dense LDL particles,
which are thought to be atherogenic and contribute to premature coronary artery
disease. Affected individuals require a low-fat, lowcholesterol diet plus multiple lipid-
lowering drugs. ¤ Primary Hypoalphalipoproteinemia: Hypoalphalipoproteinemia is
defined as a plasma HDL-C level below the 10th percentile in the setting of relatively

VSM
normal cholesterol and triglyceride levels. This syndrome is often referred to as
“isolated low HDL.” The metabolic etiology of this disease appears to be primarily
accelerated catabolism of HDL and its apolipoproteins. There is an increased
incidence of premature atherosclerosis.
Fatty liver is characterized by the accumulation of fat in hepatocytes, a condition
called steatosis. The liver becomes yellow and enlarges owing to excessive fat
accumulation. The pathogenesis of fatty liver can depend on the amount of alcohol
consumed, dietary fat content, body stores of fat, hormonal status, and other factors.
68. Disturbances in lipids metabolism in fat tissue. Obesity. Metabolic syndrome.
Fat cells, residing within widely distributed adipose tissue depots, are adapted to
store excess energy efficiently as triglyceride and, when needed, to release stored
energy as free fatty acids for use at other sites. This physiologic system,
orchestrated through endocrine and neural pathways, permits humans to survive
starvation for as long as several months. Definition and Measurement: Obesity is a
state of excess adipose tissue mass. Although not a direct measure of adiposity, the
most widely used method to gauge obesity is the body mass index (BMI), which is
equal to weight/height2 (in kg/m2). Other approaches to quantifying obesity include
anthropometry (skin-fold thickness), densitometry (underwater weighing), computed
tomography (CT) or magnetic resonance imaging (MRI), and electrical impedance.
Based on unequivocal data of substantial morbidity, a BMI of 30 is most commonly
used as a threshold for obesity in both men and women. Individuals with BMIs
between 25 and 30 are termed overweight and should be viewed as medically
significant and worthy of therapeutic intervention, especially in the presence of risk
factors that are influenced by adiposity, such as hypertension and glucose
intolerance. The distribution of adipose tissue in different anatomic depots also has
substantial implications for morbidity. Specifically, intraabdominal and abdominal
subcutaneous fat have more significance than subcutaneous fat present in the
buttocks and lower extremities. This distinction is most easily made by determining
the waist-to-hip ratio, with a ratio >0.9 in women and >1.0 in men being abnormal.
Intraabdominal adipocytes are more lipolytically active than those from other depots.
Release of free fatty acids into the portal circulation has adverse metabolic actions,
especially on the liver. Body weight is regulated by both endocrine and neural
components that ultimately influence the effector arms of energy intake and
expenditure. This exquisite regulation of energy balance cannot be monitored easily
by calorie-counting in relation to physical activity. Rather, body weight regulation or
dysregulation depends on a complex interplay of hormonal and neural signals.
Alterations in stable weight by forced overfeeding or food deprivation induce
physiologic changes that resist these perturbations: with weight loss, appetite
increases and energy expenditure falls; with overfeeding, appetite falls and energy
expenditure increases. This latter compensatory mechanism frequently fails,
however, permitting obesity to develop when food is abundant and physical activity is
limited. A major regulator of these adaptative responses is the adipocyte-derived
hormone leptin, which acts through brain circuits (predominantly in the

VSM
hypothalamus)to influence appetite, energy expenditure, and neuroendocrine
function.
Appetite is influenced by many factors that are integrated by the brain, most
importantly within the hypothalamus. Signals that impinge on the hypothalamic
center include neural afferents, hormones, and metabolites. Vagal inputs are
particularly important, bringing information from viscera, such as gut distention.
Hormonal signals include leptin, insulin, cortisol, and gut peptides such as ghrelin,
peptide YY (PYY), and cholecystokinin, which signal to the brain through direct
action on hypothalamic control centers and/ or via the vagus nerve. Metabolites,
including glucose, can influence appetite, as seen by the effect of hypoglycemia to
induce hunger. These diverse hormonal, metabolic, and neural signals act by
influencing the expression and release of various hypothalamic peptides [e.g.,
neuropeptide Y (NPY), Agouti-related peptide (AgRP), α melanocyte-stimulating
hormone (α-MSH), and melanin-concentrating hormone (MCH)] that are integrated
with serotonergic, catecholaminergic, and opioid signaling pathways.
Energy expenditure includes the following components:
(1) resting or basal metabolic rate;
(2) the energy cost of metabolizing and storing food;
(3) the thermic effect of exercise; and
(4) adaptive thermogenesis, which varies in response to chronic caloric intake (rising
with
increased intake). Basal metabolic rate accounts for about 70% of daily energy
expenditure, whereas active physical activity contributes 5 to 10%. The Adipocyte
And Adipose Tissue: Adipose tissue is composed of the lipid-storing adipose cell and
a stromal/vascular compartment in which preadipocytes reside. Adipose mass
increases by enlargement of adipose cells through lipid deposition, as well as by an
increase in the number of adipocytes. The process by which adipose cells are
derived from a mesenchymal preadipocyte involves an orchestrated series of
differentiation steps mediated by a cascade of specific transcription factors. One of
the key transcription factors is peroxisome proliferator-activated receptor γ (PPAR γ),
a nuclear receptor.The adipocyte is also an endocrine cell that releases numerous
molecules in a regulated fashion. These include the energy balance-regulating
hormone leptin, cytokines such as tumor necrosis factor (TNF α), complement
factors such as factor D (alsoknown as adipsin), prothrombotic agents such as
plasminogen activator inhibitor I, and a component of the blood pressure regulating
system, angiotensinogen. Adiponectin enhances insulin sensitivity and lipid
oxidation, whereas resistin may induce insulin resistance. These factors play a role
in the physiology of lipid homeostasis, insulin sensitivity, blood pressure control, and
coagulation and are likely to contribute to obesity-related pathologies.Etiology of
Obesity: Obesity is a heterogeneous group of disorders.Obesity is commonly seen in
families, and the heredibility of body weight is similar to that for height. Inheritance is
usually not Mendelian, however, and it is difficult to distinguish the role of genes and
environmental factors.The environment plays a key role in obesity, as evidenced by

VSM
the fact that famine prevents obesity in even the most obesity-prone individual.
Cultural factors are also important— these relate to both availability and composition
of the diet and to changes in the level of physical activity.Leptin is secreted by
adipose cells and acts primarily through the hypothalamus. Its level of production
provides an index of adipose energy stores. High leptin levels decrease food intake
and increase energy expenditure.The OB gene is present in humans and expressed
in fat. Obesity caused by inactivating mutations in either leptin or the leptin receptor
have been described, thus demonstrating the biologic relevance of leptin in humans.
The obesity in these individuals begins shortly after birth, is severe, and is
accompanied by neuroendocrine abnormalities. The most prominent of these is
hypogonadotropic hypogonadism, which is reversed by leptin replacement.Central
hypothyroidism and growth retardation are seen in the mouse model, but their
occurrence in leptin-deficient humans is less clear.Some disorders contribute to
generation of obesity including; Cushing’s syndrome, Hypothyroidism (not common),
Insulinoma (overeating to avoid hypoglycemia symptoms), some hypothalamic
disorders.
Pathogenesis of Common Obesity: Obesity can result from increased energy intake,
decreased energy expenditure, or a combination of the two. Thus, identifying the
etiology of obesity should involve measurements of both parameters.
There is increased interest in the concept of a body weight “set point.” There is a
sensing system in adipose tissue that reflects fat stores and a receptor, or
“adipostat,” that is in the hypothalamic centers. When fat stores are depleted, the
adipostat signal is low, and the hypothalamus responds by stimulating hunger and
decreasing energy expenditure to conserve energy. Conversely, when fat stores are
abundant, the signal is increased, and the hypothalamus responds by decreasing
hunger and increasing energy expenditure.
Average energy expenditure increases as people get more obese, due primarily to
the fact that metabolically active lean tissue mass increases with obesity. Given the
laws of thermodynamics, the obese person must therefore eat more than the
average lean person to maintain their increased weight. Leptin in Typical Obesity:
The vast majority of obese people have increased leptin levels but do not have
mutations of either leptin or its receptor. They appear, therefore, to have a form of
functional “leptin resistance.” Pathologic Consequences of Obesity: Obesity has
major adverse effects on health. Morbidly obese individuals have as much as a
twelvefold increase in mortality. ¤ Insulin Resistance and Type 2 Diabetes Mellitus:
Hyperinsulinemia and insulin resistance are pervasive features of obesity, increasing
with weight gain and diminishing with weight loss. Insulin resistance is more strongly
linked to intraabdominal fat than to fat in other depots.
The molecular link between obesity and insulin resistance in tissues such as fat,
muscle, and liver include:
(1) insulin itself, by inducing receptor downregulation;
(2) free fatty acids, known to be increased and capable of impairing insulin action;
(3) intracellular lipid accumulation; and

VSM
(4) various circulating peptides produced by adipocytes, including the cytokines TNF-
α and interleukin (IL)6 , and the “adipokines” adiponectin and resistin, which are
produced by adipocytes, have altered expression in obese adipocytes, and are
capable of modifying insulin action.
¤ Reproductive Disorders: Male hypogonadism is associated with increased adipose
tissue. In men, plasma testosterone and sex hormone–binding globulin (SHBG)are
often reduced, and estrogen levels (derived from conversion of adrenal androgens in
adipose tissue) are increased. Gynecomastia may be seen. However,
masculinization, libido, potency, and spermatogenesis are preserved in most of
these individuals.
Obesity is associated with menstrual abnormalities in women, particularly in women
with upper body obesity. Common findings are increased androgen production,
decreased SHBG, and increased peripheral conversion of androgen to estrogen.
Most obese women with oligomenorrhea have the polycystic ovarian syndrome
(PCOS), with its associated anovulation and ovarian hyperandrogenism. The
increased conversion of androstenedione to estrogen, which occurs to a greater
degree in women with lower body obesity, may contribute to the increased incidence
of uterine cancer in postmenopausal women with obesity.
¤ Cardiovascular Disease: When the additional effects of hypertension and glucose
intolerance associated with obesity are included, the adverse impact of obesity is
even more evident. Obesity, especially abdominal obesity, is associated with an
atherogenic lipid profile, with increased lowdensity lipoprotein (LDL) cholesterol, very
low density lipoprotein, and triglyceride, and decreased high-density lipoprotein
cholesterol. Obesity is also associated with hypertension. Obesity-induced
hypertension is associated with increased peripheral resistance and cardiac output,
increased sympathetic nervous system tone, increased salt sensitivity, and insulin-
mediated salt retention; it is often responsive to modest weight loss. ¤ Pulmonary
Disease: These include reduced chest wall compliance, increased work of breathing,
increased minute ventilation due to increased metabolic rate, and decreased total
lung capacity and functional residual capacity. Severe obesity may be associated
with obstructive sleep apnea and the “obesity hypoventilation syndrome”. ¤
Gallstones: Obesity is associated with enhanced biliary secretion of cholesterol,
supersaturation of bile, and a higher incidence of gallstones, particularly cholesterol
gallstones. Paradoxically, fasting increases supersaturation of bile by decreasing the
phospholipid component.¤ Cancer: Obesity in males is associated with higher
mortality from cancer, including cancer of the esophagus, colon, rectum, pancreas,
liver, and prostate; obesity in females is associated with higher mortality from cancer
of the gallbladder, bile ducts, breasts, endometrium, cervix, and ovaries.¤ Bone,
Joint, and Cutaneous Disease: Obesity is associated with an increased risk of
osteoarthritis, partly due to the trauma of added weight bearing and joint
malalignment. The prevalence of gout may also be increased. Among the skin
problems is acanthosis nigricans, manifested by darkening and thickening of the skin
folds on the neck, elbows, and dorsal interphalangeal spaces. Friability of skin may
be increased, especially in skin folds, enhancing the risk of fungal and yeast
infections. Finally, venous stasis is increased in the obese.

VSM
69. Ketosis. Atherosclerosis
In humans and most other mammals, acetyl-CoA formed in the liver during oxidation
of fatty acids can either enter the citric acid cycle or undergo conversion to the
“ketone bodies,” acetone, acetoacetate, and D-β-hydroxybutyrate, for export to other
tissues. (these compounds are quite soluble in blood and urine.) Acetone, produced
in smaller quantities than the other ketone bodies, is exhaled. Acetoacetate and D-β-
hydroxybutyrate are transported by the blood to tissues other than the liver
(extrahepatic tissues), where they are converted to acetyl-CoA and oxidized in the
citric acid cycle, providing much of the energy required by tissues such as skeletal
and heart muscle and the renal cortex. The brain, which preferentially uses glucose
as fuel, can adapt to the use of acetoacetate or D-β-hydroxybutyrate under
starvation conditions, when glucose is unavailable. Ketone Bodies, Formed in the
Liver, Are Exported to Other Organs as Fuel The first step in the formation of
acetoacetate, occurring in the liver, is the enzymatic condensation of two molecules
of acetyl-CoA. In healthy people, acetone is formed in very small amounts from
acetoacetate, which is easily decarboxylated, either spontaneously or by the action
of acetoacetate decarboxylase. Because individuals with untreated diabetes produce
large quantities of acetoacetate, their blood contains significant amounts of acetone,
which is toxic. Acetone is volatile and imparts a characteristic odor to the breath,
which is sometimes useful in diagnosing diabetes.In extrahepatic tissues, D-β-
hydroxybutyrate is oxidized to acetoacetate by D-β-hydroxybutyrate dehydrogenase.
The acetoacetate is activated to its coenzyme A ester by transfer of CoA from
succinyl-CoA, an intermediate of the citric acid cycle, in a reaction catalyzed by β-
ketoacyl-CoA transferase. The acetoacetyl-CoA is then cleaved by thiolase to yield
two acetyl-CoAs, which enter the citric acid cycle. Thus the ketone bodies are used
as fuels.The production and export of ketone bodies by the liver allow continued
oxidation of fatty acids with only minimal oxidation of acetyl-CoA. When
intermediates of the citric acid cycle are being siphoned off for glucose this is simply
the reversal of the last step of β oxidation.The acetoacetyl-CoA then condenses with
acetyl-CoA to formβ-hydroxyβ-methylglutaryl-CoA (HMG-CoA), which is cleaved to
free acetoacetate and acetyl-CoA.The acetoacetate is reversibly reduced by D-β-
hydroxybutyrate dehydrogenase, a mitochondrial enzyme, to D-β-hydroxybutyrate.
This enzyme is specific for the synthesis by gluconeogenesis, for example, oxidation
of cycle intermediates slows—and so does acetyl-CoA oxidation.Moreover, the liver
contains only a limited amount of coenzyme A, and when most of it is tied up in
acetyl-CoA, β oxidation slows for want of the free coenzyme. The production and
export of ketone bodies free coenzyme A, allowing continued fatty acid oxidation.
Ketone Bodies Are Overproduced in Diabetes and during Starvation Starvation and
untreated diabetes mellitus lead to overproduction of ketone bodies. During
starvation, gluconeogenesis depletes citric acid cycle intermediates, diverting acetyl-
CoA to ketone body production. In untreated diabetes, when the insulin level is
insufficient, extrahepatic tissues cannot take up glucose efficiently from the blood,
either for fuel or for conversion to fat. Under these conditions, levels of malonyl-CoA
(the starting material for fatty acid synthesis) fall, inhibition of carnitine
acyltransferase I is relieved, and fatty acids enter mitochondria to be degraded to
acetyl CoA—which cannot pass through the citric acid cycle because cycle

VSM
intermediates have been drawn off for use as substrates in gluconeogenesis. The
resulting accumulation of acetyl-CoA accelerates the formation of ketone bodies
beyond the capacity of extrahepatic tissues to oxidize them. The increased blood
levels of acetoacetate and D-β-hydroxybutyrate lower the blood pH, causing the
condition known as acidosis. Extreme acidosis can lead to coma and in some cases
death. Ketone bodies in the blood and urine of untreated diabetics can reach
extraordinary levels—a blood concentration of 90 mg/100 mL (compared with a
normal level of <3 mg/100 mL) and urinary excretion of 5,000 mg/24 hr (compared
with a normal rate of ≤125 mg/24 hr). This condition is called ketosis.
Individuals on very low-calorie diets, using the fats stored in adipose tissue as their
major energy source, also have increased levels of ketone bodies in their blood and
urine.
Prevalence & Significance: A condition that afflicts the large and medium-sized
arteries of almost every human, at least in societies in which cholesterol-rich
foodstuffs are abundant and cheap, is atherosclerosis. This condition begins in
childhood and, in the absence of accelerating factors, develops slowly until it is
widespread in old age. However, it is accelerated by a wide variety of genetic and
environmental factors. It is characterized by localized fibrous thickenings of the
arterial wall associated with lipid-infiltrated plaques that may eventually calcify. Old
plaques are also prone to ulceration and rupture, triggering the formation of thrombi
that obstruct flow. Therefore, atherosclerosis leads to vascular insufficiency in the
limbs, abnormalities of the renal circulation, and dilations (aneurysms) and even
rupture of the aorta and other large arteries. It also leads to common severe and life-
threatening diseases of the heart and brain because of formation of intravascular
clots at the site of the plaques. Pathogenesis: The initial event in atherosclerosis is
infiltration of low-density lipoproteins (LDLs) into the subendothelial region. The
endothelium is subject to shear stress, the tendency to be pulled along or deformed
by flowing blood. This is most marked at points where the arteries branch, and this is
where the lipids accumulate to the greatest degree.The LDLs are oxidized or altered
in other ways. Thus, altered LDLs activate various components of innate immune
system including macrophages, natural antibodies, and innate effector proteins such
as C-reactive protein and complement. Altered LDLs are recognized by a family of
scavenger receptors expressed on macrophages. These scavenger receptors
mediate uptake of the oxidized LDL into macrophages and the formation of foam
cells. The foam cells form fatty streaks. The streaks appear in the aorta in the first
decade of life, in the coronary arteries in the second decade, and in the cerebral
arteries in the third and fourth decades.Oxidized LDLs have a number of deleterious
effects, including stimulation of release of cytokines and inhibition of NO production.
Vascular smooth muscle cells in the vicinity of foam cells are stimulated and move
from the media to the intima, where they proliferate, lay down collagen and other
matrix molecules, and contribute to the bulk of the lesion. Smooth muscle cells also
take up oxidized LDL and become foam cells. Lipids accumulate both intracellularly
and extracellularly. As the atherosclerotic lesions age, T cells of the immune system
as well as macrophages are attracted to them. The plaques contains a variety of cell-
damaging substances, including ozone. Growth factors and cytokines involved in cell

VSM
migration and proliferation are also produced by smooth muscle cells and endothelial
cells, and there is evidence for shear stress response elements in the flanking DNA
of relevant genes in the endothelial cells.As plaques mature, a fibrous cap forms
over them. The plaques with defective or broken caps are most prone to rupture. The
lesions alone may distort vessels to the point that they are occluded, but it is usually
rupture or ulceration of plaques that triggers thrombosis, blocking blood
flow.Oxidized LDLs inhibit NO production. If acetylcholine is infused via catheter into
normal coronary arteries, the vessels dilate; however, if it is infused when
atherosclerosis is present, the vessels constrict. This indicates that endothelial
secretion of NO is defective.
Relation to Dietary Cholesterol & Other Lipids: Transforming a monocyte into a lipid-
ingesting macrophage involves the appearance on its surface of a unique type of
oxidized LDL receptor, the scavenger receptor, and monocytes are stimulated to
produce these receptors by the action of macrophage colony-stimulating factor
secreted by endothelial cells and vascular smooth muscle cells. When oxidized LDL-
receptor complexes are formed, they are internalized and the receptors recycle to
the membrane while the lipid is stored.Because lipids are relatively insoluble, they
are transported as special lipoprotein particles that increase their solubility. Dietary
cholesterol and triglycerides are packaged in the protein-coated chylomicrons in
intestinal epithelial cells. Under the influence of lipoprotein lipase, these particles
release triglycerides to fat depots and muscles, and the resulting chylomicron
remnants are taken up by the liver. The liver also synthesizes cholesterol and
packages it with specific proteins to form very-low-density lipoproteins (VLDLs).
These lipoprotein particles enter the circulation and under the influence of lipoprotein
lipase donate triglycerides to tissues. In this way, they become cholesterol-rich
intermediate-density lipoproteins (IDLs) and low-density lipoproteins (LDLs). The
LDL supply cholesterol to the tissues. They provide all cells with the cholesterol for
production of cell membranes and other uses. They also provide most of the
cholesterol that is the precursor for all steroid hormones. Oxidized LDLs are taken up
by macrophages and smooth muscle cells in atherosclerotic lesions. On the other
hand, high-density lipoproteins (HDLs) take cholesterol from peripheral cells and
transport it to the liver where it is metabolized, keeping plasma and tissue cholesterol
low. Clinical Manifestations: In general, atherosclerosis is asymptomatic until one of
its complications develops. In coronary arteries, atherosclerotic narrowing that
reduces the lumen of a coronary artery more than 75% causes angina pectoris, the
chest pain that results when pain-producing substances accumulate in the
myocardium. Typically, the pain comes on during exertion and disappears with rest,
as the substances are washed out by the blood. When atherosclerotic lesions cause
clotting and occlusion of a coronary artery, the myocardium supplied by the artery
dies (myocardial infarction). In the cerebral circulation, arterial blockage at the site of
atherosclerotic plaques causes thrombotic strokes. In the abdominal aorta,
extensive atherosclerosis can lead to aneurysmal dilation and rupture of the vessel.
In the renal vessels, localized constriction of one or both renal arteries causes
renovascular hypertension. In the circulation to the legs, vascular insufficiency
causes intermittent claudication (fatigue and usually pain on walking that is relieved
by rest). If the circulation of a limb is severely compromised, the skin can ulcerate,

VSM
producing lesions that are slow to heal. Frank gangrene of the extremities may also
occur. Less frequently, clot formation and obstruction may occur in vessels supplying
the intestines or other parts of the body. Risk Factors: The progression of
atherosclerosis is accelerated by a wide variety of genetic and environmental factors
(risk factors).Estrogen increases cholesterol removal by the liver, and the
progression of atherosclerosis is less rapid in premenopausal women that in men.
On the other hand, large doses of estrogens increase the incidence of blood clots,
and even small doses produce a slight increase in clotting.The effect of increased
plasma levels of homocysteine and related molecules such as homocystine and
homocysteine thiolactone are associated with accelerated atherosclerosis, and the
magnitude of the plasma elevation is positively correlated with the severity of the
atherosclerosis. Homocysteine is a significant source of H2O2 and other reactive
forms of oxygen, and this may accelerate the oxidation of LDL.Homocysteine is an
intermediate in the synthesis of methionine. It is metabolized by enzymes that are
dependent on vitamin B6, vitamin B12, and folic acid. Supplementation of the diet
with these vitamins reduces plasma homocysteine, usually to normal.Lowering
plasma cholesterol and triglyceride levels and increasing plasma HDL levels slows,
and in some cases reverses, the atherosclerotic process. When dietary treatment is
not adequate, reducing conversion of mevalonate to cholesterol with statins, drugs
that inhibit hepatic 3methylglutaryl coenzyme A (HMG-CoA) reductase, the enzyme
which catalyzes this reaction, is beneficial. In cases in which there is severe
hypercholesterolemia because of congenitally defective LDL receptors, gene therapy
may be an option. The deleterious effects of smoking include endothelial damage
caused by carbon monoxide-induced hypoxia. Because of the increased shear
stress imposed on the endothelium by an elevated blood pressure, hypertension is
another important modifiable risk factor for atherosclerosis. In diabetics, there are
microvascular complications and macrovascular complications. The latter are
primarily related to atherosclerosis. There is a twofold increase in the incidence of
myocardial infarction compared with nondiabetics; severe circulatory deficiency in
the legs with gangrene is relatively common; there are more thrombotic strokes; and
renal failure is a serious problem. The nephrotic syndrome and hypothyroidism also
accelerate the progression of atherosclerosis.
70. Disturbances in digestion, absorption and transport of carbohydrates. Hyper- and
hypoglycemia.
Abnormalities of carbohydrate metabolism are usually caused by enzymopathies or
abnormal regulation. Galactosaemia Galactose is normally converted to glucose.
However, a deficiency of the enzyme galactose-1phosphate uridyltransferase or,
less commonly, uridine diphosphate galactase-4-epimerase, results in accumulation
of galactose- 1-phosphate in the blood. The transferase deficiency, inherited as an
autosomal recessive, is due in 70% of patients to a glutamine to arginine missense
mutation in Q188R. Galactose ingestion (i.e. milk) leads to inanition, failure to thrive,
vomiting, hepatomegaly and jaundice, diabetes, cataracts and developmental delay.
A lactose-free diet stops the acute toxicity but poor growth and problems with speech
and mental development still occur with the transferase deficiency. A newborn
screening programme to detect galactosaemia is used in almost all countries.

VSM
Prenatal diagnosis and diagnosis of the carrier state are possible by measurement of
the level of galactose-1- phosphate in the blood. Galactokinase deficiency also
results in galactosaemia and early cataract formation. Defects of fructose
metabolism Absorbed fructose is chiefly metabolized in the liver to lactic acid or
glucose. Three defects of metabolism in the liver and intestine occur; all are inherited
as autosomal recessivetraits: Fructosuria is due to fructokinase deficiency. It is a
benign asymptomatic condition. Hereditary fructose intolerance is due to fructose-1-
phosphate aldolase deficiency. Fructose-1phosphate accumulates after fructose
ingestion, inhibiting both glycogenolysis and gluconeogenesis, resulting in symptoms
of severe hypoglycaemia. Hepatomegaly and renal tubular defects occur but are
reversible on a fructose- and sucrose-free diet. Intelligence is normal and there is an
absence of dental caries. Hereditary fructose-1,6-diphosphatase deficiency leads to
a failure of gluconeogenesis. Infants present with hypoglycaemia, ketosis and lactic
acidosis. Dietary control can lead to normal growth. Glycogen storage diseases.
Glucose is stored in muscles and liver as glycogen. Breakingit down provides
glucose that is used locally or reaches other organs. If the breakdown of glycogen is
blocked, glycogen overloading and hypoglycemia result. This is caused by enzyme
deficiencies. Hyperglycemia or high blood sugar, is a condition in which an excessive
amount of glucose circulates in the blood plasma. This is generally a glucose level
higher than (200 mg/dl). Reference ranges for blood tests are 11.1 mmol/l, but
symptoms may not start to become noticeable until even higher values such as 250–
300 mg/dl or 15–20 mmol/l. A subject with a consistent range above 126 mg/dl or 7
mmol/l is generally held to have hyperglycemia. Chronic levels exceeding 7 mmol/l
(125 mg/dl) can produce organ damage. It is critical for patients who monitor glucose
levels at home to be aware of which units of measurement their testing kit uses.
Glucose levels are measured in either:Milligrams per decilitre (mg/dl), in the United
States and other countries (e.g., Japan, France, Egypt, Colombia); or Millimoles per
litre (mmol/l), which can be acquired by dividing (mg/dl) by factor of 18.[1] Glucose
levels vary before and after meals, and at various times of day; the definition of
"normal" varies among medical professionals. In general, the normal range for most
people (fasting adults) is about 80 to 110 mg/dl or 4 to 6 mmol/l. A subject with a
consistent range above 126 mg/dl or 7 mmol/l is generally held to have
hyperglycemia, whereas a consistent range below 70 mg/dl or 4 mmol/l is
considered hypoglycemic. In fasting adults, blood plasma glucose should not exceed
126 mg/dL. Sustained higher levels of blood sugar cause damage to the blood
vessels and to the organs they supply, leading to the complications of diabetes.
Chronic hyperglycemia can be measured via the HbA1c test. The definition of acute
hyperglycemia varies by study, with mmol/l levels from 8 to 15. Temporary
hyperglycemia is often benign and asymptomatic. Blood glucose levels can rise well
above normal for significant periods without producing any permanent effects or
symptoms. However, chronic hyperglycemia at levels more than slightly above
normal can produce a very wide variety of serious complications over a period of
years, including kidney damage, neurological damage, cardiovascular damage,
damage to the retina or damage to feet and legs. Diabetic neuropathy may be a
result of long-term hyperglycemia.

VSM
In diabetes mellitus (by far the most common cause of chronic hyperglycemia),
treatment aims at maintaining blood glucose at a level as close to normal as
possible, in order to avoid these serious long-term complications. Acute
hyperglycemia involving glucose levels that are extremely high is a medical
emergency and can rapidly produce serious complications (such as fluid loss
through osmotic diuresis). It is most often seen in persons who have uncontrolled
insulin-dependent diabetes.
The following symptoms may be associated with acute or chronic hyperglycemia,
with the first three composing the classic hyperglycemic triad: Polyphagia - frequent
hunger, especially pronounced hunger Polydipsia - frequent thirst, especially
excessive thirst Polyuria - frequent urination Blurred vision Fatigue (sleepiness).
Weight loss Poor wound healing (cuts, scrapes, etc.) Dry mouth
Seizures Frequent hunger without other symptoms can also indicate that blood sugar
levels are too low. This may occur when people who have diabetes take too much
oral hypoglycemic medication or insulin for the amount of food they eat. The
resulting drop in blood sugar level to below the normal range prompts a hunger
response. This hunger is not usually as pronounced as in Type I diabetes, especially
the juvenile onset form, but it makes the prescription of oral hypoglycemic
medication difficult to manage. Polydipsia and polyuria occur when blood glucose
levels rise high enough to result in excretion of excess glucose via the kidneys
(glycosuria), producing osmotic diuresis. Causes
The following conditions can also cause hyperglycemia in the absence of diabetes.
1) Dysfunction of the thyroid, adrenal, and pituitary glands 2) Numerous diseases of
the pancreas 3) Severe increases in blood glucose may be seen in sepsis and
certain infections 4) Intracranial diseases (frequently overlooked) can also cause
hyperglycemia. Encephalitis, brain tumors (especially those located near the pituitary
gland), brain bleeds, and meningitis are prime examples. 5) Mid to high blood sugar
levels are often seen in convulsions and terminal stages of many diseases.
Prolonged, major surgeries can temporarily increase glucose levels. Certain forms of
severe stress and Physical trauma can increase levels for a brief time as well yet
rarely exceeds 120 mg/dl. Hypoglycemia ( <2.8mmol/L) Causes Glucose
underproduction (hormone deficiency) Enzyme abnormalities Damage to the liver
Drugs or chemicals (salicylates, alcohol) İncreased insulin activity If the amount of
glucose supplied by the blood falls, the brain is one of the first organs affected. In
most people, subtle reduction of mental efficiency can be observed when the
glucose falls below 65 mg/dl (3.6 mM). Impairment of action and judgment usually
becomes obvious below 40 mg/dl (2.2 mM). Seizures may occur as the glucose falls
further. As blood glucose levels fall below 10 mg/dl (0.55 mM), most neurons
become electrically silent and nonfunctional, resulting in coma. These brain effects
are collectively referred to as neuroglycopenia.The importance of an adequate
supply of glucose to the brain is apparent from the number of nervous, hormonal and
metabolic responses to a falling glucose level. Most of these are defensive or
adaptive, tending to raise the blood sugar via glycogenolysis and gluconeogenesis or
provide alternative fuels. If the blood sugar level falls too low the liver converts a
storage of glycogen into glucose and releases it into the bloodstream, to prevent the

VSM
person going into a diabetic coma, for a short period of time. Brief or mild
hypoglycemia produces no lasting effects on the brain, though it can temporarily alter
brain responses to additional hypoglycemia. Prolonged, severe hypoglycemia can
produce lasting damage of a wide range. This can include impairment of cognitive
function, motor control, or even consciousness. The likelihood of permanent brain
damage from any given instance of severe hypoglycemia is difficult to estimate, and
depends on a multitude of factors such as age, recent blood and brain glucose
experience, concurrent problems such as hypoxia, and availability of alternative
fuels. It has been frequently found that those Type 1 diabetics found "dead in bed" in
the morning after suspected severe hypoglycemia had some underlying coronary
pathology that led to an induced fatal heart attack. Recently, several of these
individuals found "dead in bed" were wearing Continuous Glucose Monitors, which
provided a history of glucose levels prior to the fatal event.
71. Disturbances in protein digestion and absorption. Changes in serum protein
levels.
Protein intolerance occurs when your body is unable to digest certain proteins found
in that food, according to the American College of Gastroenterology. For example, if
you're intolerant to milk proteins, your body lacks the appropriate enzymes needed to
break down the proteins. Proteins are too complex for the body to absorb them. They
require enzymes to break them down into a more simple form that the body can
absorb. If you cannot digest certain proteins, inflammation and swelling will occur in
your intestines. This can lead to gas, diarrhea, stomach pain, cramping, bloating and
nausea.Most food allergies are the result of an immune system malfunction from the
proteins found in the food. The immune system mistakes the proteins in the food as
a dangerous substance when they are actually safe. This mistake triggers the
immune system to create immunoglobulin E antibodies that attempt to fight off the
proteins, according to the Mayo Clinic. The creation of IgE antibodies causes cells in
your soft tissue to produce histamine. Histamine released in your intestines leads to
inflammation. The digestive tract is one of the most common parts of the body
affected by a food allergy. Absorption disorders, also sometimes called
malabsorption syndromes, are characterized by problems digesting or absorbing
substances (called nutrients) in the diet. Nutrients include vitamins, minerals,
carbohydrates (e.g., sugars, starches), fats, and proteins. The term, "absorption
disorder," does not refer to a specific disease—in most cases, the disorders are
related to another medical condition.Digestion, which occurs in the gastrointestinal
(GI) tract or digestive system, is the process of breaking down food into a form that
can be absorbed into the bloodstream and used by the body. The digestive tract
consists of the following organs: Mouth Esophagus (muscular tube that carries food
to the stomach) Stomach Small intestine (small bowel) Large intestine (large bowel
or colon) Biliary system (i.e., liver, pancreas, gallbladder, bile ducts) During
digestion, most nutrients in food are absorbed in the small intestine. The lining of the
small intestine (called the mucosa) contains folds, creases, and finger-like
projections (called villi and microvilli) that increase the surface area of the intestinal
lining, providing a larger surface with which to absorb nutrients. The mucosa also
contains special cells that increase the absorption of nutrients.

VSM
Types of Absorption Disorders
There are a number of different types of absorption disorders, most of which result in
a decreased ability to absorb nutrients (i.e., malabsorption). Disorders that increase
the absorption of certain nutrients include hemochromatosis (increased iron
absorption; also called iron overload disease) and Wilson's disease (increased
copper absorption).Common absorption disorders include celiac disease (caused by
intolerance to gluten, a protein in wheat, barley, and rye) and lactose intolerance.
Lactose intolerance is an inability to break down a sugar in dairy products (lactose).
It is caused by a lack of a certain enzyme (called lactase) that helps convert lactose
into a form that can be used by the body (i.e., dextrose and galactose). Lactose
intolerance also is called primary or secondary lactase deficiency. Primary lactase
deficiency is a common condition that develops over time as the body produces less
lactase. This process begins around 2 years of age, but symptoms usually develop
much later in life. Secondary lactase deficiency occurs when damage to the small
intestine reduces the production of lactase. Conditions that can cause secondary
lactase deficiency include celiac disease and inflammatory bowel disease (e.g.,
Crohn's disease, ulcerative colitis). Other types of absorption disorders include
tropical sprue and Whipple's disease. Tropical sprue is more common in tropical and
subtropical areas of the world, including the Caribbean and Southeast Asia. The
exact cause for the condition is unknown, but it may be related to an infection that
damages the lining of the small intestine. Symptoms include anemia, diarrhea,
weight loss, and malnutrition. Tropical sprue is treated with antibiotics.
72. Disturbances in protein metabolism.
Metabolic disorders, or inborn errors of metabolism, are inherited genetic diseases in
which a baby is unable to metabolize a specific nutrient. Without appropriate dietary
management, the unmetabolized nutrient and metabolic by-products will accumulate
in the body, and may result in adverse effects that inhibit normal growth and
development. However, many infants who start treatment early in life can grow and
develop normally. Mead Johnson Nutrition is a pioneer in the field of specialized
nutrition for metabolic disorders and has been a leader in the development of
science-based metabolic products for more than 50 years. Protein Metabolism
Disorders Protein is a key constituent of most foods we eat, including meat, beans,
milk products and grains. Infants with protein metabolism disorders cannot drink
human milk because it also contains proteins and amino acids that cannot be
metabolized. Infants with protein metabolism disorders are unable to metabolize
certain amino acids and require specialized formulas without the offending amino
acid, allowing the baby to receive essential nutrients for growth.
Examples of protein metabolism disorders include: Phenylketonuria (PKU) Maple
Syrup Urine Disease (MSUD) Tyrosinemia Homocystinuria A Closer Look at
Phenylketonuria (PKU) Phenylketonuria, often referred to as PKU, is one of the most
common protein metabolism disorders. Normally, the human body metabolizes the
essential amino acid phenylalanine into tyrosine. Infants with PKU do not have the
enzyme needed to make this conversion. Consequently, their bodies have excess
amounts of phenylalanine and low tyrosine levels. The following symptoms may

VSM
result from untreated PKU: Lethargy Light pigment Eczema Intellectual disability
Seizures Hyperactivity Dietary management is critical for individuals with PKU to
assure normal growth and development, as well as to support normal neurocognitive
function. Fortunately, PKU can be diagnosed early in life through genetic disorders
screenings, which are mandatory in many countries. Once diagnosed, the dietary
management of PKU can start right away.
PKU management involves a highly restrictive diet that minimizes the amount of
phenylalanine consumed and regular monitoring by healthcare professionals
experienced in the dietary management of metabolic diseases. For infants with PKU,
special phenylalanine-free formulas act as their primary source of protein. Regular
clinic visits allow monitoring phenylalanine in the blood and the growth of the infant
to ensure that the dietary management is successful.
73. Disturbances in purine metabolism. Gout.
Gout develops in the setting of excessive stores of uric acid in the form of
monosodium urate. Uric acid is an end-stage by-product of purine metabolism.
Humans remove uric acid primarily by renal excretion. When excretion is insufficient
to maintain serum urate levels below the saturation level of 6.8 mg/dL (with some
variability depending on temperature and pH), hyperuricemia may develop, and urate
can crystallize and deposit in soft tissues. Purine metabolism Ninety percent of
patients with gout develop excess urate stores due to an inability to excrete sufficient
amounts of normally produced uric acid in the urine (underexcretion). The remaining
patients either overconsume purines or produce excessive amounts of uric acid
endogenously (overproduction). In rare cases, overproduction of uric acid is primary,
due to a genetic disorder. These disorders include hypoxanthine-guanine
phosphoribosyltransferase deficiency (Lesch-Nyhan syndrome), glucose-6-
phosphatase deficiency (von Gierke disease), fructose 1-phosphate aldolase
deficiency, and PP-ribose-P synthetase variants. It has been suggested that a link
exists between several autosomal dominant disorders and the development of gout.
However, there has not been a specific genetic marker for those predisposed to
developing gout. Overproduction of uric acid may also occur in disorders that cause
high cell turnover with release of purines, which are present in high concentration in
cell nuclei. These disorders include myeloproliferative and lymphoproliferative
disorders, psoriasis, hemolytic anemias, pernicious anemia, and ineffective
erythropoiesis (as in B-12 deficiency). Cell lysis from chemotherapy for
malignancies, especially those of the hematopoietic or lymphatic systems, can raise
uric acid levels, as can excessive exercise and obesity. Common causes of
secondary gout due to underexcretion of uric acid include renal insufficiency, lead
nephropathy (saturnine gout), starvation or dehydration, hypothyroidism,
hyperparathyroidism, drugs, and chronic ethanol (especially beer and hard liquor)
abuse. These disorders should be identified and corrected, if possible.
Comorbidities, including hypertension, diabetes, renal insufficiency,
hypertriglyceridemia, hypercholesterolemia, diabetes, obesity, and early menopause,
are associated with a higher incidence of gout. Foods that are rich in purines include
anchovies, sardines, sweetbread, kidney, liver, and meat extracts. Consumption of
fructose-rich foods and beverages (eg, those sweetened with high-fructose corn

VSM
syrup) are associated with an increased risk of gout in both men and women.
Pathophysiology Gout can be considered a disorder of metabolism that allows uric
acid/urate to accumulate in blood and tissues. When tissues become
supersaturated, the urate salts precipitate, forming crystals. In addition, the crystals
also are less soluble under acid conditions, so any condition predisposing to acidosis
also precipitates urate crystals. Urate initially precipitates in the form of needlelike
crystals. Many conditions and drugs have been associated with an increase in
plasma (and subsequent synovial) urate levels. A genetic predisposition for the
disease exists. The CPP crystals that produce pseudogout are produced by
nucleoside triphosphate pyrophosphohydrolase (NTPPPH), a catalytic enzyme found
in vesicles that develop within osteoarthritic cartilage. A genetic predisposition exists
for the condition, but any process that leads to osteoarthritis also can be associated
with subsequent pseudogout. Although the presence of urate crystals in the soft
tissues and synovial tissues is a prerequisite for a gouty attack, the fact that these
crystals can also be found in synovial fluid in the absence of joint inflammation
suggests that the mere presence of intrasynovial urate crystals is not sufficient to
cause flares of gouty arthritis. One explanation for this may lie in the observation
that clumps or microtophi of highly negatively charged and reactive urate crystals are
normally coated with serum proteins (apolipoprotein [apo] E or apo B) that physically
inhibit the binding of the crystals to cell receptors. A gout attack may be triggered by
either a release of uncoated crystals (eg, due to partial dissolution of a microtophus
caused by changing serum urate levels) or precipitation of crystals in a
supersaturated microenvironment (eg, release of urate due to cellular damage).
From either source, naked urate crystals are then believed to interact with
intracellular and surface receptors of local dendritic cells and macrophages, serving
as a danger signal to activate the innate immune system. This interaction may be
enhanced by immunoglobulin G (IgG) binding. Triggering of these receptors,
including Toll-like receptors, NALP3 inflammasomes, and the triggering receptors
expressed on myeloid cells (TREMs) by MSU, results in the production of interleukin
(IL)–1, which in turn initiates the production of a cascade of pro-inflammatory
cytokines, including IL-6, IL-8, neutrophil chemotactic factors, and tumor necrosis
factor (TNF)–alpha.[16, 17] Neutrophil phagocytosis leads to another burst of
inflammatory mediator production. Subsidence of an acute gout attack is due to
multiple mechanisms, including the clearance of damaged neutrophils, recoating of
urate crystals, and the production of anti-inflammatory cytokines including, IL-1RA,
IL10, and transforming growth factor (TGF)–beta.
74. Alterations of kidney structure and function in disease. Manifestations of altered
kidney function.
Renal damage can impair renal perfusion as well as glomerular and/or tubular
functions (!A). In addition, abnormal urine composition can lead to precipitations
(urolithiasis) that inhibit the free flow of urine. Abnormal renal function can be caused
by reduced renal excretion of useless or harmful substances (e.g., uric acid, urea,
creatinine, vanadate, foreign substances [xenobiotics], and so-called uremic toxins)
whose plasma concentration then rises correspondingly.Reduced renal excretory
function affects the kidney’s decisive contribution to the regulation of the metabolism

VSM
of water, electrolytes, minerals, and acid–base balance.The capacity of the kidney to
regulate the composition of extracellular fluid is a function of volume, which, per unit
time, is under the control of its epithelia. For substances that are not secreted by
tubular cells, the controlled volume corresponds to the glomerular filtration rate.
An abnormality of prerenal factors can, despite unimpaired tubular transport, raise
the excretion of the affected substance via an increase in its plasma concentration
and the amount filtered. Thus, glycosuria may occur even when renal transport of
glucose is normal,if the plasma concentration of glucose is higher than its renal
threshold, as is the case in diabetes mellitus. Similarly, impaired breakdown of amino
acids leads to overflow aminoaciduria.
Abnormalities of Urinary Concentration:Depending on requirements, the kidney can
normally excrete hypotonic (< 100mosm/L) or hypertonic (> 1200 mosmol/L) urine.
Concentration and dilution of urine are in the first instance the result of processes in
the thick ascending loop of Henle (pars ascendens) which transports NaCl (, red
arrow) to the interstitial space of the renal medulla without water (blue arrow) being
able to follow. The tubular fluid becomes hypotonic (50–100 mosmol/L) by the time it
passes the ascending part, while the interstitial space becomes hypertonic.A protein-
low diet impairs the concentrating ability of the kidney because of reduced
contribution of urea to the concentrating mechanism.
Abnormalities of Glomerular Function: Reduced hydraulic conductivity or areduced
filtration surface decreases the GFR.No filtration equilibrium can be achieved; as
aresult of the reduced increase in, Peff ultimately rises. But this does not
compensate for the reduced conductivity. Constriction of the vas afferens when
systemic blood pressure remains constant reduces the filtration pressure and thus
the proportion of filtered plasma water (filtration fraction = GFR/RPF). At the same
time the renal blood flow and the GFR fall because of the increased resistance.
Constriction of the vas efferens (!A4) raises the effective filtration pressure and thus
also GFR/RPF. Simultaneously it reduces glomerular perfusion and thus GFR at any
given filtration fraction. The constriction of the vas efferens (e.g., on infusion of
angiotensin II) orobstruction of venous flow (e.g., by renalvein thrombosis) can thus
ultimately reduce GFR.
The function of the glomeruli is to produce an adequate GFR, i.e., the volume of
plasma water that is controlled by the renal epithelium. The selective permeability of
this filter ensures the formation of a nearly protein-free filtrate. As all of the blood
flowing through the kidney must pass through the glomerular vessels, the resistance
of these vessels also determines RPF. The GFR is determined by the effective
filtration pressure (Peff), the hydraulic conductivity (Kf), and the filtering surface (F):
GFR =Kf • F • Peff Masugi’s nephritis, caused by autoantibodies against the
basement membrane, is much less common than immune complex nephritis. The
local inflammation initially results in hyperemia, accumulation of neutrophils
(exudative phase), and damage to the often markedly thickened basement
membrane. It is common for endothelial, mesangial, or capsular epithelial cells to
proliferate and ultimately for excess mesangial matrix to form (sclerosing). The
glomeruli may also be damaged without any local inflammation, for example, by

VSM
deposition of amyloid in amyloidosis, by a high concentration of filtrable proteins in
plasma (e.g., in multiple myeloma), by high pressure in the glomerular capillaries
(e.g., in arterial hypertension, renal vein thrombosis, venous back pressure in right
heart failure, or hyperfiltration in diabetic nephropathy) as well as by reduced
perfusion (e.g., in atherosclerosis, arteriosclerosis).İn glomerulonephritis, resistance
in thevasa afferentia and efferentia is increased and the RPF is reduced despite
filtration pressure usually being high. The reduced hydraulic conductivity prevents
filtration equilibrium being achieved and lowers GFR. The reduced renal perfusion
stimulates the release of renin which, via angiotensin and aldosterone, raises blood
pressure. In addition, the development of hypertension is aided by reduced excretion
of NaCl and H2O, brought about by the decrease in GFR. Selective permeability is
lost by damage to the glomerular filter, thus leading to proteinuria and edema
.Damage to the kidney can, for example, destroy erythropoietin-producing cells and
thus result in the development of anemia.
Interstitial Nephritis: The term interstitial nephritis is applied to inflammatory changes
in the kidney if the inflammation does not originate in the glomeruli. Renal tissue is
infiltrated by inflammatory cells (especially granulocytes) and the inflammation can
lead to local destruction of renal tissue. Nephrotic syndrome: If proteinuria,
hypoproteinemia, and peripheral edema occur together, this is termed nephrotic
syndrome. As the lipoproteins are not filtered even if the filter is damaged, but
hypoproteinemia stimulates the formation of lipoproteins in the liver, hyperlipidemia
results and thus also hypercholesterolemia. It remains debatable whether a loss of
glomerular lipoprotein lipase contributes to the effect.
75. Glomerulonephritis.
Glomerulonephritis is a type of kidney disease in which the part of your kidneys that
helps filter waste and fluids from the blood is damaged. Types : 1-Non Proliferative
This is characterised by absence of increase in the number of cells (lack of
hypercellularity) in the glomeruli. They usually cause nephrotic syndrome. This
includes the following types: Minimal change GN (also known as Minimal Change
Disease) This form of GN causes 78.4% of nephrotic syndrome in children, but only
20% in adults. As the name indicates, there are no changes visible on simple light
microscopy, but on electron microscopy there is fusion of podocytes (supportive cells
in the glomerulus). Immunohistochemistry staining is negative. Focal Segmental
Glomerulosclerosis (FSGS) FSGS may be primary or secondary to reflux
nephropathy, Alport syndrome, heroin abuse or HIV. FSGS presents as a nephrotic
syndrome with varying degrees of impaired renal function (seen as a rising serum
creatinine, hypertension). As the name suggests, only certain foci of glomeruli within
the kidney are affected, and then only a segment of an individual glomerulus.
Membranous glomerulonephritis Membranous glomerulonephritis (MGN), a relatively
common type of glomerulonephritis in adults, frequently produces a mixed nephrotic
and nephritic picture. Its cause is usually unknown, but may be associated with
cancers of the lung and bowel, infection such as hepatitis and malaria, drugs
including penicillamine, and connective tissue diseases such as systemic lupus
erythematosus. Individuals with cerebral shunts are at risk of developing shunt
nephritis, which frequently produces MGN.

VSM
2- Proliferative This type is characterised by increased number of cells in the
glomerulus (hypercellular). Usually present as a nephritic syndrome and usually
progress to end-stage renal failure (ESRF) over weeks to years (depending on type).
IgA nephropathy (Berger's disease) IgA nephropathy is the most common type of
glomerulonephritis in adults worldwide. It usually presents as macroscopic
haematuria (visibly bloody urine). It occasionally presents as a nephrotic syndrome.
It often affects young males within days (24-48hrs) after an upper respiratory tract or
gastrointestinal infection. Microscopic examination of biopsy specimens shows
increased number of mesangial cells with increased matrix (the 'cement' that holds
everything together). Immuno-staining is positive for immunoglobulin A deposits
within the matrix.
Post-infectious Post-infectious glomerulonephritis can occur after essentially any
infection, but classically occurs after infection with Streptococcus pyogenes. It
typically occurs 10–14 days after a skin or pharyngeal infection with this bacterium.
Membranoproliferative/mesangiocapillary GN:This may be primary, or secondary to
SLE, viral hepatitis, or hypocomplementemia. One sees 'hypercellular and
hyperlobular' glomeruli due to proliferation of both cells and the matrix within the
mesangium. Usually presents with a combined nephritic-nephrotic picture, with
inevitable progression to end stage renal failure. The primary form consists of two
types: Type 1 (Classical and Alternative Complement activation) Type 2 (also known
as Dense Deposit Disease) Alternative Complement activation only. C3 Nephritic
Factor stabilizes C3 convertase, leading to Hypocomplementemia. Unlike Type 1, no
IgG is detected.
Rapidly progressive Glomerolonephritis(Crescentic GN) has a poor prognosis, with
rapid progression to kidney failure over weeks. Steroid therapy is sometimes
used.[5] Any of the above types of GN can be rapidly progressive. Additionally two
further causes present as solely RPGN. One is Goodpasture syndrome, an
autoimmune disease whereby antibodies are directed against basal membrane
antigens found in the kidney and lungs. As well as kidney failure, patients have
hemoptysis (cough up blood). The second cause is vasculitic disorders such as
Wegener granulomatosis and polyarteritis. There is a lack of immune deposits on
staining, but blood tests are positive for ANCA antibody.
Causes, incidence, and risk factors Glomerulonephritis may be caused by specific
problems with the body's immune system. Often, the precise cause of
glomerulonephritis is unknown.Damage to the glomeruli causes blood and protein to
be lost in the urine.The condition may develop quickly, with loss of kidney function
occurring over weeks and months (called rapidly progressive glomerulonephritis).In
about a quarter of people with chronic glomerulonephritis there is no history of
kidney disease and the disorder first appears as chronic renal failure. The following
increase your risk of developing this condition: History of cancer Blood or lymphatic
system disorders Exposure to hydrocarbon solvents Infections such as strep
infections, viruses, heart infections,or abscesses Diabetes Many conditions are
known to cause or increase the risk for glomerulonephritis, including: Focal
segmental glomerulosclerosis Goodpasture syndrome Membranoproliferative GN

VSM
IgA nephropathy Lupus nephritis or Henoch-Schonlein purpura Anti-glomerular
basement membrane antibody disease Blood vessel diseases such as vasculitis or
polyarteritis Common symptoms of glomerulonephritis are: Blood in the urine (dark,
rust-colored, or brown urine) Foamy urine Swelling (edema) of the face, eyes,
ankles, feet, legs, or abdomen Symptoms that may also appear include the following:
Abdominal pain Cough.
76. Nephrotic syndrome.
Certain glomerular diseases virtually always produce the nephrotic syndrome. In
addition, many other forms of primary and secondary.
Pathophysiology. The manifestations of the nephroticsyndrome include: 1. Massive
proteinuria, with the daily loss of 3.5 gm or more of protein (less in children) 2.
Hypoalbuminemia, with plasma albumin levels less than 3 gm/dL 3. Generalized
edema 4. Hyperlipidemia and lipiduria
The various components of nephrotic syndrome bear a logical relationship to one
another. The initial event is a derangement in glomerular capillary walls resulting in
increased permeability to plasma proteins. It will be remembered that the glomerular
capillary wall, with its endothelium, GBM, and visceral epithelial cells, acts as a size
and charge barrier through which the glomerular filtrate must pass. Increased
permeability resulting from either structural or physicochemical alterations allows
protein to escape from the plasma into the glomerular filtrate. Massive proteinuria
results.
The heavy proteinuria leads to depletion of serum albumin levels below the
compensatory synthetic abilities of the liver,with consequent hypoalbuminemia and a
reversed albuminglobulin ratio. Increased renal catabolism of filtered albumin also
contributes to the hypoalbuminemia. The generalized edema is, in turn, the
consequence of the loss of colloid osmotic pressure of the blood with subsequent
accumulation of fluid in the interstitial tissues. There is also sodium and water
retention, which aggravates the edema. This appears to be due to several factors,
including compensatory secretion of aldosterone, mediated by the
hypovolemiaenhanced antidiuretic hormone secretion; stimulation of the sympathetic
system; and a reduction in the secretion of natriuretic factors such as atrial
peptides.The largest proportion of protein lost in the urine is albumin, but globulins
are also excreted in some diseases. The ratio of low- to high-molecular-weight
proteins in the urine in various cases of nephrotic syndrome is a manifestation of the
selectivity of proteinuria. The genesis of the hyperlipidemia is complex. Most patients
have increased blood levels of cholesterol, triglyceride, very-low-density lipoprotein,
low-density lipoprotein, Lp(a) lipoprotein, and apoprotein, and there is a decrease in
highdensity lipoprotein concentration in some patients. These defects seem to be
due in part to increased synthesis of lipoproteins in the liver, abnormal transport of
circulating lipid particles, and decreased catabolism. Lipiduria follows the
hyperlipidemia because not only albumin molecules but also lipoproteins leak across
the glomerular capillary wall. The lipid appears in the urine either as free fat or as
oval fat bodies, representing lipoprotein resorbed by tubular epithelial cellsand then
shed along with the degenerated cells.These patients are particularly vulnerable to

VSM
infection, especially with staphylococci and pneumococci. This vulnerability could be
related to loss of immunoglobulins or lowmolecular- weight complement components
in the urine.
Thrombotic and thromboerrtbolic complications are also common in nephrotic
syndrome, owing in part to loss of anticoagulant factors (e.g., antithrombin III) and
antiplasmin activity through the leaky glomerulus. Renal vein thrombosis, once
thought to be a cause of nephrotic syndrome, is most often a consequence of this
hypercoagulable state. Causes. The relative frequencies of the several causes of the
nephrotic syndrome vary according to age and geography. In children younger than
17 years the nephrotic syndrome is almost always caused by a lesion primary to the
kidney; whereas among adults, it may often be associated with a systemic
diseaseThe most frequent systemic causes of the nephrotic syndrome are diabetes,
amyloidosis, and SLE. The most important of the primary glomerular lesions are
minimal change disease, membranous glomerulopathy, and focal segmental
glomerulosclerosis. The first is most common in children in North America, the
second is most common in older adults, but focal segmental glomerulosclerosis
occurs at all ages." These three lesions, as well as a fourth, less common disorder,
membranoproliferative glomerulonephritis, are discussed individually in the following
sections. Other primary causes, the various proliferative glomerulonephritides,
frequently present as a mixed syndrome with nephrotic and nephritic features.
77. Acute renal failure.
Obstruction of the urinary tract, for example,by urinary stones can stop urinary
excretion, even though the kidney remainsintact—at least at first.In hemolysis and in
the destruction of muscle cells (myolysis) hemoglobin or myoglobin, respectively, is
filtered through the glomeruli and precipitated in the acidic tubular lumen, especially
because their tubular concentration is increased by fluid absorption. The resulting
leads to urine formation being interrupted. Renal function can also cease as a result
of rapidly progressing renal diseases (e.g., glomerulonephritis) or toxic damage tothe
kidney.Loss of blood and fluid impairs renal perfusion and glomerular filtration
because in centralization of the circulation. the kidney is treated like a peripheral
organ, i.e.,sympathetic activation produces renal vascular constriction via "-
adrenoceptors. The result is acute ischemic renal failure. Energy deficiency impairs
Na+/K+-ATPase;the resulting increase in intracellular concentration of Na+ also
causes, via the 3Na+/ Ca2+ exchanger, a rise in intracellular Ca2+ concentration
(!p.10,12) and thus vasoconstriction.
– The ischemia promotes the release of renin both primarily and via an increased
NaCl supply in the macula densa (reduced Na+ absorption in the ascending tubules)
and thus the intrarenal formation of angiotensin II, which has a vasoconstrictor
action.
– If there is a lack of energy supply, adenosine is freed from ATP. It acts on the
kidney—in contrast to the other organs—as a marked vasoconstrictor.Intravascular
stasis (“sludge”) that cannot be flushed out of the network between renal medulla
and cortex, even if the perfusion pressure rises. In the first three days of acute renal
failure no urine (anuria) or only a little volume of poorly concentrated urine (oliguria)

VSM
is excreted as a rule (oliguric phase). However, urinary volume alone is a very poor
indicator of the functional capacity of the kidney in acute renal failure, because the
tubular transport processes are severely restricted and the reabsorption of filtered
fluid is thus reduced.Despite normal-looking urine volume, renal excretion of all
those substances that must normally be excreted in the urine may be markedly
impaired. In this case determination of the plasma and urine creatinine concentration
provides information on the true functional state of the kidneys.Recovery after the
oliguric phase will lead to a polyuric phase characterized by the gradual increase of
the GFR while the reabsorption function of the epithelial nephron is still impaired
(salt-losing kidney). If the renal tubules are damaged (e.g., by heavy metals),
polyuric renal failure occurs as a primary response, i.e., large volumes of urine are
excreted despite a markedly decreased GFR.The dangers of acute renal failure lie in
the inability of the kidney to regulate the water and electrolyte balance. The main
threat in the oliguric phase is hyperhydration (especially with infusion of large
volumes of fluid) and hyperkalemia (especially with the simultaneous release of
intracellular K+, as in burns, contusions, hemolysis, etc.). In the polyuric phase the
loss of Na+, water, HCO3 and especially of K+ may be so large as to be life-
threatening.
78. Chronic renal failure. Uremia.
A number of renal diseases can ultimately lead to the destruction of renal tissue.If
the residual renal tissue is not in a position to adequately fulfill its tasks, the picture
of renal failure evolves.Reduced renal excretion is particularly significant.The
decreased GFR leads to an inversely proportional rise in the plasma level of
creatinine. The plasma concentration of reabsorbed substances also rises, but less
markedly, because renal tubular reabsorption is impaired in renal failure. The
reabsorption of Na+ and water is inhibited in renal failure by a variety of factors,such
as natriuretic hormone, PTH, and vanadate. The reduced reabsorption of Na+ in the
proximal tubules also directly or indirectly decreases the reabsorption of
othersubstances, such as phosphate, uricacid,HCO3, Ca2+, urea, glucose, and
amino acids.The reabsorption of phosphate is also inhibited by PTH.
It is probably the disruption in renal water and electrolyte excretion that is
responsible, at least partially, for the development of most of symptoms of chronic
renal failure. Excess volume and the changed electrolyte concentrations lead to
edemas, hypertension, osteomalacia, acidosis, pruritus, and arthritis, either directly
or via the activation of hormones. Also, abnormalities of the excitatory cells
(polyneuropathy, confusion, coma, seizures,cerebral edemas), of gastrointestinal
function (nausea, peptic ulcer, diarrhea), and of blood cells (hemolysis, abnormal
leukocyte function, abnormal blood clotting) are due to this.The reduced
consumption of fatty acids by the kidney contributes to hyperlipidemia,while reduced
gluconeogenesis favors the development of hypoglycemia..While uric acid can be
precipitated at high concentrations, especially in the joints, and thus cause gout ,
sufficiently high concentrations of uric acid are only rarely
achieved in renal failure. The role of reduced elimination of so-called uremia toxins
(e.g.,acetone, 2,3-butyleneglycol, guanidinosuccinic acid, methylguanidine, indoles,

VSM
phenols,aliphatic and aromatic amines, etc.) as well as of so-called middle molecules
(lipids or peptides with a molecular weight of 300– 2000 Da) in producing the
symptoms of renal failure remains the subject of considerable debate. High
concentrations of urea can destabilize proteins and bring about cell shrinkage. But its
effect is partly canceled by the cellular uptake of stabilizing osmolytes.
Uremia is a clinical syndrome associated with fluid, electrolyte, and hormone
imbalances and metabolic abnormalities, which develop in parallel with deterioration
of renal function. The term uremia, which literally means urine in the blood, was first
used by Piorry to describe the clinical condition associated with renal failure. Uremia
more commonly develops with chronic renal failure (CRF) or the later stages of
chronic kidney disease (CKD), but it also may occur with acute renal failure (ARF) if
loss of renal function is rapid. As yet, no single uremic toxin has been identified that
accounts for all of the clinical manifestations of uremia. Toxins, such as parathyroid
hormone (PTH), beta2-microglobulin, polyamines, advanced glycosylation end
products, and other middle molecules, are thought to contribute to the clinical
syndrome. Causes : Anemia Anemia-induced fatigue is thought to be one of the
major contributors to the uremic syndrome. Erythropoietin (EPO), a hormone
necessary for red blood cell production in bone marrow, is produced by peritubular
cells in the kidney in response to hypoxia. Coagulopathy Acidosis Hyperkalemia
Calcium, parathyroid, and vitamin D abnormalities Endocrine abnormalities
Cardiovascular abnormalities Cardiovascular abnormalities, including uremic
pericarditis, pericardial effusions, calcium and phosphate deposition–associated
worsening of underlying valvular disorders, and uremic suppression of myocardial
contractility, are common in patients with CRF. Left ventricular hypertrophy is a
common disorder found in approximately 75% of patients who have not yet
undergone dialysis. Malnutrition
79. Renal stones (nephrolithiasis).
Stones may form at any level in the urinary tract, but most arise in the kidney.
Urolithiasis is a frequent clinical problem, affecting 5% to 10% of Americans in their
lifetime.?' Men are affected more often than women are, and the peak age at onset is
between 20 and 30 years. Familial and hereditary predisposition to stone formation
has long been known. Many of the inborn errors of metabolism, such as gout,
cystinuria, and primary hyperoxaluria, provide good examples of hereditary disease
characterized by excessive production and excretion of stone-forming substances.
Cause and Pathogenesis. There are four main types of calculi 11 '' 1 ' (1) most
stones (about 70%) are calcium containing, composed largely of calcium oxalate or
calcium oxalate mixed with calcium phosphate; (2) another 15% are so-called triple
stones or struvite stones, composed of magnesium ammonium phosphate; (3) 5% to
10% are uric acid stones; and (4) 1 % to 2% are made up of cystine. An organic
matrix of mucoprotein, making up 1% to 5% of the stone by weight, is present in all
calculi. Although there aremany causes for the initiation and propagation of stones,
the most important determinant is an increased urinary concentration of the stones'
constituents, such that it exceeds their solubility in urine (supersaturation). A low
urine volume in some metabolically normal patients may also favor supersaturation.
Calcium oxalate stones are associated in about 5% of patients with both

VSM
hypercalcemia and hypercalciuria, caused by hyperparathyroidism, diffuse bone
disease, sarcoidosis, and other hypercalcemic states. About 55% have
hypercalciuria without hypercalcemia. This is caused by several factors, including
hyperabsorption of calcium from the intestine (absorptive hypercalciuria), an intrinsic
impairment in renal tubular reabsorption of calcium (renal hypercalciuria), or
idiopathic fasting hypercalciuria with normal parathyroid function.
Magnesium ammonium phosphate stones are formed largely after infections by
urea-splitting bacteria (e.g., Proteus and some staphylococci), which convert urea to
ammonia. The resultant alkaline urine causes the precipitation of magnesium
ammonium phosphate salts.
Uric acid stones are common in patients with hyperuricemia, such as gout, and
diseases involving rapid cell turnover, such as the leukemias. However, more than
half of all patients with orate calculi have neither hyperuricemia nor increased urinary
excretion of uric acid. Cystine stones are caused by genetic defects in the renal
reabsorption
of amino acids, including cystine, leading to cystinuria. Stones form at low urinary
pH. It can therefore be appreciated that increased concentration of stone
constituents, changes in urinary pH, decreased urine volume, and the presence of
bacteria influence the formation of calculi.
Morphology: Stones are unilateral in about 80% of patients. The favored sites for
their formation are within the renal calyces and pelves (Fig. 20-57) and in the
bladder.
Clinical Course. Stones are of importance when they obstruct urinary flow or produce
ulceration and bleeding. They may be present without producing any symptoms or
significant renal damage. In general, smaller stones are most hazardous, because
they may pass into the ureters, producing pain referred to as colic (one of the most
intense forms of pain) as well as ureteral obstruction. Larger stones cannot enter the
ureters and are more likely to remain silent within the renal pelvis.
80. Disturbances in water-electrolyte balance. Edema.
The movement of body fluids between the ICF and ECF compartments occurs at the
cell membrane and depends on regulation of ECF water and sodium. Water provides
approximately 90% to 93% of the volume of body fluids and sodium salts
approximately 90% to 95% of extracellular solutes. Normally, equivalent changes in
sodium and water are such that the volume and osmolality of ECF are maintained
within a normal range. Because it is the concentration of sodium (in milligrams per
liter) that controls ECF osmolality, changes in sodium are usually accompanied by
proportionate changes in water volume. Protection of the circulatory volume can be
viewed as the single most important characteristic of body fluid homeostasis. In
situations in which multiple physiologic variables are threatened simultaneously, the
homeostatic response protects the vascular volume even at the expense of
aggravating another electrolyte disorder. Two mechanisms protect the ECF (and
vascular fluid) volume:

VSM
(1) alterations in hemodynamic variables such as vasoconstriction and an increase in
heart rate, and
(2) alterations in sodium and water balance. Tachycardia, peripheral arterial
vasoconstriction, and venoconstriction occur within minutes of external fluid losses,
whereas salt and water retention take hours to become effective. Alterations of
sodium and water balance can be divided into two main categories:
(1) isotonic contraction or expansion of ECF volume, and (2) hypotonic dilution
(dilutional hyponatremia) or hypertonic concentration (hypernatremia) of extracellular
sodium brought about by changes in extracellular water.
Isotonic disorders usually are confined to the ECF compartment producing a
contraction (fluid volume deficit) or expansion (fluid volume excess) of the interstitial
and vascular fluids.
Regulation of Water Balance: Total body water (TBW) varies with gender and
weight. These differences can be explained by differences in body fat, which is
essentially water free. In men, body water approximates 60% of body weight during
young adulthood and decreases to approximately 50% in old age; in young women,
it is approximately 50%, and in elderly women, approximately 40%. Obesity
produces further decreases in body water, sometimes reducing these levels to
values as low as 30% to 40% of body weight in adults. Infants and young children
have a greater water content than adults. In addition to having proportionately more
body water than adults have, infants have relatively more water in their ECF
compartment and a greater water turnover. Infants have more than half of their TBW
in the ECF compartment, whereas adults have only approximately one third.
Because ECF is more readily lost from the body, infants are more vulnerable to fluid
deficit than are older children and adults. As an infant grows older, TBW decreases,
and by the second year of life, the percentages and distribution of body water
approach those of an adult.
Edema is the medical term for excessive fluid accumulation within the interstitial
space (between cells) or within the cavities of the body. This does not include fluid
within the bladder or in the gastrointestinal tract (gut). Edema may be localized,
affecting only one organ or area of the body or it can be generalized where it affects
many areas or the whole body simultaneously. Edema Pathophysiology -Fluid in the
body exists within cells (intracellular fluid), within the tissue space between cells
(interstitial fluid), within the blood vessels (blood) or lymphatic vessels (lymph or
lymphatic fluid). This fluid is not just water but there are also cells, nutrients,
electrolytes and waste products existing with water in these areas. -Two forces are
responsible for maintaining the fluid in specific areas or ‘pulling’ and ‘pushing’ fluid
into other areas. These forces are known as hydrostatic pressure and osmotic
pressure. Hydrostatic pressure is the force that pushes fluid from an area of high
pressure to low pressure. Osmotic pressure is the force that draws fluid from an area
of low electrolyte concentration to one of a higher electrolyte concentration. Fluid
within our blood vessels are at a higher pressure than the fluid in the tissue spaces.
This is due to the pumping heart that pushes the blood with force within the vessels.
Blood does not just ooze out of the vessels unless the vessel wall becomes

VSM
permeable and allows it to exit. -Fluid within the tissue spaces also have a
hydrostatic force but this is usually smaller than the pressure within the vessels so
very little fluid flow from the tissues spaces into the vessel due to hydrostatic
pressure. Usually fluid from the tissue spaces enters the blood vessel due to a
difference in osmotic pressure. -Fluid is pushed away from an area of high
hydrostatic pressure but is pulled into an area of high osmotic pressure. Therefore
the blood vessels which have a higher hydrostatic and a higher osmotic pressure will
push out some fluid into the tissue spaces and draw other fluids from the tissue
spaces. This allows for the exchange of nutrients, gases and wastes. The lymphatic
system also plays an essential part here as the interstitial fluid is pulled from the
tissue spaces into the lymphatic vessels which then empties into the blood vessels.
Causes of Edema Edema will occur under these circumstances : Increased
hydrostatic pressure will push fluid out of the vessels into tissue spaces. This results
in edema. • Reduced osmotic pressure within the vessels will not pull fluid from the
tissue spaces into the vessel. The fluid accumulates within the tissue space and
results in edema.
• Fluid retention (water retention) where there is excessive fluid within the blood
vessel and tissue spaces. If the body is not able to pass out this excess fluid, it will
be retained within the tissue spaces thereby resulting in edema.
• Increased vascular permeability is when blood vessel wall allows fluid to pass out
of the blood vessel unabated. Fluid from the tissue spaces are not drawn into the
blood vessel fast enough and fluid remains in the tissue space thereby resulting in
edema.
.Lymphatic obstruction is where the lymph vessels are blocked at some point and the
interstitial fluid cannot be drained from the tissue spaces. Fluid accumulates in the
tissue space and the result is edema. Types of Edema There are two types of
edema that can be identified upon examination. 1. Pitting edema is the swelling of a
body part where an indentation will persist after pressure is applied to the area. This
indentation will slowly disappear over time. 2. Non-pitting edema is where there is
swelling of a body part with no indentation upon applying pressure.
81. Disturbances in mineral metabolism – sodium, potassium, chlorides.
Sodium Sodium is the most abundant cation in the body, averaging approximately 60
mEq/kg of body weight. Most of the body’s sodium is in the ECF compartment (135
to 145 mEq/L), with only a small amount (10 to 14 mEq/L) located in the ICF
compartment. The resting cell membrane is relatively impermeable to sodium.
Sodium that enters the cell is transported out of the cell against an electrochemical
gradient by the energy-dependent Na+/K+-ATPase membrane pump. Sodium
functions mainly in regulating extracellular and vascular volume. As the major cation
in the ECF compartment, Na+ and its attendant anions (Cl− and HCO3−) account for
the osmotic activity in the ECF. Because sodium is part of the sodium bicarbonate
molecule, it is important in regulating acid-base balance. As a current-carrying ion,
Na+ contributes to the function of the nervous system and other excitable tissue.

VSM
82. Disturbances in calcium and phosphate metabolism.
Disorders of calcium metabolism occur when the body has too little or too much
calcium. The serum level of calcium is closely regulated within a fairly limited range
in the human body. In a healthy physiology, extracellular calcium levels are
maintained within a tight range through the actions of parathyroid hormone, vitamin
D and the calcium sensing receptor. Disorders in calcium metabolism can lead to
hypocalcemia, decreased plasma levels of calcium or hypercalcemia, elevated
plasma calcium levels. Hypocalcemia Hypocalcemia is common and can occur
unnoticed with no symptoms or, in severe cases, can have dramatic symptoms and
be life-threatening . Hypocalcemia can be parathyroid related or vitamin D related.
Parathyroid related hypocalcemia includes post-surgical hypoparathyroidism,
inherited hypoparathyroidism, pseudohypoparathyroidism, and pseudo-
pseudohypoparathyroidism [1]. Postsurgical hypoparathyroidism is the most
common form, and can be temporary (due to suppression of tissue after removal of a
malfunctioning gland) or permanent, if all parathyroid tissue has been removed [1].
Inherited hypoparathyroidism is rare and is due to a mutation in the calcium sensing
receptor. Pseudohypoparathyroidism is maternally inherited and is categorized by
hypocalcemia and hyperphosphatemia. Finally, pseudo-pseudohypoparathyroidism
is paternally inherited. Patients display normal parathyroid hormone action in the
kidney, but exhibit altered parathyroid hormone action in the bone [1]. Vitamin D
related hypocalcemia may be associated with a lack of vitamin D in the diet, a lack of
sufficient UV exposure, or disturbances in renal function. Low vitamin D in the body
can lead to a lack of calcium absorption and secondary hyperparathyroidism
(hypocalcemia and raised parathyroid hormone) [1].Symptoms of hypocalcemia
include numbness in fingers and toes, muscle cramps, irritability, impaired mental
capacity and muscle twitching . Hypercalcemia Hypercalcemia is suspected to occur
in approximately 1 in 500 adults in the general adult population . Like hypocalcemia,
hypercalcemia can be non-severe and present with no symptoms, or it may be
severe, with life-threatening symptoms. Hypercalcemia is most commonly caused by
hyperparathyroidism and by malignancy, and less commonly by vitamin D
intoxication, familial hypocalciuric hypercalcemia and by sarcoidosis [2].
Hyperparathyroidism occurs most commonly in postmenopausal women.
Hyperparathyroidism can be caused by a tumor, or adenoma, in the parathyroid
gland or by increased levels of parathyroid hormone due to hypocalcemia [2].
Approximately 10% of cancer sufferers experience hypercalcemia due to malignancy
[2]. Hypercalcemia occurs most commonly in breast cancer, lymphoma, prostate
cancer, thyroid cancer, lung cancer, myeloma, and colon cancer [2]. It may be
caused by secretion of parathyroid hormonerelated peptide by the tumor (which has
the same action as parathyroid hormone), or may be a result of direct invasion of the
bone, causing calcium release. Symptoms of hypercalcemia include anorexia,
nausea, vomiting, constipation, abdominal pain, lethargy, depression, confusion,
polyuria, polydipsia and generalized aches and pains. A variety of genetic diseases
and disorders due to therapeutic agents affect phosphate homeostasis. Not
surprisingly, since the kidney is the primary regulatory site for phosphate
homeostasis, aberrant phosphate metabolism results most commonly from altered
renal Pi handling. Moreover, the majority of the primary diseases are phosphate

VSM
losing disorders in which renal Pi wasting and hypophosphatemia predominate and
osteomalacia and rickets are characteristic. Osteomalacia and rickets are disorders
of calcification characterized by defects of bone mineralization in adults and bone
and cartilage mineralization during growth. In osteomalacia, there is a failure to
normally mineralize the newly formed organic matrix (osteoid) of bone. In rickets, a
disease of children, there is not only abnormal mineralization of bone but defective
cartilage growth plate calcification at the epiphyses as well. Apoptosis of
chondrocytes in the hypertrophic zone is reduced, typically resulting in an expanded
hypertrophic zone, delayed mineralization and vascularization of the calcification
front, with an overall appearance of a widened and disorganized growth plate.
83. Basic disturbances in acid-base equilibrium. Acidosis.
cidosis is an increased acidity in the blood. (i.e., an increased hydronium ion
concentration). If not further qualified, it usually refers to acidity of the blood
plasma.Acidosis is said to occur when arterial pH falls below 7.35 (except in the
fetus- see below), while its counterpart (alkalosis) occurs at a pH over 7.45. Arterial
blood gas analysis and other tests are required to separate the main causes. The
term acidemia describes the state of low blood pH, while acidosis is used to describe
the processes leading to these states. Nevertheless, the terms are sometimes used
interchangeably. The distinction may be relevant where a patient has factors causing
both acidosis and alkalosis, wherein the relative severity of both determines whether
the result is a high or a low pH. The rate of cellular metabolic activity affects and, at
the same time, is affected by the pH of the body fluids. In mammals, the normal pH
of arterial blood lies between 7.35 and 7.50 depending on the species (e.g., healthy
human-arterial blood pH varies between 7.35 and 7.45). Blood pH values compatible
with life in mammals are limited to a pH range between 6.8 and 7.8. Changes in the
pH of arterial blood (and therefore the extracellular fluid) outside this range result in
irreversible cell damage. Metabolic acidosis Metabolic acidosis may result from
increased production of metabolic acids or disturbances in the ability to excrete acid
via the kidneys. Renal acidosis is associated with an accumulation of urea and
creatinine as well as metabolic acid residues of protein catabolism. An increase in
the production of other acids may also produce metabolic acidosis. For example,
lactic acidosis may occur from 1) severe (PaO2 <36mm Hg) hypoxemia causing a
fall in the rate of oxygen diffusion from arterial blood to tissues, or 2) hypoperfusion
(e.g., hypovolemic shock) causing an inadequate blood delivery of oxygen to tissues.
A rise in lactate out of proportion to the level of pyruvate, e.g., in mixed venous
blood, is termed "excess lactate", and may also be an indicator of fermention due to
anaerobic metabolism occurring in muscle cells, as seen during strenuous exercise.
Once oxygenation is restored, the acidosis clears quickly. Another example of
increased production of acids occurs in starvation and diabetic acidosis. It is due to
the accumulation of ketoacids (ketosis) and reflects a severe shift from glycolysis to
lipolysis for energy needs.Acid consumption from poisoning, elevated levels of iron in
the blood, and chronically decreased production of bicarbonate may also produce
metabolic acidosis.Metabolic acidosis is compensated for in the lungs, as increased
exhalation of carbon dioxide promptly shifts the buffering equation to reduce
metabolic acid. This is a result of stimulation to chemoreceptors, which increases

VSM
alveolar ventilation, leading to respiratory compensation, otherwise known as
Kussmaul breathing (a specific type of hyperventilation). Should this situation persist,
the patient is at risk for exhaustion leading to respiratory failure.Mutations to the V-
ATPase 'a4' or 'B1' isoforms result in distal renal tubular acidosis, a condition that
leads to metabolic acidosis, in some cases with sensorineural deafness.
Arterial blood gases will indicate low pH, low blood HCO3, and normal or low
PaCO2. In addition to arterial blood gas, an anion gap can also differentiate between
possible causes. The Henderson-Hasselbalch equation is useful for calculating blood
pH, because blood is a buffer solution. The amount of metabolic acid accumulating
can also be quantitated by using buffer base deviation, a derivative estimate of the
metabolic as opposed to the respiratory component. In hypovolemic shock for
example, approximately 50% of the metabolic acid accumulation is lactic acid, which
disappears as blood flow and oxygen debt are corrected. Treatment of
uncompensated metabolic acidosis is focused upon correcting the underlying
problem. When metabolic acidosis is severe and can no longer be compensated for
adequately by the lungs, neutralizing the acidosis with infusions of bicarbonate may
be required.
Fetal metabolic acidemia In the fetus, the normal range differs based on which
umbilical vessel is sampled (umbilical vein pH is normally 7.25 to 7.45; umbilical
artery pH is normally 7.18 to 7.38).[4] Fetal metabolic acidemia is defined as an
umbilical vessel pH of less than 7.20 and a base excess of less than -8] Respiratory
acidosis Respiratory acidosis results from a build-up of carbon dioxide in the blood
(hypercapnia) due to hypoventilation. It is most often caused by pulmonary
problems, although head injuries, drugs (especially anaesthetics and sedatives), and
brain tumors can cause this acidemia. Pneumothorax, emphysema, chronic
bronchitis, asthma, severe pneumonia, and aspiration are among the most frequent
causes. It can also occur as a compensatory response to chronic metabolic
alkalosis. One key to distinguish between respiratory and metabolic acidosis is that
in respiratory acidosis, the CO2 is increased while the bicarbonate is either normal
(uncompensated) or increased (compensated). Compensation occurs if respiratory
acidosis is present, and a chronic phase is entered with partial buffering of the
acidosis through renal bicarbonate retention. However, in cases where chronic
illnesses that compromise pulmonary function persist, such as latestage emphysema
and certain types of muscular dystrophy, compensatory mechanisms will be unable
to reverse this acidotic condition. As metabolic bicarbonate production becomes
exhausted, and extraeneous bicarbonate infusion can no longer reverse the extreme
buildup of carbon dioxide associated with uncompensated respiratory acidosis,
mechanical ventilation will usually be applied. Fetal respiratory acidemia In the fetus,
the normal range differs based on which umbilical vessel is sampled (umbilical vein
pH is normally 7.25 to 7.45; umbilical artery pH is normally 7.20 to 7.38). In the fetus,
the lungs are not used for ventilation. Instead, the placenta performs ventilatory
functions (gas exchange). Fetal respiratory acidemia is defined as an umbilical
vessel pH of less than 7.20 and an umbilical artery PCO2of 66 or higher or umbilical
vein PCO2 of 50 or higher.

VSM
84. Basic disturbances in acid-base equilibrium. Alkalosis.
Metabolic activities of the body require the precise regulation of acid-base balance,
which is reflected in the pH of extracellular fluids. Membrane excitability, enzyme
systems, and chemical reactions depend on acid-base balance being regulated
within a narrow physiologic range to function in an optimal way.Normally, the
concentration of body acids and bases is regulated so that the pH of extracellular
body fluids is maintained within a very narrow range of 7.35 to 7.45. This balance is
maintained through mechanisms that generate, buffer, and eliminate acids and
bases.
An acid is a molecule that can release a hydrogen ion (H+), and a base is a molecule
that can accept or combine with an hydrogen ion.1,2 When an acid (HA) is added to
water, it dissociates reversibly to form H+ and anions (A−); for example, HA H+ +
A−.
Acids are continuously generated as byproducts of metabolic processes.
Physiologically, these acids fall into two groups: the volatile acid H2CO3 and all
other nonvolatile or fixed acids.
The difference between the two types of acids arises because H2CO3 is in
equilibrium with the volatile gas CO2, which leaves the body by way of the lungs.
The fixed acids (e.g., sulfuric,hydrochloric, phosphoric) are nonvolatile and are not
eliminated by the lungs. Instead, they are buffered by body proteins or extracellular
buffers, such as HCO3−, and then excreted by the kidney.Carbon Dioxide and
Bicarbonate Production: Body metabolism results in the production of approximately
15,000 mmol of CO2 each day. Carbon dioxide is transported in the circulation in
three forms: attached to hemoglobin, as dissolved CO2 in the plasma, and as
HCO3−. Although CO2 is not an acid, a small percentage of the gas combines with
water in the bloodstream to form H2CO3.The reaction that generates H2CO3 from
CO2 and water is catalyzed by an enzyme called carbonic anhydrase, which is
present in large quantities in red blood cells, renal tubular cells, and other tissues in
the body. Production of Noncarbonic Acids and Bases: The metabolism of dietary
proteins and other substances results in the generation of noncarbonic acids and
bases. Oxidation of the sulfurcontaining amino acids (e.g., methionine, cysteine,
cystine) results in the production of sulfuric acid. Oxidation of arginine and lysine
produces hydrochloric acid, and oxidation of phosphorus-containing nucleic acids
yields phosphoric acid. Incomplete oxidation of glucose results in the formation of
lactic acid, and incomplete oxidation of fats results in the production of ketoacids.
The major source of base is the metabolism of amino acids such as aspartate and
glutamate and the metabolism of certain organic anions (e.g., citrate, lactate,
acetate).The plasma pH can be calculated using an equation called the Henderson-
Hasselbalch equation. This equation uses the negative logarithm of the dissociation
constant and the logarithm of the HCO3− to CO2 (HCO3−/CO2) ratio to calculate
pH. Intracellular and Extracellular Buffer Systems: The moment-by-moment
regulation of pH depends on intracellular and extracellular buffer systems. A buffer
system consists of a weak acid and the base salt of that acid or of a weak base and
its acid salt. In the process of preventing large changes in pH, the system trades a

VSM
strong acid for a weak acid or a strong base for a weak base.The two major buffer
systems that protect the pH of body fluids are proteins and the bicarbonate buffer
system. These buffer systems are immediately available to combine with excess
acids or bases and prevent large changes in pH from occurring during the time it
takes for respiratory and renal mechanisms to become effective. Bone also
represents an important site for buffering of acids and bases. One consequence of
bone buffering is the release of calcium from bone and increased renal excretion of
calcium. In addition to causing demineralization of bone, it also predisposes to
kidney stones. Protein Buffer Systems: Proteins are the largest buffer system in the
body. Proteins are amphoteric, meaning that they can function as acids or bases.
They contain many ionizable groups that can release or bind H+. The protein buffers
are largely located in cells, and H+ ions and CO2 diffuse across cell membranes for
buffering by intracellular proteins. Albumin and plasma globulins are the major
protein buffers in the vascular compartment. Bicarbonate Buffer System: The
bicarbonate buffer system uses H2CO3 as its weak acid and a bicarbonate salt such
as sodium bicarbonate (NaHCO3) as its weak base. It substitutes the weak H2CO3
for a strong acid such as hydrochloric acid (HCl + NaHCO3 H2CO3 + NaCl) or the
weak bicarbonate base for a strong base such as sodium hydroxide (NaOH +
H2CO3 NaHCO3 + H2O). Metabolism provides an ample supply of CO2, which
can replace any H2CO3 that is lost when excess base is added, and CO2 can be
readily eliminated when excess acid is added. Likewise, the kidney can conserve or
form new HCO3− when excess acid is added, and it can excrete HCO3− when
excess base is added. Plasma Potassium–Hydrogen Exchange: Potassium ions (K+)
and H+ ions interact in important ways in the regulation of acid-base balance. Both
ions are positively charged, and both ions move freely between the intracellular and
extracellular compartments. In situations of acidosis, excess H+ ions move into the
intracellular compartment for buffering. When this happens, another cation, in this
case the K+ ion, must leave the cell and move into the extracellular fluid. When
extracellular potassium levels fall, K+ ions move out of the cell and are replaced by
H+ ions. Thus, alterations in extracellular potassium levels can affect acid-base
balance, and changes in acid-base balance can influence extracellular potassium
levels. Potassium shifts tend to be more pronounced in metabolic acidosis than
respiratory acidosis. Also, metabolic acidosis caused by an accumulation of
nonorganic acids (e.g., HCl that occurs in diarrhea, phosphoric acid that occurs in
renal failure) produces a greater increase in extracellular potassium than does
acidosis caused by an accumulation of organic acids (e.g., lactic acid, ketoacids).
Metabolic Alkalosis
Metabolic alkalosis is a systemic disorder caused by an increase in pH due to a
primary excess of plasma HCO3− ions. It can be caused by a loss of H+ ions, net
gain in HCO3− ions, or loss of Cl− ions in excess of HCO3− ions.
Causes: Most of the body’s plasma HCO3− is obtained from three sources: from
CO2 that is produced during metabolic processes, from reabsorption of filtered
HCO3−, or from generation of new HCO3 − by the kidney. Usually, HCO3−
production and renal reabsorption are balanced in a manner that prevents alkalosis
from occurring. The proximal tubule reabsorbs 99.9% of the filtered HCO3−. When

VSM
the plasma levels of HCO3− rise above the threshold for tubular reabsorption, the
excess is excreted in the urine. Many of the conditions that increase plasma HCO3 −
also raise the level for HCO3 − reabsorption; thus, an increase in HCO3 −
contributes not only to the generation of metabolic alkalosis but also to its
maintenance.
Metabolic alkalosis involves both the factors that generate the loss of H+ or gain of
HCO3− ions and those that maintain it by interfering with excretion of the excess
HCO3−. Factors that serve to maintain metabolic alkalosis include extracellular fluid
volume contraction accompanied by hypokalemia and hypochloremia and
mineralocorticoid (aldosterone) excess.
- Excess Alkali Intake: Excessive alkali ingestion, as in the use of bicarbonate-
containing antacids or NaHCO3 administration during cardiopulmonary resuscitation,
can cause metabolic alkalosis.
Other sources of alkali intake are acetate in hyperalimentation solutions, lactate in
parenteral solutions such as Ringer’s lactate, and citrate used in blood transfusions.
A condition called the milkalkali syndrome may develop in persons who consume
excessive amounts of milk along with an antacid such as calcium carbonate. The
most common cause at present is the use of calcium carbonate as a phosphate
buffer in persons with renal failure. - Hydrogen, Chloride, and Potassium Ion Loss
Associated With Bicarbonate Ion: Vomiting, removal of gastric secretions through
use of nasogastric suction, and low potassium levels resulting from diuretic therapy
are the most common causes of metabolic alkalosis in hospitalized patients. Gastric
secretions contain high concentrations of HCl and lesser concentrations of
potassium chloride (KCL). Under normal conditions, each 1 mEq of H+ ion that is
secreted into the stomach generates 1 mEq of plasma HCO3−. Because the entry of
acid into the duodenum stimulates an equal amount of pancreatic HCO3− secretion,
the increase in plasma HCO3− concentration is usually transient, and pH returns to
normal within a matter of hours. However, loss of H+ and Cl− ions from the stomach
due to vomiting or gastric suction stimulates continued production of gastric acid and
thus the addition of more bicarbonate into the blood. - Maintenance of Metabolic
Alkalasis by Volume Contraction, Hypokalemia, and Hypochloremia: Vomiting also
results in the loss of water, sodium, and potassium. The resultant volume depletion
and hypokalemia maintain the generated metabolic alkalosis by increasing the renal
reabsorption of HCO3− ion. Administration of diuretics (e.g., loop and thiazide
diuretics) is often associated with metabolic alkalosis. Loop diuretics decrease Na+,
K+, and Cl− reabsorption, leading to volume contraction and hypokalemia. The
thiazide diuretics increase renal K+ loss, leading to increased HCO3− reabsorption.
Extracellular volume depletion is one of the most important factors affecting HCO3−
reabsorption in the proximal tubule. A decrease in extracellular fluid volume activates
the renin-angiotensinaldosterone system, which increases Na+ reabsorption as a
means of maintaining extracellular fluid volume. The reabsorption of Na+ requires
concomitant anion reabsorption; because there is a Cl− deficit, HCO3− is
reabsorbed along with Na+. Hypokalemia is a potent stimulus for H+ secretion and
HCO3 − reabsorption. In hypokalemia, K+ moves out of the tubular cell into the

VSM
blood and is replaced by the H+ ion. This results in intracellular acidosis and
increased HCO3− reabsorption.
85. Primary & secondary hyperparathyroidism.
Primary hyperparathyroidism is one of the most common endocrine disorders, and it
is an important cause of hypercalcemia. The frequency of the various parathyroid
lesions underlying the hyperfunction is as follows: ■ Adenoma: 75% to 80% ■
Primary hyperplasia (diffuse or nodular): 10% to 15% ■ Parathyroid carcinoma: less
than 5% Primary hyperparathyroidism is usually a disease of adults and is more
common in women than in men by a ratio of nearly 3:1. Studies have begun to
provide a molecular understanding of the pathogenesis of primary
hyperparathyroidism. In more than 95% of cases, the disorder is caused by sporadic
parathyroid adenomas or sporadic hyperplasia . Although familial syndromes are a
distant second, they have provided a unique insight into the pathogenesis of primary
hyperparathyroidism. The genetic syndromes associated with familial primary
hyperparathyroidism include the following:
■ Multiple endocrine neoplasia-1 (MEN-1): The MEN1 gene on chromosome 1 ig13
is a tumor suppressor gene inactivated in a variety of MEN-1-related parathyroid
lesions, including parathyroid adenomas and hyperplasia.
■ Multiple endocrine neoplasia-2 (MEN-2): The MEN-2 syndrome is caused by
activating mutations in the tyrosine kinase receptor, RET, on chromosome IOq.
Morphology. The morphologic changes seen in primary hyperparathyroidism include
those in the parathyroid glands as well as those in other organs affected by elevated
levels of calcium. Parathyroid adenomas are almost always solitary and, similar to
the normal parathyroid glands, may lie in close proximity to the thyroid gland or in an
ectopic site (e.g., the mediastinum).
Clinical Cases: Primary hyperparathyroidism presents in one of two general ways:
(1) It may be asymptomatic and be identified after a routine chemistry profile, or (2)
patients may have the classic clinical manifestations of primary
hyperparathyroidism." Asymptomatic Hyperparathyroidism. Because serum calcium
levels are routinely assessed in the work-up of most patients who need blood tests
for unrelated conditions, clinically silent hyperparathyroidism is often detected early.
SECONDARY HYPERPARATHYROIDISM Secondary hyperparathyroidism is
caused by any condition associated with a chronic depression in the serum calcium
level because low serum calcium leads to compensatory overactivity of the
parathyroid glands. ; ' Renal failure is by far the most common cause of secondary
hyperparathyroidism, although a number of other diseases, including inadequate
dietary intake of calcium, steatorrhea, and vitamin ll deficiency, may also cause this
disorder. The mechanisms by which chronic renal failure induces secondary
hyperparathyroidism are complex and not fully understood. Chronic renal
insufficiency is associated with decreased phosphate excretion, which in turn results
in hyperphosphatemia. The elevated serum phosphate levels directly depress serum
calcium levels and thereby stimulate parathyroid gland activity. In addition, loss of
renal substance reduces the availability of a-1-hydroxylase necessary for the

VSM
synthesis of the active form of vitamin D, which in turn reduces intestinal absorption
of calcium.
Morphology.The parathyroid glands in secondary hyperparathyroidism are
hyperplastic.As in the case of primary hyperplasia,the degree of glandular
enlargement is not neccessarily symetric.
Clinical Course. The clinical features of secondary hyperparathyroidism are usually
dominated by those associated with chronic renal failure. Bone abnormalities (renal
osteodystrophy) and other changes associated with PTH excess are, in general, less
severe than are those seen in primary hyperparathyroidism.
■ Familial hypocalciuric hypercalcemia (FHH) is an autosomal- dominant disorder
characterized by enhanced parathyroid function due to decreased sensitivity to
extracellular calcium. Mutations in the parathyroid calciumsensing receptor gene
(CASK) on chromosome 3q are a primary cause for this disorder.'' Patients with
homozygous CASR mutations present in the neonatal period with severe
hyperparathyroidism. CASR mutations have not been described in sporadic
parathyroid tumors.
86. Hypoparathyroidism.
Hypoparathyroidism Hypoparathyroidism is far less common than is
hyperparathyroidism. There are many possible causes of deficient PTH secretion
resulting in hypoparathyroidism: ■ Surgically induced hypoparathyroidism occurs
with inadvertent removal of all the parathyroid glands during thyroidectomy, excision
of the parathyroid glands in the mistaken belief that they are lymph nodes during
radical neck dissection for some form of malignant disease, or removal of too large a
proportion of parathyroid tissue in the treatment of primary hyperparathyroidism. ■
Congenital absence of all glands, as in certain developmental abnormalities, such as
thymic aplasia and cardiac defects ■ Familial hypoparathyroidism is often
associated with chronic mucocutaneous candidiasis and primary adrenal
insufficiency; this syndrome is known as autoimmune polyendocrine syndrome type
1 (APS1) and is caused by mutations
The syndrome typically presents in childhood with the onset of candidiasis, followed
several years later by hypoparathyroidism and then adrenal insufficiency during
adolescence. APS1 is discussed further in the section on adrenal glands.
■ Idiopathic hypoparathyroidism most likely represents an autoimmune disease with
isolated atrophy of the glands. Sixty per cent of the patients with this disorder have
autoantibodies directed against the calcium-sensing receptor (CASK) in the
parathyroid gland The major clinical manifestations of hypoparathyroidism are
referable to hypocalcemia and are related to the severity and chronicity of the
hypocalcemia. ■ The hallmark of hypocalcemia is tetany, which is characterized by
neuromuscular irritability, resulting from decreased serum ionized calcium
concentration. ■ Mental status changes ■ Intracranial manifestations ■ Ocular
disease results in calcification of the lens leading to cataract formation. ■
Cardiovascular manifestations ■ Dental abnormalities

VSM
87. Osteoporosis and osteomalacia.
Osteoporosis
Osteoporosis is a disease characterized by increased porosity of the skeleton
resulting from reduced bone mass. The associated structural changes predispose
the bone to fracture. Thedisorder may be localized to a certain bone or region, as in
disuse osteoporosis of a limb, or may involve the entire skeleton, as a manifestation
of a metabolic bone disease. Generalized osteoporosis may be primary, or
secondary to a large variety of conditions . The most common forms of osteoporosis
are senile and postmenopausal osteoporosis. In these disorders, the critical loss of
bone mass makes the skeleton vulnerable to fractures. Pathogenesis. Peak bone
mass is achieved during young adulthood. Its magnitude is determined largely by
hereditary factors, especially the vitamin D receptor allele that is expressed, as well
as the genes for collagen 1 A I, estrogen receptor, and insulin-like growth factor 1
and its binding protein.'Physical activity, muscle strength, diet, and hormonal
state,however, all contribute.
Age-related changes in bone cells and matrix have a strong impact on bone
metabolism. Osteoblasts from elderly individuals have reduced replicative and
biosynthetic potential when compared with osteoblasts from younger individuals."
Also, proteins bound to the extracellular matrix (such as growth factors, which are
mitogenic to osteoprogenitor cells and stimulate osteoblastic synthetic activity) lose
their biologic potency over time.
■ Reduced physical activity increases the rate of bone loss in experimental animals
and humans because mechanical forces are important stimuli for normal bone
remodeling.
■ Genetic factors are also important, as noted previously.The type of vitamin D
receptor molecule that is inherited accounts for approximately 75% of the maximal
peak mass achieved. ■ The body 's calcium nutritional state is important. It has
been shown that adolescent girls (but not boys) have insufficient calcium intake in
the diet. ■ Hormonal influences. In the decade after menopause, yearly reductions in
bone mass may reach up to 2% of cortical bone and 9% of cancellous bone.
Morphology. The entire skeleton is affected in postmenopausal and senile
osteoporosis , but certain regions tend to be more severely involved than others. In
postmenopausal osteoporosis, the Osteoporotic vertebral body (right) shortened by
compression fractures, compared with a normal vertebral body.The osteoporotic
vertebra has a characteristic loss of horizontal trabeculae and thickened vertical
trabeculae. increase in osteoclast activity affects mainly bones or portions of bones
that have increased surface area, such as the cancellous compartment of vertebral
bodies.
Osteomalacia results from defective bone mineralisation. This is a result of a lack of
one or more of the factors necessary for osteogenesis: namely, a normal
extracellular concentration of calcium and phosphate and a normal pH at the site of
calcification. Normal mineralisation depends on interdependent factors that supply
adequate calcium and phosphate to the bones. Vitamin D maintains calcium and

VSM
phosphate homeostasis through its actions on the GI tract, the kidneys, bone, and
the parathyroid glands. Vitamin D is obtained from the diet, or it can be produced
from a sterol precursor (7-dehydrocholesterol) in the skin following exposure to UV-B
light.
Sequential hydroxylation of vitamin D is required to produce the metabolically active
form of vitamin D. Hydroxylation occurs first in the liver and then in the kidneys and
produces vitamin D 1,25(OH). Dysfunction in any of these metabolic steps results in
osteomalacia and secondary hyperparathyroidism in adults.
88. Diabetes mellitus. Pathology & pathogenesis.
Diabetes mellitus (DM) is not a single disease entity, but rather agroup of metabolic
disorders sharing the common underlying feature of hyperglycemia. Hyperglycemia
in diabetes results from defects in insulin secretion, insulin action, or, most
commonly, both. The chronic hyperglycemia and attendant metabolic dysregulation
may be associated with secondary damage in multiple organ systems, especially the
kidneys, eyes, nerves, and blood vessels.
CLASSIFICATION Although all forms of diabetes mellitus share hyperglycemia as a
common feature, the pathogenic processes involved in the development of
hyperglycemia vary widely. The previous classification schemes of diabetes mellitus
were based on the age at onset of the disease or on the mode of therapy; in
contrast, the recently revised classification reflects our greater understanding of the
pathogenesis of each variant."The vast majority of cases of diabetes fall into one of
two broad classes: Type 1 diabetes is characterized by an absolute deficiency of
insulin caused by pancreatic B-cell destruction. It accounts for approximately 10% of
all cases. Type 2 diabetes is caused by a combination of peripheral resistance to
insulin action and an inadequate secretory response by the pancreatic B-
cells("relative insulin deficiency" ). Approximately 80% to 90% of patients have type
2 diabetes. NORMAL INSULIN PHYSIOLOGY Normal glucose homeostasis is tightly
regulated by three interrelated processes: glucose production in the liver; glucose
uptake and utilization by peripheral tissues, chiefly skeletal muscle; and actions of
insulin and counter-regulatory hormones, including glucagon, on glucose. Insulin and
glucagon have opposing regulatory effects on glucose homeostasis. During fasting
states, low insulin and high glucagon levels facilitate hepatic gluconeogenesis and
glycogenolysis (glycogen breakdown) while decreasing glycogen synthesis, thereby
preventing hypoglycemia. Thus, fasting plasma glucose levels are determined
primarily by hepatic glucose output. Following a meal, insulin levels rise and
glucagon levels fall in response to the large glucose load. Insulin promotes glucose
uptake and utilization in tissues (discussed later). The skeletal muscle is the major
insulinresponsive site for postprandial glucose utilization, and is critical for preventing
hyperglycemia and maintaining glucose homeostasis. Regulation of Insulin Release
The insulin gene is expressed in the B cells of the pancreatic islets (Fig. 24–27).
Preproinsulin is synthesized in the rough endoplasmic reticulum from insulin mRNA
and delivered to the Golgi apparatus. There, a series of proteolytic cleavage steps
generate the mature insulin and a cleavage peptide, C-peptide.Both insulin and C-
peptide are then stored in secretory granules and secreted in equimolar quantities

VSM
after physiologic stimulation; increasingly, C-peptide levels are being used as a
clinical assay to measure endogenous insulin secretion. The most important stimulus
that triggers insulin synthesis and release is glucose itself.
Insulin Action and Insulin Signaling Pathways Insulin is the most potent anabolic
hormone known, with multiple synthetic and growth-promoting effects. Its principal
metabolic function is to increase the rate of glucose transport into certain cells in the
body.
PATHOGENESIS OF TYPE 1 DIABETES MELLITUS This form of diabetes results
from a severe lack of insulin caused by an immunologically mediated destruction of
[3 cells. Type I diabetes most commonly develops in childhood, becomes manifest at
puberty, and progresses with age. Since the disease can develop at any age,
including late adulthood, the appellation "juvenile diabetes " is now considered
obsolete.Type 1 diabetes is an autoimmune disease in which islet destruction is
caused primarily by T lymphocytes reacting against as yet poorly defined 13-cell
antigens.
PATHOGENESIS OF TYPE 2 DIABETES MELLITUS Environmental factors, such
as a sedentary life style and dietary habits, clearly play a role, as will become evident
when obesity is considered. Nevertheless, genetic factors are even more important
than intype 1 diabetes.
PATHOGENESIS OF THE COMPLICATIONS OF DIABETES The morbidity
associated with long-standing diabetes of either type results from a number of
serious complications, involving both large- and medium-sized muscular arteries
(macrovascular disease), as well as capillary dysfunction in target organs (
microvascular disease). Macrovascular disease causes accelerated atherosclerosis
among diabetics, resulting in increased risk of myocardial infarction, stroke, and
lowerextremity gangrene. The effects of microvascular disease are most profound in
the retina, kidneys, and peripheral nerves, resulting in diabetic retinopathy,
nephropathy, and neuropathy, respectively.
89. Pathobiochemistry of diabetes mellitus

VSM
90. Complications in diabetes mellitus.
The morbidity associated with long-standing diabetes of either type results from a
number of serious complications, involving both large- and medium-sized muscular
arteries (macrovascular disease), as well as capillary dysfunction in

VSM
target organs ( microvascular disease). Macrovascular disease causes accelerated
atherosclerosis among diabetics, resulting in increased risk of myocardial infarction,
stroke, and lowerextremity gangrene. The effects of microvascular disease are most
profound in the retina, kidneys, and peripheral nerves, resulting in diabetic
retinopathy, nephropathy, and neuropathy, respectively. Diabetes is the leading
cause of blindness and end-stage renal disease in the Western hemisphere, besides
contributing substantially to the incidence of cardiovascular events each year.
Hence, the basis of long-term complications of diabetes is the subject of a great deal
of research. Most of the available experimental and clinical evidence suggests that
the complications of diabetes are a consequence of the metabolic derangements,
mainly hyperglycemia. Formation of Advanced Glycation End Products. Advanced
glycation end products (AGEs) are formed as a result of nonenzymatic reactions
between intracellular glucose-derived dicarbonyl precursors (glyoxal, methylglyoxal,
and 3-deoxyglucosone) with the amino group of both intracellular and extracellular
proteins. ■ On extracellular matrix components, such as collagen or laminin, the
formation of AGEs causes cross-linking between polypeptides, resulting in abnormal
matrix— matrix and matrix—cell interactions. For example, crosslinking between
collagen type I molecules in large vessels decreases their elasticity, which may
predispose these vessels to shear stress and endothelial injury
■ Circulating plasma proteins are modified by addition of AGE residues; these
proteins, in turn, bind to AGE receptors on several cell types (endothelial cells,
mesangial cells,macrophages).
AGE residues; these proteins, in turn, bind to AGE receptors on several cell types
(endothelial cells, mesangial cells, macrophages). The AGE-receptor ligation results
in activation and nuclear translocation of the pleotropic transcription factor NF-KB,
generating a variety of cytokines, growth factors and other pro-inflammatory
molecules. AGEs also contribute to microvascular injury in diabetes.The AGE
inhibitor aminoguanidine has recently been shown to retard the progression of
nephropathy in type 1 diabetics.
Activation of Protein Kinase C. Activation of intracellular protein kinase C (PKC) by
calcium ions and the second messenger diacylglycerol (DAG) is an important signal
transduction pathway in many cellular systems. Intracellular hyperglycemia can
stimulate the de novo synthesis of DAG from glycolytic intermediates and hence
cause activation of PKC. The downstream effects of PKC activation are numerous
and include the following:"

■ Production of the proangiogenic molecule vascular endothelial growth factor ■


Increased activity of the vasoconstrictor endothelin-1 and decreased activity of the
vasodilator endothelial nitric oxide synthase. ■ Production of profibrogenic molecules
like transforming growth factor-B (TGF-B), leading to increased deposition of
extracellular matrix and basement membrane material ■ Production of the
procoagulant molecule plasminogen activator inhibitor-1 (PAI-1), leading to reduced
fibrinolysis and possible vascular occlusive episodes ■ Production of pro-
inflammatory cytokines by the vascular endothelium.

VSM
Intracellular Hyperglycemia with Disturbances inPolyol Pathways. In some tissues
that do not require insulin for glucose transport (e.g., nerves, lenses, kidneys, blood
vessels), hyperglycemia leads to an increase in intracellular glucose that is then
metabolized by the enzyme aldose reductase to sorbitol, a polyol, and eventually to
fructose. In this process, intracellular NADPH is used as a cofactor. NADPH is also
required as a cofactor by the enzyme glutathione reductase for regenerating reduced
glutathione (GSH). GSH is one of the important antioxidant mechanisms in the cell,
and a reduction in GSH levels increases cellular susceptibility to oxidative stress."
91. Pituitary adenoma.
The most common cause of hyperpituitarism is an adenoma arising in the anterior
lobe. Other, less common, causes include hyperplasia and carcinomas of the
anterior pituitary, secretion of hormones by some extrapituitary tumors, and certain
hypothalamic disorders. Pituitary adenomas can be functional (i.e., associated with
hormone excess and clinical manifestations thereof) or silent (i.e.,
immunohistochemical and/or ultrastructural demonstration of hormone production at
the tissue level only, without clinical symptoms of hormone excess). Both functional
and silent pituitary adenomas are usually composed of a single cell type and
produce a single predominant hormone, although exceptions are known to occur.
Pituitary adenomas are classified on the basis of hormone(s) produced by the
neoplastic cells detected by immunohistochemical stains performed on tissue
sections (Table 24-1). Some pituitary adenomas can secrete two hormones (GH and
prolactin being the most common combination), and rarely, pituitary adenomas are
plurihormonal.
Finally, pituitary adenomas may be hormone-negative, based on absence of
immunohistochemical reactivity and ultra- structural demonstration of lineage-
specific differentiation. Both silent and hormone-negative pituitary adenomas may
cause hypopituitarism as they encroach on and destroy adjacent anterior pituitary
parenchyma. Clinically diagnosed pituitary adenomas are responsible for about 10%
of intracranial neoplasms; they are discovered incidentally in up to 25% of routine
autopsies. In fact, using highresolution computed tomography or magnetic
resonance imaging suggest that approximately 20% of "normal" adult pituitary glands
harbor an incidental lesion measuring 3 mm or more in diameter, usually a silent
adenoma.' Pituitary adenomas are usually found in adults, with a peak incidence
from the thirties to the fifties. Most pituitary adenomas occur as isolated lesions. In
about 3% of cases, however, adenomas are associated with multiple endocrine
neoplasia (MEN) type 1.
■ The great majority of pituitary adenomas are monoclonal in origin, even those that
are plurihormonal, suggesting that most arise from a single somatic cell.
■ G-protein mutations are possibly the best-characterized molecular abnormalities in
pituitary adenomas.
■ Multiple endocrine neoplasia (MEN) syndrome (discussed in detail below) is a
familial disorder associated with tumors and hyperplasias of multiple endocrine
organs, including the pituitary. ■ Additional molecular abnormalities present in

VSM
aggressive or advanced pituitary adenomas include activating mutations of the RAS
oncogene and overexpression of the c-MYC oncogene, suggesting that these
genetic events arelinked to disease progression.'
PROLACTINOMAS Prolactinomas (lactotroph adenomas) are the most frequent type
of hyperfunctioning pituitary adenoma, accounting for about 30% of all clinically
recognized pituitary adenomas. These lesions range from small microadenomas to
large, expansile tumors associated with substantial mass effect.
GROWTH HORMONE (SOMATOTROPH CELL) ADENOMAS GH-secreting tumors
are the second most common type of functioning pituitary adenoma. As we have
mentioned, 40% of somatotroph cell adenomas express a mutant GTPasedeficient
a-subunit of the G-protein, G,. Somatotroph cell adenomas may be quite large by the
time they come to clinical attention because the manifestations of excessive GH may
be subtle. CORTICOTROPH CELL ADENOMAS Corticotroph adenomas are usually
small microadenomas at the time of diagnosis. These tumors are most often
basophilic (densely granulated) and occasionally chromophobic (sparsely
granulated). Both variants stain positively with periodic acid—Schiff (PAS) because
of the presence of carbohydrate in pre-opiomelanocorticotropin (POMC), the ACTH
precursor molecule; in addition, they demonstrate variable immunoreactivity for
POMC and its derivatives, including ACTH and [3endorphin.
OTHER ANTERIOR PITUITARY ADENOMAS Gonadotroph((LH-producing and
FSH-producing) Thyrotroph (TSH-producing) Nonfunctioning pituitary adenomas
92. Hypopituitarism.
Hypopituitarism refers to decreased secretion of pituitary hormones, which can result
from diseases of the hypothalamus or of the pituitary. Hypofunction of the anterior
pituitary occurs when approximately 75% of the parenchyma is lost or absent. This
may be congenital or the result of a variety of acquired abnormalities that are
intrinsic to the pituitary.
Most cases of hypofunction arise from destructive processes directly involving the
anterior pituitary, although other mechanisms have been identified:
■ Tumors and other mass lesions: Pituitary adenomas, other benign tumors arising
within the sella, primary and metastatic malignancies, and cysts can cause
hypopituitarism. Any mass lesion in the sella can cause damage by exerting
pressure on adjacent pituitary cells.
■ Pituitary surgery or radiation: Surgical excision of a pituitary adenoma may
inadvertently extend to the nonadenomatous pituitary. Radiation of the pituitary, used
to prevent regrowth of residual tumor after surgery, can damage the
nonadenomatous pituitary. ■ Pituitary apoplexy: As has been mentioned, this is a
sudden hemorrhage into the pituitary gland, often occurring into a pituitary
adenoma.; ■ Ischemic necrosis of the pituitary and Sheehan syndrome:Ischemic
necrosis of the anterior pituitary is an important cause of pituitary nsufficiency.
Sheehan syndrome, or postpartum necrosis of the anterior pituitary, is the most
common form of clinically significant ischemic necrosis of the anterior pituitary. `

VSM
During pregnancy, the anterior pituitary enlarges to almost twice its normal size. This
physiologic expansion of the gland is not accompanied by an increase in blood
supply from the low-pressure venous system; hence, there is relative anoxia of the
pituitary. ■ Ratlike cleft cyst: These cysts, lined by ciliated cuboidal epithelium with
occasional goblet cells and anterior pituitary cells, can accumulate proteinaceous
fluid and expand, compromising the normal gland. ■ Empty sella syndrome: Any
condition that destroys part or all of the pituitary gland, such as ablation of the
pituitary by surgery or radiation, can result in an empty sella. ■ Genetic defects: Rare
congenital deficiencies of one or more pituitary hormones have been recognized in
children. Less frequently, disorders that interfere with the delivery of pituitary
hormone—releasing factors from the hypothalamus, such as hypothalamic tumors,
may also cause hypofunction of the anterior pituitary. Any disease involving the
hypothalamus can alter secretion of one or more of the hypothalamic hormones that
influence secretion of the corresponding pituitary hormones.
Hypothalamic lesions that cause hypopituitarism include: ■ Tumors, including benign
lesions that arise in the hypothalamus, such as craniopharyngiomas, and malignant
tumors that metastasize to that site, such as breast and lung carcinomas. ■
Inflammatory disorders and infections, such as sarcoidosis or tuberculous meningitis,
can cause deficiencies of anterior pituitary hormones and diabetes insipidus.
93. Diabetes insipidus.
The clinically relevant posterior pituitary syndromes involve ADH and include
diabetes insipidus and secretion of inappropriately high levels of ADH. ■ Diabetes
insipidus. ADH deficiency causes diabetes insipidus, a condition characterized by
excessive urination (polyuria) owing to an inability of the kidney to resorb water
properly from the urine. It can result from a variety of processes, including head
trauma, tumors, and inflammatory disorders of the hypothalamus and pituitary as
well as surgical procedures involving these organs. The condition can also arise
spontaneously, in the absence of an underlying disorder. Diabetes insipidus from
ADH deficiency is designated as central to differentiate it from nephrogenic diabetes
insipidus, which is a result of renal tubular unresponsiveness to circulating ADH. The
clinical manifestations of the two diseases are similar and include the excretion of
large volumes of dilute urine with an inappropriately low specific gravity. Serum
sodium and osmolality are increased owing to excessive renal loss of free water,
resulting in thirst and polydipsia. Patients who can drink water can generally
compensate for urinary losses; patients who are obtunded, bedridden, or otherwise
limited in their ability to obtain water may develop life-threatening dehydration.
94. Hyperthyroidism.
Thyrotoxicosis is a hypermetabolic state caused by elevated circulating levels of free
T, and T 4 . Because it is caused most commonly by hyperfunction of the thyroid
gland, it is often referred to as hyperthyroidism. However, in certain conditions the
oversupply is related to either excessive release of preformed thyroid hormone (e.g.,
in thyroiditis) or to an extrathyroidal source, rather than hyperfunction of the gland (
Table 24-2). Thus, strictly speaking, hyperthyroidism is only Not Associated with
HyperthyroidismSubacute granulomatous thyroiditis (painful)Subacute lymphocytic

VSM
thyroiditis (painless) Struma ovarii (ovarian teratoma with ectopic thyroid) Factitious
thyrotoxicosis (exogenous thyroxine intake) "'Associated with increased TSH; all
other causes of thyrotoxicosisassociated with decreased TSH.one (albeit the most
common) cause of thyrotoxicosis. The terms primary and secondary hyperthyroidism
are sometimes used to designate hyperthyroidism arising from an intrinsic thyroid
abnormality and that arising from processes outside of the thyroid, such as a TSH-
secreting pituitary tumor. With this disclaimer, we will follow the common practice of
using the terms thyrotoxicosis and hyperthyroidism interchangeably. The three most
common causes of thyrotoxicosis are also associated with hyperfunction of the gland
and include the following:
■ Diffuse hyperplasia of the thyroid associated with Graves disease (accounts for
85% of cases) ■ Hyperfunctional multinodular goiter ■ Hyperfunctional adenoma of
the thyroid
Clinical Course. The clinical manifestations of hyperthyroidism are protean and
include changes referable to the hypermetabolic state induced by excess thyroid
hormone as well as those related to overactivity of the sympathetic nervous system
(i.e., an increase in the 13-adrenergic "tone").
Excessive levels of thyroid hormone result in an increase in the basal metabolic rate.
The skin of thyrotoxic patients tends to be soft, warm, and flushed because of
increased blood flow and peripheral vasodilation to increase heat loss. Heat
intolerance is common. Sweating is increased because of higher levels of
calorigenesis. Increased basal metabolic rate also results in characteristic weight
loss despite increased appetite. Cardiac manifestations are among the earliest and
most consistent features of hyperthyroidism.
Myocardial changes, such as foci of lymphocytic and eosinophilic infiltration, mild
fibrosis in the interstitium, fatty changes in myofibers, and an increase in size and
number of mitochondria, have been described. In the neuromuscular system,
overactivity of the sympathetic nervous system produces tremor, hyperactivity,
emotional lability, anxiety, inability to concentrate, and insomnia.
Ocular changes often call attention to hyperthyroidism. A wide, staring gaze and lid
lag are present because of sympathetic overstimulation of the levator palpebrae
superioris. However, true thyroid ophthalmopathy associated with proptosis is a
feature seen only in Graves disease.
In the gastrointestinal system, sympathetic hyperstimulation of the gut results in
hypermotility, malabsorption, and diarrhea.
The skeletal system is also affected in hyperthyroidism.Thyroid hormone stimulates
bone resorption, resulting in increased porosity of cortical bone and reduced volume
of trabecular bone.
A diagnosis of hyperthyroidism is made using both clinical and laboratory findings.
The measurement of serum TSH concentration using sensitive TSH (sTSH) assays
provides the most useful single screening test for hyperthyroidism, as its levels are
decreased even at the earliest stages, when the disease may still be subclinical.

VSM
95. Hypothyroidism.
Hypothyroidism is caused by any structural or functional derangement that interferes
with the production of adequate levels of thyroid hormone. It can result from a defect
anywhere in the hypothalamic-pituitary-thyroid axis. As in the case of
hyperthyroidism, this disorder is divided into primary and secondary categories,
depending on whether the hypothyroidism arises from an intrinsic abnormality in the
thyroid or occurs as a result of pituitary disease; rarely, hypothalamic failure is a
cause of tertiary hypothyroidism. Primary hypothyroidism accounts for the vast
majority of cases of hypothyroidism. Primary hypothyroidism can be thyroprivic (due
to absence or loss of thyroid parenchyma) or goitrous (due to enlargement of the
thyroid gland under the influence of TSH). The causes of primary hypothyroidism
include the following. Surgical or radiation-induced ablation of thyroid parenchyma
can cause hypothyroidism. A large resection of the gland (total thyroidectomy) for the
treatment of hyperthyroidism of a primary neoplasm can lead to hypothyroidism. The
gland may also be ablated by radiation, whether in the form of radioiodine
administered for the treatment of hyperthyroidism, or exogenous irradiation, such as
external radiation therapy to the neck.
Autoimmune hypothyroidism is the most common cause of goitrous hypothyroidism
in iodinesufficient areas of the world. The vast majority of cases of autoimmune
hypothyroidism are due to Hashimoto thyroiditis. Circulating autoantibodies,
including anti–TSH receptor autoantibodies, are commonly found in Hashimoto
thyroiditis.
Causes Of Hypothyroidism
Primary:
Developmental (thyroid dysgenesis : PAX-8,TTF-2,TSH-receptor Mutations) Thyroid
hormone resistance syndrome (TRb Mutations) Postablative Surgery,radioiodine
therapy, or external radiation Autoimmune hypothyroidism Hashimoto thyroiditis*
Iodine deficiency* Drugs(lithium,iodidesip-aminosaliycylic acid)* Congenital
biosynthetic defect (dyshormonogenetic goiter)
Secondary: Pituitary failure
Tertiary
Hypothalamic failure(rare) Associated with enlargement of thyroid ("goitrous
hypothyroidism").Hashimoto thyroiditis and postablative hypothroidism account for
the majority of cases.
96. Goiter.
Diffuse and Multinodular Goiters Enlargement of the thyroid, or goiter, is the most
common manifestation of thyroid disease. Diffuse and multinodular goiters reflect
impaired synthesis of thyroid hormone, most often caused by dietary iodine
deficiency. Impairment of thyroid hormone synthesis leads to a compensatory rise in
the serum TSH level, which, in turn, causes hypertrophy and hyperplasia of thyroid
follicular cells and, ultimately, gross enlargement of the thyroid gland. The

VSM
compensatory increase in functional mass of the gland is able to overcome the
hormone deficiency, ensuring an euthyroid metabolic state in the vast majority of
individuals. If the underlying disorder is sufficiently severe (e.g., a congenital
biosynthetic defect or endemic iodine deficiency, see below), the compensatory
responses may be inadequate to overcome the impairment in hormone synthesis,
resulting in goitrous hypothyroidism. The degree of thyroid enlargement is
proportional to the level and duration of thyroid hormone deficiency.
DIFFUSE NONTOXIC (SIMPLE) GOITER Diffuse nontoxic (simple) goiter specifies a
form of goiter that diffusely involves the entire gland without producing nodularity.
Because the enlarged follicles are filled with colloid, the term colloid goiter has been
applied to this condition.This disorder occurs in both an endemic and a sporadic
distribution. Endemic goiter occurs in geographic areas where the soil, water, and
food supply contain only low levels of iodine. The term endemic is used when goiters
are present in more than10% of the population in a given region. Variations in the
prevalence of endemic goiter in regions with similar levels of iodine deficiency point
to the existence of other causative influences, particularly dietary substances,eferred
to as goitrogens. Sporadic goiter occurs less frequently than does endemic goiter.
There is a striking female preponderance and a peak incidence at puberty or in
young adult life. Sporadic goiter can be caused by a number of conditions, including
the ingestion of substances that interfere with thyroid hormone synthesis.
Clinical Course. The vast majority of patients with simple goiters are clinically
euthyroid. Therefore, the clinical manifestations are primarily related to mass effects
from the enlarged thyroid gland.
MULTINODULAR GOITER With time, recurrent episodes of hyperplasia and
involution combine to produce a more irregular enlargement of the thyroid, termed
multinodular goiter. Virtually all longstanding simple goiters convert into multinodular
goiters. They may be nontoxic or may induce thyrotoxicosis (toxic multinodular
goiter). Multinodular goiters produce the most extreme thyroid enlargements and are
more frequently mistaken for neoplastic involvement than any other form of thyroid
disease. Because they derive from simple goiter, they occur in both sporadic and
endemic forms, having the same female-tomale distribution and presumably the
sane origins but affecting older individuals because they are late complications.
97. Excessive secretion of adrenal glucocorticoid hormones. Cushing’s syndrome.
Just as there are three basic types of corticosteroids elaborated by the adrenal
cortex (glucocorticoids, mineralocorticoids, and sex steroids), so there are three
distinctive hyperadrenal clinical syndromes: (1) Cushing syndrome, characterized by
an excess of cortisol; (2) hyperaldosteronism; and (3) adrenogenital or virilizing
syndromes caused by an excess of androgens. The clinical features of these
syndromes overlapsomewhat because of the overlapping functions of some of the
adrenal steroids.
Hypercortisolism (Cushing Syndrome) Pathogenesis. This disorder is caused by any
condition that produces an elevation in glucocorticoid levels."' There are four
possible sources of excess cortisol .In clinical practice, most causes of Cushing

VSM
syndrome are the result of the administration of exogenous glucocorticoids. The
other three sources of the hypercortisolism call be categorized as endogenous
Cushing syndrome:
■ Primary hypothalamic-pituitary diseases associated with hypersecretion of ACTH ■
Hypersecretion of cortisol by an adrenal adenoma, carcinoma, or nodular
hyperplasia ■ The secretion of ectopic ACTH by a nonendocrine neoplasm
Primary hypersecretion of ACTH accounts for 70% to 80% of cases of endogenous
hypercortisolism. In recognition of the neurosurgeon who first published the full
description of this syndrome and related it to a pituitary lesion,"' this pituitary form of
Cushing syndrome is referred to as Cushing disease. The disorder affects women
about five times more frequently than men, and it occurs most frequently during the
twenties and thirties. In the vast majority of cases, the pituitary gland contains an
ACTH-producing microadenoma that does not produce mass effects in the brain;
some corticotroph tumors qualify as macroadenomas (>10 mm). In most of the
remaining cases, the anterior pituitary contains areas of corticotroph cell hyperplasia
without a discrete adenoma. Corticotroph cell hyperplasia may be primary or may
arise secondarily from excessive stimulation of ACTH release by a hypothalamic
corticotropin releasing hormone (CRH)-producing tumor.
Morphology. The main lesions of Cushing syndrome are found in the pituitary and
adrenal glands. The pituitary in Cushing syndrome shows changes regardless of the
cause. The most common alteration, resulting from high levels of endogenous or
exogenous glucocorticoids, is termed Crooke hyaline change. In this condition, the
normal granular, basophilic cytoplasm of the ACTH-producing cells in the anterior
pituitary is replaced by homogeneous, lightly basophilic material. This alteration is
the result of the accumulation of intermediate keratin filaments in the cytoplasm. The
morphology of the adrenal glands depends on the cause of the hypercortisolism. The
adrenals have one of the following abnormalities: (1) cortical atrophy; (2) diffuse
hyperplasia; (3) nodular hyperplasia; and (4) an adenoma, rarely a carcinoma.
Diffuse hyperplasia is found in 60% to 70% of cases of Cushing syndrome. Both
glands are enlarged, either subtly or markedly, weighing up to 25 to 40 gm.The
adrenal cortex is diffusely thickened and yellow, owing to an increase in the size and
number of lipidrich cells in the zonae fasciculata and reticularis. Some degree of
nodularity is common but is pronounced in nodular hyperplasia. Primary
adrenocortical neoplasms causing Cushing syndrome may be malignant orbenign.
Adenomas or carcinomas of the adrenal cortex as the source of cortisol secretion
are not macroscopically distinctive from nonfunctioning adrenal neoplasms to be
described later.
98. Adrenocortical insufficiency. Addison’s disease.
ADRENAL INSUFFICIENCY Adrenocortical insufficiency, or hypofunction, may be
caused by either primary adrenal disease (primary hypoadrenalism) or decreased
stimulation of the adrenals owing to a deficiency of ACTH (secondary
hypoadrenalism). The patterns of adrenocortical insufficiency can be considered
under the following headings: (1) primary acute adrenocortical insufficiency (adrenal

VSM
crisis), (2) primary chronic adrenocortical insufficiency (Addison disease), and (3)
secondary adrenocortical insufficiency. Primary Acute Adrenocortical Insufficiency
Acute adrenal cortical insufficiency occurs in a variety of clinical settings:
■ As a crisis in patients with chronic adrenocortical insufficiency precipitated by any
form of stress that requires an immediate increase in steroid output from glands
incapable of responding ■ In patients maintained on exogenous corticosteroids, in
whom rapid withdrawal of steroids or failure to increase steroid doses in response to
an acute stress may precipitate an adrenal crisis, owing to the inability of the atrophic
adrenals to produce glucocorticoid hormones ■ As a result of massive adrenal
hemorrhage, which destroys the adrenal cortex sufficiently to cause acute
adrenocortical insufficiency. This occurs in newborns following prolonged and difficult
delivery with considerable trauma and hypoxia, leading to extensive adrenal
hemorrhages beginning in the medulla and extending into the cortex. Secondary
Insufficiency Hypothalamic pituitary disease Neoplasm, inflammation (sarcoidosis,
tuberculosis, pyogens, fungi) Hypothalamic pituitary suppression Long-term steroid
administration Steroid-producing neoplasms
Primary Chronic Adrenocortical Insufficiency (Addison Disease) In a paper published
in 1855, Thomas Addison described a group of patients suffering from a constellation
of symptoms, including "general languor and debility, remarkable feebleness of the
heart 's action, and a peculiar change in the color of the skin" associated with
disease of the "suprarenal capsules" or,in more current terminology, the adrenal
glands. Addison disease, or chronic adrenocortical insufficiency, is an uncommon
disorder resulting from progressive destruction of the adrenal cortex. Pathogenesis.
A large number of diseases may attack the adrenal cortex, including lymphomas,
amyloidosis, sarcoidosis, hemochromatosis, fungal infections, and adrenal
hemorrhage, but more than 90% of all cases are attributable to one of four disorders:
autoimmune adrenalitis, tuberculosis, the acquired immune deficiency syndrome
(AIDS), or metastatic cancers. ■ Autoinmrune polyendocrine syndrome type 1
(APSI) is also known as autoimmune polyendocrinopathy, candidiasis, and
ectodermal dystrophy (APECED). ■ Autoimmune polyendocrine syndrome type 2
(APS2) usually starts in early adulthood and presents as a combination of adrenal
insufficiency with autoimmune thyroiditis or type I diabetes. ■ Isolated autoimmune A
ddison disease presents with autoimmune destruction restricted to the adrenal
glands.However, in terms of age at presentation and linkage to HLA and other
susceptibility loci, isolated autoimmune adrenalitis overlaps with APS2, suggesting
that the former may be a variant of the latter. Infections, particularly tuberculosis and
those produced by fungi, may also cause primary chronic adrenocortical
insufficiency.Tuberculous adrenalitis, which once accounted for as much as 90% of
Addison disease, has become less common with the development of antituberculous
agents.Histoplasma capsulatum and Coccidioides immitis may also result in chronic
adrenocortical insufficiency. Patients with AIDS are at risk for developing adrenal
insufficiency from several infectious (cytomegalovirus, Mycobacterium
aviuinintercellulare)and noninfectious complications (Kaposi sarcoma).Metastatic
neoplasms involving the adrenals are another potential cause of adrenal
insufficiency. Genetic disorders of adrenal insufficiency include adrenal hypoplasia

VSM
congenital (AHC) and adrenoleukodystrophy. Morphology. The anatomic changes in
the adrenal glands depend on the underlying disease. Primary autoimmune
adrenalitis is characterized by irregularly shrunken glands, which may be difficult to
identify within the suprarenal adipose tissue. Clinical Course. Addison disease
begins insidiously and does not come to attention until at least 90% of the cortex of
both glands is destroyed and the levels of circulating glucocorticoids and
mineralocorticoids are significantly decreased.The initial manifestations include
progressive weakness and easy fatigability, which may be dismissed as nonspecific
complaints. Gastrointestinal disturbances are common and include anorexia,
nausea, vomiting, weight loss, and diarrhea. Secondary Adrenocortical Insufficiency
Any disorder of the hypothalamus and pituitary, such as metastatic cancer, infection,
infarction, or irradiation, that reduces the output of ACTH leads to a syndrome of
hypoadrenalism that has many similarities to Addison disease.
Analogously, prolonged administration of exogenous glucocorticoids suppresses the
output of ACTH and adrenal function. With secondary disease, the
hyperpigmentation of primary Addison disease is lacking because melanotropic
hormone levels are low.ACTH deficiency can occur alone, but in some instances, it
is only one part of panhypopituitarism, associated with multiple primary trophic
hormone deficiencies. The differentiation of secondary disease from Addison
disease can be confirmed with demonstration of low levels of plasma ACTH in the
former. Morphology. In cases of hypoadrenalism secondary to hypothalamic or
pituitary disease (secondary hypoadrenalism), depending on the extent of ACTH
lack, the adrenals may be moderately to markedly reduced in size. They are reduced
to small, flattened structures that usually retain their yellow color owing to a small
amount of residual lipid. The cortex may be reduced to a thin ribbon composed
largely of zona glomerulosa. The medulla is unaffected.
99. Hyperaldosteronism. Conn’s syndrome.
Hyperaldosteronism is the generic term for a small group of closely related,
uncommon syndromes, all characterized by chronic excess aldosterone secretion.
Excessive levels of aldosterone cause sodium retention and potassium excretion,
with resultant hypertension and hypokalemia. Hyperaldosteronism may be primary,
or it may be a secondary event resulting from an extra-adrenal cause. Primary
hyperaldosteronism indicates an autonomous overproduction of aldosterone, with
resultant suppression of the renin-angiotensin system and decreased plasma renin
activity. Primary hyperaldosteronism is caused by one of three mechanisms; ■
Adrenocortical neoplasm, either an aldosteroneproducing adrenocortical adenoma
(the most common cause) or, rarely, an adrenocortical carcinoma. In approximately
80% of cases, primary hyperaldosteronism is caused by a solitary aldosterone-
secreting adenoma, a condition referred to as Corm syndrome. This syndrome
occurs most frequently in adult middle life and is more common in women than in
men (2:1). Multiple adenomas may be present in an occasional patient.
■ Primary adrenocortical hyperplasia (idiopathic hyperaldosteronism), characterized
by bilateral nodular hyperplasia of the adrenal glands, highly reminiscent of those
found in the nodular hyperplasia of Cushing syndrome. The genetic basis of

VSM
idiopathic hyperaldosteronism is not clear, although it is possibly caused by an
overactivity of the aldosterone synthase gene, CY P11 NI Glucocorticoid-remediable
hyperaldosteronism is an uncommon cause of primary hyperaldosteronism that is
familial and genetic. In some families, it is caused by a chimeric gene resulting from
fusion between CY P11BI (the 11B-hydroxylase gene) and CY P11 B2 (the
aldosterone synthase gene)."_ This leads to a sustained production of hybrid
steroids in addition to both cortisol and aldosterone. The activation of aldosterone
secretion is under the influence of ACTH and hence is suppressible by exogenous
administration of dexamethasone. In secondary hyperaldosteronism, in contrast,
aldosterone release occurs in response to activation of the reninangiotensin system.
It is characterized by increased levels of plasma renin and is encountered in
conditions such as the following:
■ Decreased renal perfusion (arteriolar nephrosclerosis, renal artery stenosis) ■
Arterial hypovolemia and edema (congestive heart failure, cirrhosis, nephrotic
syndrome) ■ Pregnancy (due to estrogen-induced increases in plasma renin
substrate).
Morphology. Aldosterone-producing adenomas are almost always solitary, small (<2
cm in diameter), well-circumscribed lesions, more often found on the left than on the
right. They tend to occur in the thirties and forties, and in women more often than in
men.These lesions are often buried within the gland and do not produce visible
enlargement, a point to be remembered in interpreting sonographic or scanning
images. Conn's syndrome is an aldosterone-producing adenoma. Primary
hyperaldosteronism has many causes, including adrenal hyperplasia and adrenal
carcinoma. The syndrome is due to:
bilateral idiopathic adrenal hyperplasia 70 % unilateral idiopathic adrenal hyperplasia
20 % aldosterone-secreting adrenal adenoma (benign tumor, < 5%) rare forms,
including disorders of the renin-angiotensin system
Pathogenesis. Aldosterone enhances exchange of sodium for potassium in the
kidney, so increased aldosteronism will lead to hypernatremia (elevated sodium
level) and low blood potassium known as hypokalemia. Once the potassium has
been significantly reduced by aldosterone, a sodium/hydrogen pump in the nephron
becomes more active, leading to increased excretion of hydrogen ions and further
exacerbating the elevated sodium level resulting in a further increase in
hypernatremia. The hydrogen ions exchanged for sodium are generated by carbonic
anhydrase in the renal tubule epithelium, causing increased production of
bicarbonate. The increased bicarbonate and the excreted hydrogen combine to
generate a metabolic alkalosis.The high pH of the blood makes calcium less
available to the tissues and causes symptoms of hypocalcemia.
100. Disorders of the female reproductive tract. Menstrual disorders.
Endometriosis - a condition involving colonization of the abdominal/pelvic cavity with
islands of endometrial tissue. Endometrium is the lining layer of the uterus which
sloughs off with each menstruation. If endometrial tissue flushes up the uterine tube
and spills into the abdomen (peritoneal cavity), the clots of endometrial tissue can

VSM
attach to abdominal organs such as the bladder, rectum, intestinal loops and then
cycle along with the uterus in response to monthly changes in ovarian hormones.
Bleeding into the abdomen irritates the lining membrane, the peritoneum, and
causes abdominal pain. Pelvic inflammatory disease (PID) – although males have a
closed abdominal cavity, the female abdominal cavity has a direct anatomical path
from the outside world via the female reproductive tract. Bacteria can make their way
up the vagina, through the uterus, and traverse the uterine tubes which open into the
abdominal cavity. Inflammation of the lining of the abdominal cavity, the peritoneum,
causes abdominal pain. Although there are many potential causes of PID,
gonorrheal infection is one of them. Chronic Inflammation of the uterine tubes can
occlude them resulting in infertility. Prolapsed uterus – the uterus is almost directly
above the vagina. In fact, the cervix, the neck region, of the uterus extends into the
upper vagina. Ligaments hold the uterus in proper position so that it does not
prolapse or herniate into the vagina. Severe prolapse can result in the uterine cervix
protruding from the vaginal opening. Surgical repair is typically required to restore
the uterus to its proper anatomical position.
Hormonal changes in the normal menstrual cycle In the ovulatory cycle, the
hypothalamus secretes gonadotropin-releasing hormone (GnRH), which stimulates
the pituitary to release follicle-stimulating hormone (FSH). This, in turn, causes an
ovarian follicle to grow and mature. In mid cycle, a surge of luteinizing hormone (LH)
occurs with an FSH surge, resulting in ovulation. The developing follicle produces
estrogen, which stimulates the endometrium to proliferate. After the ovum is
released, FSH and LH levels fall, corpus luteum develops at the site of the ruptured
follicle, and progesterone is secreted from the ovary. Progesterone causes the
proliferating endometrium to differentiate and stabilize. Fourteen days after ovulation,
menstruation results from endometrial shedding secondary to the rapid decline in the
levels of estrogen and progesterone from the involuting corpus luteum. Hormonal
changes during anovulatory cycles Anovulatory cycles are common in the first 2
years after menarche because of the immaturity of the HPO axis. They can also
occur in various pathological conditions.In anovulatory cycles, the follicular growth
occurs with the stimulation from FSH; however, due to lack of LH surge, ovulation
fails to occur. Consequently, no corpus luteum is formed and no progesterone is
secreted. The endometrium continues its proliferative phase excessively.

101. Disorders of the female reproductive tract. Infertility.


Ovulatory dysfunction Hypo-gonadotrophic anovulation occurs as a result of
hypothalamic or pituitary abnormalities. Hyper-gonadotrophic anovulation occurs as
a result of ovarian failure.Polycystic ovarian syndrome is the most common cause of
eugonadotrophic anovulation. Tubal disease Most often caused by gonorrhoea and
chlamydia infection. Chlamydia trachomatis is obligate intracellular parasite that
invades the cervix, uterus, and fallopian tubes. This organism is the leading cause
for acute salpingitis worldwide. The manifestation of this disease is varied, ranging
from sub-clinical to an acute tubo-ovarian abscess that can include peritonitis and
peri-hepatitis. [19] High anti-chlamydial antibody titres highly correlate to abnormal

VSM
tubal pathology. The risk of tubal occlusion has been approximated as 10% for an
initial episode of salpingitis, and then doubled with every subsequent infection. Any
pelvic infection, including appendicitis and diverticulitis, can damage the fallopian
tubes. Endometriosis Endometriosis can cause intra-abdominal inflammation and
scar tissue. This growth of hormonally responsive endometrial tissue outside the
uterus may cause anatomical obstruction of the fallopian tubes. It may also lead to
infertility by producing cytokines that may be toxic to sperm or embryos. Age-related
Age-related decreases in fecundity are caused by declining oocyte numbers and
poorer oocyte quality. Oogenesis begins in utero. By month 7 of gestation, mitosis
completes and the peak number of oocytes (approximately 7 million) is achieved.
Hormone-independent apoptosis begins at this time and continues until menopause,
regardless of factors such as contraceptive use and pregnancy. Although the
number of oocytes remaining in the ovary (ovarian reserve) impact on pregnancy
rates, age also leads to a higher rate of oocyte aneuploidy due to decreased
chromosomal crossover, [23] meiotic spindle fragility, [24] and telomeric shortening.
This leads to a high likelihood of implantation failure, miscarriage, and
chromosomally abnormal offspring (e.g., trisomy 21).
Unexplained Unexplained infertility or subfertility is defined as the failure to conceive
after 2 years of regular unprotected sexual intercourse in the face of normal
investigations (namely normal ovulation, normal semen analysis, patent fallopian
tubes). As couples go through the diagnostic and treatment pathways, an increasing
number will acquire some form of diagnosis so that the proportion of couples with so-
called unexplained subfertility will decline.The label of unexplained subfertility
recognises that there are numerous candidate sites for abnormalities causing
reduced fertility that cannot be recognised by standard diagnostic tests, but that,
ultimately, treatment may improve the chance of a pregnancy. Uterine abnormalities
Uterine abnormalities can be congenital or acquired. Failure of Müllerian duct fusion
results in uterine malformations including uterine didelphys, bi-cornuate or uni-
cornuate uterus, and uterine septum. [27] Submucosal or large intramural
leiomyomata may have an impact on implantation or cause tubal obstruction.
Endometritis, particularly when associated with a dilation and curettage procedure,
can destroy the endometrial lining and cause Asherman's syndrome (intrauterine
adhesions). Cervical abnormalities Cervical mucus is critical to facilitate sperm entry
into the uterus and to initiate sperm capacitation, the final step in sperm maturation.
During the peri-ovulatory period the mucus becomes abundant, thin, and stretchable.
Cervical maladies such as surgery or infection can disrupt the cervical glands and/or
mucus production.
102. Disorders of the male reproductive tract. Male infertility.
Hypospadias - literally “below the fleshy spike.” A condition in which the external
urinary meatus (opening) opens anywhere below the tip of the penis rather than at
the tip. Hydrocele - a fluid filled sac partially surrounding the testis. Manifests itself as
a swelling on the side of the scrotum. May cause discomfort. Can be surgically
corrected. And, who would most likely be doing the surgery? A urologist! Varicocele -
dilated and twisted veins of the testis, sort of “hemorrhoids” of the scrotum! Manifests
itself as a swelling on the side of the scrotum which may look and feel like a “bag of

VSM
worms.” May be surgically corrected if causing discomfort. This condition may also
cause reduced sperm count and male sterility due to sluggish blood flow elevating
testicular temperature. Cryptorchidism - literally “hidden testicle.” A condition of lack
of descent of one or both testes into the scrotum. If not corrected, usually by surgery,
before puberty, can lead to sterility and increased risk of testicular cancer. Benign
prostatic hypertrophy (BPH) - swelling of the prostate gland which surrounds the
base of the male bladder and urethra causing difficulty urinating, dribbling, and
nocturia (remember that word? See urinary system). The bane of old men! BPH
becomes more common as men age. Transurethral resection of the prostate (TURP)
- the surgical cure for BPH. An instrument inserted through the penile urethra is used
to partially cut away the prostate to relieve obstruction of the urinary tract. Prostate
Specific Antigen (PSA) – PSA is a marker protein for prostate cell secretions which
can be detected with a lab test. A rising PSA may be an early sign of prostate
cancer, although there may be other causes including false positive tests. How often
should men get a PSA test? Check in with The Prostate Cancer Foundation. Male
fertility requires normal sperm production and sperm transport, and adequate sexual
performance.These functions require normal levels of testosterone. Testosterone is
produced by the testicular Leydig cells under the influence of LH. Spermatogenesis
is controlled directly by testosterone and FSH. FSH is believed to act on the Sertoli
cells, which support spermatogenesis in the seminiferous tubules. The pituitary
production of FSH and LH is controlled by the hypothalamic gonadotrophin releasing
hormone (GnRH). GnRH and FSH production by the hypothalamus and pituitary is
negatively controlled directly by testosterone levels and through its aromatisation to
oestradiol at the central or peripheral levels. Inhibin is produced by Sertoli cells and
has a negative effect on FSH secretion. Circulating testosterone is bound mainly to
the sex hormone-binding globulin (SHBG) and albumin. Levels of SHBG affect the
active portion of circulating testosterone or free testosterone. Hyperprolactinaemia
has a negative affect on GnRH secretion.The presence of an intact Y and 1 copy of
the X chromosome is essential for the differentiation of the embryonic gonads into
testicles and for the later development of adequate spermatogenesis. Normal
fertilisation of the oocyte requires many motile sperm with intact acrosomal
reaction.Sperm production is negatively affected by endocrine effects of obesity,
increased local testicular heat, and exposure to environmental toxins or endocrine
disruptors. Smoking and alcohol also have a toxic effect on sperm production. A
variety of medications can alter testosterone production or have an antitestosterone
effect at the receptor level. Other medications or environmental exposures can
directly alter spermatogenesis or sperm motility.
103. Disorders of the male reproductive tract. Benign prostatic hyperplasia.
Hypospadias - literally “below the fleshy spike.” A condition in which the external
urinary meatus (opening) opens anywhere below the tip of the penis rather than at
the tip. Hydrocele - a fluid filled sac partially surrounding the testis. Manifests itself as
a swelling on the side of the scrotum. May cause discomfort. Can be surgically
corrected. And, who would most likely be doing the surgery? A urologist! Varicocele -
dilated and twisted veins of the testis, sort of “hemorrhoids” of the scrotum! Manifests
itself as a swelling on the side of the scrotum which may look and feel like a “bag of

VSM
worms.” May be surgically corrected if causing discomfort. This condition may also
cause reduced sperm count and male sterility due to sluggish blood flow elevating
testicular temperature. Cryptorchidism - literally “hidden testicle.” A condition of lack
of descent of one or both testes into the scrotum. If not corrected, usually by surgery,
before puberty, can lead to sterility and increased risk of testicular cancer. Benign
prostatic hypertrophy (BPH) - swelling of the prostate gland which surrounds the
base of the male bladder and urethra causing difficulty urinating, dribbling, and
nocturia (remember that word? See urinary system). The bane of old men! BPH
becomes more common as men age. Transurethral resection of the prostate (TURP)
- the surgical cure for BPH. An instrument inserted through the penile urethra is used
to partially cut away the prostate to relieve obstruction of the urinary tract.
Benignprostatic hyperplasia is another common example of pathologic hyperplasia
induced by responses to hormones, in this case, androgens. Although these forms of
hyperplasia are abnormal, the process remains controlled, because the hyperplasia
regresses if the hormonal stimulation is eliminated. As is discussed in Chapter 7, it is
this response to normal regulatory control mechanisms that distinguishes benign
pathologic hyperplasias from cancer, in which the growth control mechanisms
become defective. Pathologic hyperplasia, however, constitutes a fertile soil in which
cancerous proliferation may eventually arise. Thus, patients with hyperplasia of the
endometrium are at increased risk for developing endometrial cancer (Chapter 22).
Hyperplasia is also an important response of connective tissue cells in wound
healing, in which proliferating fibroblasts and blood vessels aid in repair (Chapter 3).
Under these circumstances, growth factors are responsible for the hyperplasia.
Stimulation by growth factors is also involved in the hyperplasia that is associated
with certain viral infections, such as papillomaviruses, which cause skin warts and a
number of mucosal lesions composed of masses of hyperplastic epithelium.

104. Motor disturbances: lower and upper motor neurons. Motor neuron disease.
Motor neurone disease is a rare condition that progressively damages the nervous
system, causing the muscles to waste away. Motor neurone disease occurs when
specialist nerve cells, called motor neurones, stop working properly. Motor neurones
control important muscle activity such as: walking speaking breathing
swallowing As the condition progresses, people with motor neurone disease will find
these activities increasingly difficult, and eventually impossible, to do. Exactly what
causes the motor neurones to stop working properly is unclear, but several theories
have been suggested. Find out about the possible causes of motor neurone disease.
In its early stages, motor neurone disease causes symptoms such as: a weakened
grip, which can cause difficulty picking up or holding objects a general feeling of
tiredness muscle pains and cramps
As the damage progresses, the symptoms become more debilitating. In the final
stages, a person with the condition will be unable to move their body (the medical
name for this is total body paralysis) and their breathing difficulties will get worse.
Read more about the symptoms of motor neurone disease. Types of motor neurone
disease

VSM
There are three main types of motor neurone disease, which have similar symptoms
but progress in different ways: Limb-onset disease begins with symptoms that
affect the arms and legs, such as muscle weakness and loss of function. This is the
most common type and accounts for 70% of cases. Bulbar-onset disease begins
with symptoms that affect the mouth and throat, such as difficulties swallowing and
slurred speech. This accounts for around 25% of cases. Respiratory-onset disease
begins with symptoms that affect the lungs, such as shortness of breath. This
accounts for around 5% of cases.
105. Motor disturbances: cerebellum and basal ganglia. Parkinson’s disease.
Parkinson's disease is a degenerative disorder of the central nervous system. The
motor symptoms of Parkinson's disease result from the death of dopamine-
generating cells in the substantia nigra, a region of the midbrain; the cause of this
cell death is unknown.
Classification The term parkinsonism is used for a motor syndrome whose main
symptoms are tremor at rest, stiffness, slowing of movement and postural instability.
Parkinsonian syndromes can be divided into four subtypes according to their origin:
primary or idiopathic, secondary or acquired, hereditary parkinsonism, and parkinson
plus syndromes or multiple system degeneration Pathophysiology
The primary symptoms of Parkinson's disease result from greatly reduced activity of
dopaminesecreting cells caused by cell death in thepars compacta region of the
substantia nigra.
There are five major pathways in the brain connecting other brain areas with the
basal ganglia. These are known as the motor, oculo-motor, associative, limbic and
orbitofrontal circuits, with names indicating the main projection area of each circuit.]
All of them are affected in PD, and their disruption explains many of the symptoms of
the disease since these circuits are involved in a wide variety of functions including
movement, attention and learning. Scientifically, the motor circuit has been examined
the most intensively.
A particular conceptual model of the motor circuit and its alteration with PD has been
of great influence since 1980, although some limitations have been pointed out
which have led to modifications. In this model, the basal ganglia normally exert a
constant inhibitory influence on a wide range of motor systems, preventing them
from becoming active at inappropriate times. When a decision is made to perform a
particular action, inhibition is reduced for the required motor system, thereby
releasing it for activation. Dopamine acts to facilitate this release of inhibition, so high
levels of dopamine function tend to promote motor activity, while low levels of
dopamine function, such as occur in PD, demand greater exertions of effort for any
given movement. Thus the net effect of dopamine depletion is to produce
hypokinesia, an overall reduction in motor output.[24] Drugs that are used to treat
PD, conversely, may produce excessive dopamine activity, allowing motor systems
to be activated at inappropriate times and thereby producing dyskinesias
106. Disturbances of the somatosensory system.

VSM
-A somatosensory deficiency may be caused by a peripheral neuropathy involving
peripheral nerves of the somatosensory system.
-This may present as numbness or paresthesia.
-Evaluation of any suspected disease of the somatosensory system is included in a
neurological examination of the peripheral nervous system.
-Hypoesthesia refers to a reduced sense of touch or sensation, or a partial loss of
sensitivity to sensory stimuli.
-Anaphia, also known as tactile anesthesia, is a medical symptom in which there is a
total or partial absence of the sense of touch. Anaphia is a common symptom of
spinal cord injury and neuropathy.
-Hyperesthesia is a condition that involves an abnormal increase in sensitivity to
stimuli of the sense. Stimuli of the senses can include sound that one hears, foods
that one tastes, textures that one feels, and so forth. Increased touch sensitivity is
referred to as "tactile hyperesthesia", and increased sound sensitivity is called
"auditory hyperesthesia". Tactile hyperesthesia may be a common symptom of many
neurologic disorders such as herpes zoster, peripheral neuropathy and
radiculopathies.
-Dysesthesia is defined as an unpleasant, abnormal sense of touch, and it may or
may not be considered as a kind of pain. It is caused by lesions of the nervous
system, peripheral or central, and it involves sensations, whether spontaneous or
evoked, such as burning, wetness, itching, electric shock, and pins and needles.
It is sometimes described as feeling like acid under the skin. Burning dysesthesia
might accurately reflect an acidotic state in the synapses and perineural space.
Some ion channels will open to a low pH, the acid sensing ion channel has been
shown to open at body temperature, in a model of nerve injury pain. Dysesthetic
burning may be hallucinatory as to the cutaneous surface, but accurate as to what is
occurring in the synapses.
Causes: Dysesthesia is commonly seen in diabetic patients, and can be relieved by
using creams containing capsaicin. Dysesthesia is among symptoms of neuropathy.
Dysesthesia, along with polyneuropathy can be a symptom of nerve damage caused
by Lyme Disease. Dysesthesia is a common symptom of a withdrawal from alcohol
or other drugs. Dysesthesia is also a common symptom of Multiple Sclerosis. It is an
effect of spinal cord injury.
107. Pain.
-Pain is defined as an unpleasant sensory experience, either physical or emotional,
associated with actual or potential tissue damage. From a medical point of view, pain
has always been referred to as the fifth vital sign.
-The pathophysiology of pain is usually divided into four distinct stages: (1)
transduction, (2) transmission, (3) pain modulation, and (4) perception. -The brain
can inhibit or facilitate the intensity and propagation of pain stimuli via specific neural
pathways. This modulation function of the brain accounts for the variations in pain

VSM
perception of different individuals who sustain identical injuries and receive the same
drug therapies. Transduction occurs at the sensory level, when a stimulus is
converted into a nerve signal. Transmission is the primary function of nerves that act
as conduits transferring pain information from the peripheral nerves to the central
nervous system. Pain modulation refers to the function of neural cells to inhibit,
reduce, or dampen the intrinsic modulatory activity of the central nervous system,
thus reducing the painful stimuli. Perception is the result of the three previous
components of transduction, transmission and pain modulation and is the conscious
awareness, usually localized in certain areas of the body. This is probably the most
recognizable component of this phenomenon since it is what the sufferer consciously
feels. Levels of pain perception depend on factors such as personal experiences,
immediate environment, and sociocultural influences. For example, if a person has
broken a limb before or has known of someone close to him who has broken an arm
or leg, he might perceive the pain level through the memories of his pain or the pain
of the other person. When people speak of pain thresholds, they may well be
referring to the pathophysiology of pain. It varies from person to person and has to
do with individual brains and their processing of pain.
108. Disturbances in consciousness, arousal & cognition. Dementia & Alzheimer’s
disease.
We become conscious of only a fraction of the information reaching our brain. The
conscious contents are stored in associative cortical areas that specialize in this task
. Conscious awareness requires not only that the specific afferents have been
transmitted to the cerebral cortex, but also nonspecific activation by the ARAS
through which neurons from the reticular formation activate wide areas of the
cerebral cortex via intralaminar neurons of the thalamus. Damage to large areas of
the cortex and/or breakdown of the ARAS brings about loss of consciousness. Loss
of consciousness can be divided into several stages: in a state of drowsiness the
patient can still be roused and will respond; in a stupor (profound sleep) patient scan
be awakened by vigorous stimuli; when in a coma this is no longer possible. In
socalled “coma dépassé” vital functions will also have ceased (e.g., respiratory
arrest.The split brain represents a special abnormality of consciousness . Uniform
consciousness presupposes communication between the two cerebral hemispheres.
This takes place along large commissural fiber bundles through the corpus callosum
and the anterior commissure.
Arousal Disturbiances. This is a state caused by withdrawal from alcohol or
barbiturates, acute encephalitis, head trauma resulting in coma, partial seizures in
epilepsy, metabolic disorders of electrolyte imbalance, Intra-cranial space- occupying
lesions, Alzheimer's disease, rabies, hemispheric lesions in stroke and multiple
sclerosis.Anatomically this is a disorder of the limbic system, hypothalamus,
temporal lobes, amygdala and frontal lobes. It is not to be confused with mania.
The philosophy of low arousal approaches is one of non-confrontation. In high-risk
situations, responses that reduce arousal are adopted by carers and staff, especially
when confronted by distressed individuals. These approaches became popular in
services for people with intellectual disabilities in the UK in the mid-1990s. This

VSM
humanistic and person-centred approach to crisis management was developed in
response to the use of restrictive responses to crises such as restraint, seclusion,
and chemical restraint. The avoidance of sanctions- and of consequence-based
punishment strategies became an implicit part of the approach.
The low arousal theory is a psychological theory explaining that people with
attention-deficit hyperactivity disorder (ADHD) seek self-stimulation or excessive
activity in order to transcend their state of abnormally low arousal.[1][2] The theory
states that one with ADHD cannot self-moderate, and his or her attention can only be
sustained by means of sustained external/environmental stimuli.[2] This results in an
inability to sustain attention on any task of waning stimulation or novelty, as well as
explaining compulsive hyperactive behavior. The low arousal theory is a
psychological theory explaining that people with attention-deficit hyperactivity
disorder (ADHD) seek self-stimulation or excessive activity in order to transcend their
state of abnormally low arousal.[1][2] The theory states that one with ADHD cannot
self-moderate, and his or her attention can only be sustained by means of sustained
external/environmental stimuli.[2] This results in an inability to sustain attention on
any task of waning stimulation or novelty, as well as explaining compulsive
hyperactive behavior.
when a person fails to be aroused in a situation that would normally produce arousal
and the lack of arousal is persistent, it may be due to a sexual arousal disorder or
hypoactive sexual desire disorder. There are many reasons why a person fails to be
aroused, including a mental disorder, such as depression, drug use, or a medical or
physical condition. The lack of sexual arousal may be due to a general lack of sexual
desire or due to a lack of sexual desire for the current partner. A person may always
have had no or low sexual desire or the lack of desire may have been acquired
during the person's life.
Cognitive disturbances: Cognitive disorders often are thought of as disorders of
elderly people. Although most common in this population, cognitive disorders can
occur at any age. Very young or very old people with cognitive disorders have
multiple health needs. Elderly clients usually have more than one chronic illness, and
psychiatric disorders can be accompanied by other comorbidities. Cognitive
disorders cause a clinically significant deficit in cognition that represents a major
change from the individual’s previous baseline level of functioning. • Two common
disorders are • Delirium • Dementiarbidities.
Cognitive disorders are a general medical condition, a result of substance use or
abuse, a reaction to medications or other ingested agents, or a combination of all of
these. In science, cognition: refers to mental processes. These processes include
attention, memory, producing and understanding language, solving problems, and
making decisions. Cognition is studied in various disciplines such as psychology,
philosophy, linguistics, science and computer science. Dementia is not merely a
problem of memory. It reduces the ability to learn, reason, retain or recall past
experience and there is also loss of patterns of thoughts, feelings and activities.
The causes of dementia depend on the age at which symptoms begin. In the elderly
population (usually defined in this context as over 65 years of age), a large majority

VSM
of cases of dementia are caused by Alzheimer's disease, vascular dementia or both.
Dementia with Lewy bodies is another fairly common cause, which again may occur
alongside either or both of the other causes. Hypothyroidism sometimes causes
slowly progressive cognitive impairment as the main symptom, and this may be fully
reversible with treatment. Normal pressure hydrocephalus, though relatively rare, is
important to recognize since treatment may prevent progression and improve other
symptoms of the condition. However, significant cognitive improvement is unusual.
Delirium is a very general and nonspecific symptom of organ dysfunction, where the
organ in question is the brain. Delirium may be caused by physical illness, which can
be mild, or any process which interferes with the normal metabolism or function of
the brain. For example, electric shock, fever, pain, poisons (including toxic drug
reactions), brain injury, hypoxia, anoxia, surgery, traumatic shock, lack of food or
water or sleep, and even withdrawal symptoms of certain drug and alcohol
dependent states, are all known to cause delirium. In addition, there is an interaction
between acute and chronic symptoms of brain dysfunction; delirious states are more
easily produced in people already suffering with underlying chronic brain dysfunction.
Dementia is a serious loss of global cognitive ability in a previously unimpaired
person, beyond what might be expected from normal aging. It may be static, the
result of a unique global brain injury, or progressive, resulting in long-term decline
due to damage or disease in the body. Although dementia is far more common in the
geriatric population, it can occur before the age of 65, in which case it is termed
"early onset dementia".[1] A recent survey done by Harvard University School of
Public Health and the Alzheimer's Europe consortium revealed that the second
leading health concern (after cancer) among adults is Dementia. Dementia is not a
single disease, but rather a non-specific illness syndrome (i.e., set of signs and
symptoms) in which affected areas of cognition may be memory, attention, language,
and problem solving. It is normally required to be present for at least 6 months to be
diagnosed. Symptoms of dementia can be classified as either reversible or
irreversible, depending upon the etiology of the disease. Fewer than 10% of cases of
dementia are due to causes that may presently be reversed with treatment. Causes
include many different specific disease processes, in the same way that symptoms
of organ dysfunction such as shortness of breath, jaundice, or pain are attributable to
many etiologies.Some of the most common forms of dementia are: Alzheimer's
disease, vascular dementia, frontotemporal dementia, semantic dementia and
dementia with Lewy bodies.
The occurrence of Alzheimer’s disease, the most common cause of (senile)
dementia (about 70%), is favored by a genetic disposition.However, the disease is
not genetically uniform. An especially severe form of the disease has an autosomal
dominant inheritance. Defects on chromosomes 1, 12, 14, 19, or 21 were found in
families with Alzheimer’s disease. Certain mutations of the !-amyloid precursor gene
promote the formation of senile plaques. Amyloid deposits can also occur under the
influence of other genetic or external factors. It is thought, for example, that toxins
can penetrate the brain via the olfactory nerves and cause the disease. Amyloid
deposits also occur in trisomy 21 (Down’s syndrome) that also leads to dementia. !-
amyloid fibrils can react with receptors at the cell surface, such as the receptor for

VSM
advanced glycation end products (RAGE), and a scavenger receptor (RA). Oxygen
radicals formed as a result may increase the neuronal intracellular concentration of
Ca2+ ("A3), possibly via depolarization of the cell membrane and activation of NMDA
receptors. The O2 radicals and Ca2+ promote cell death.
Cholinergic neurons in the basal nucleus of Meynert, in the hippocampus (especially
CA1, the subiculum) and in the entorhinal cortex are particularly affected by cell
death, but neurons also die in other cerebral areas, such as the frontal lobes,
anterior temporal lobes, parietal lobes, olfactory cortex, hypothalamus, locus
ceruleus, and raphe nuclei. Neuronal death is accompanied by decreased formation
and concentration of neurotransmitters in the brain. Acetylcholine is markedly
affected: in the cerebral cortex and the hippocampus there is an up to 90% decrease
in the concentration of choline-acetyl transferase, the enzyme that is necessary for
the formation of acetylcholine. The concentration of other neurotransmitters is also
reduced. A consequence of the degenerative changes is an increased loss of
cerebral functions. The disease typically begins insidiously with subtle deficits of
memory, neglect of appearance and body hygiene, phases of confusion, and taking
wrong decisions. As the disease progresses, anterograde amnesia will be followed
by impairment of past memories as well as procedural memory. Lesions in the limbic
system express themselves alternately through restlessness and lethargy. Motor
deficits (speech disorders, abnormal muscle tone, ataxia, hyperkinesia, myoclonus)
occur relatively late.
109. Epilepsy.
An epileptic seizure (epileptic attack, epileptic fit) is triggered by a spontaneous,
synchronized,massive excitation of a large number ofneurons, resulting in localized
or generalized activation of motor (fits or seizures), sensory(sensory impressions),
autonomic (e.g., salivation), or complex (cognitive, emotional) functions.The epileptic
seizures can occur locally, for example, in the left precentral gyrus in the area of
those neurons that control the right foot (partial seizure). They can spread from there
to the entire precentral gyrus (Jacksonian epilepsy). Clonic cramps may spread, as
in this example, from the right foot to the entire right half of the body (“Jacksonian
motor march”), the patient not necessarily losing consciousness. However, should
the seizures spread to the other side of the body, the patient will lose consciousness
(partial seizure with secondary generalization). Primary generalized seizures are
always associated with loss of consciousness. Neuronal excitation or the spread of
excitation to neighboring neurons is promoted by a number of cellular mechanisms:
The dendrites of the pyramidal cells contain voltage-gated Ca2+ channels that open
on depolarization and thus increase depolarization.In lesions of neurons more of
these Ca2+ channels are expressed. They are inhibited by Mg2+,while
hypomagnesemia promotes the activity of these channels (!A2). An increased
extracellular concentration of K+ reduces K+ efflux through the K+ channels, i.e., it
has a depolarizing effect and thus at the same time promotes the activation of Ca2+
channels. The dendrites of pyramidal cells are also depolarized by glutamate from
excitatory synapses. Glutamate acts on a cation channel that is impermeable to
Ca2+ (AMPA channel) and one that is permeable to Ca2+ (NMDA channel). The
NMDA channel is normally blocked by Mg2+. However, the depolarization that is

VSM
triggered by activation of the AMPA channel abolishes the Mg2+ block (co-operation
of the two channels). Mg2+ deficiency and depolarization thus favor activation of the
NMDA channel.The membrane potential of the neurons is normally maintained by
the K+ channels. A precondition for this is an adequate K+ gradient across the cell
membrane. This gradient is created by Na+/K+-ATPase (!A4). A lack of available
energy (e.g., due to O2 deficiency or hypoglycemia) impairs Na+/K+-ATPase and
thus promotes depolarization of the cell. Normally depolarizations are reduced by
inhibitory neurons that activate K+ and/or Cl–channels via GABA, among others
(!A5).GABA is formed by glutamate decarboxylase(GD), an enzyme that needs
pyridoxine (vitaminB6) as co-factor. Vitamin B6 deficiency or a reduced affinity of the
enzyme for vitamin B6(genetic defect) favors the occurrence of
epilepsy.Hyperpolarization of thalamic neurons can increase the readiness of T-type
Ca2+ channels to be activated, thereby promoting the onset of absences.
110. Stroke.
The two major mechanisms causing brain damage in stroke are, ischemia and
hemorrhage. In ischemic stroke, which represents about 80% of all strokes,
decreased or absent circulating blood deprives neurons of necessary substrates.
The effects of ischemia are fairly rapid because the brain does not store glucose,
the chief energy substrate and is incapable of anaerobic metabolism. Nontraumatic
intracerebral hemorrhage represents approximately 10% to 15% of all strokes.
Intracerebral hemorrhage originates from deep penetrating vessels and causes
injury to brain tissue by disrupting connecting pathways and causing localized
pressure injury. Ischemic Stroke The three main mechanisms causing ischemic
strokes are: (a) thrombosis, (2) embolism and (3) global ischemia (hypotensive)
stroke. All ischemic strokes do not neatly fall into these categories and the list of
entities responsible for unusual stroke syndromes is very long.
Atherosclerosis is the most common pathological feature of vascular obstruction
resulting in thrombotic stroke. Atherosclerotic plaques can undergo pathological
changes such as ulcerations, thrombosis, calcifications, and intra-plaque
hemorrhage. The susceptibility of the plaque to disrupt, fracture or disrupt or
ulcerate depends on the structure of the plaque, and its composition and
consistency. Embolic stroke (ES) can result from embolization of an artery in the
central circulation from a variety of sources. Besides clot, fibrin, and pieces of
atheromatous plaque, materials known to embolize into the central circulation
include fat, air, tumor or metastasis, bacterial clumps, and foreign bodies. Superficial
branches of cerebral and cerebellar arteries are the most frequent targets of emboli.
Most emboli lodge in the middle cerebral artery distribution because 80% of the
blood carried by the large neck arteries flow through the middle cerebral arteries.
Global ischemia causes the greatest damage to areas between the territories of the
major cerebral and cerebellar arteries known as the “boundary zone” or “watershed
area.” The parietal-temporal-occipital triangle at the junction of the anterior, middle,
and posterior cerebral arteries is most commonly affected. Watershed infarction in
this area causes a clinical syndrome consisting of paralysis and sensory loss
predominantly involving the arm; the face is not affected and speech is spared.

VSM
111. Stress and distress.

VSM

You might also like